Any Flashcards

Un-categorised Vivas

1
Q

What is an abscess?

A

Localised collection of pus in an enclosed tissue space.

Aetiology:

  • Pyogenic bacteria: Staphylococcus aureus, Streptococcus pyogenes and E.coli
  • Reaction foreign body e.g. needle, splinter

Pathogenesis:

  1. Pathogens or foreign material cause localised cell damage and release of cytokines.
  2. This triggers an inflammatory response attracting WBCs, particularly neutrophils.
  3. There is a build-up of pus which is a mixture containing cellular debris, WBCs and bacteria.
  4. Simultaneously, nearby cells form a capsule of tissue around the pus to prevent it from spreading.

PC:
Superficial (e.g. skin):
- Signs of inflammation: Redness, swelling, pain, heat
- Fluctuant mass

Deep (e.g. liver, brain, renal):

  • Systemic symptoms: Fever, anorexia, weight loss, fatigue
  • Abnormal organ function

Mx:
Incision and drainiage
- Superficial: Via scalpel
- Deep: Percutaneously via US/CT guidance
- Abx usually not indicated as it can’t perforate core of abscess. If perforated, consider empirical Abx (Varies on location, refer to local guidelines).

Examples:

  • Skin abscess
  • Liver abscess following portal vein pyema related to bowel leakage/peritonitis or billiary infection
  • Renal/perinephric abscess usually complicate urologic infection due to G-ve enteric bacilli or secondary haematogenous seeding
How well did you know this?
1
Not at all
2
3
4
5
Perfectly
2
Q

What is cancer staging?

A

System used to classify cancers to provide prognostic information and guide mx.

TNM Staging:

  • T: Extent of tumor invasion. Tx = cannot be found. T0 = occult (can’t be found). T1-4 = based of size and local invasion.
  • N: Nodal involvement. Nx = cannot be measured. N0 = no nodal involvement. N1-3 = stage increases with increasing number of nodes, presence of large/matted nodes, or nodes more distant from the primary tumor.
  • M: Mx = cannot be measured. M0 = cancer has not metastasized. M1 = metastatic disease present (usually implying haematogenous spread).

Note there are some malignancies that do not use TNM e.g. Gynaecological with FIGO system, Lymphoma with the Arbor system, Colon cancer which uses the dukes classification system.

Tumor Grading:
Different to staging and and provides quantification of cellular changes of cancer cells, assessed by microscopic analysis. Grades from X-4.

How well did you know this?
1
Not at all
2
3
4
5
Perfectly
3
Q

What is the purpose of “staging” a patient’s cancer?

A
  1. Assist with planning of treatment
    - Deciding whether aim is curative or palliative
    - Type of treatment that is most likely to be successful
  2. Gives an estimate of prognosis
  3. Provides common terminology in assessing cancers
    - Helps with communication between healthcare providers and researchers
    - Helps to evaluate and compare clinical trials
How well did you know this?
1
Not at all
2
3
4
5
Perfectly
4
Q

What modalities are used to stage a patient’s cancer?

A

The TNM criteria, explained further in the “What is cancer staging” viva, is often used and there are several modalities are used to determine the extent of tumor size/invasion, nodal involvement and metastatic spread including:

  • History and Physical examination:
    Assess for signs/symptoms of cancer and/or metastatic disease (e.g. weight loss, fever, night sweats).
  • Imaging:

Structural (X-ray, CT, MRI, Scopes)
Detects the presence of a tumor and to an extent it’s size/shape.
To increase sensitivity a contrast agent may be used.

Functional (e.g. PET scan, (18-FDG), Bone scan (Tc-99m)

  • Detect presence of metastatic disease bone and soft tissue.
  • False positives may result from infection, inflammation, or granulomatous disease.
  • Not useful in cancers with a low metabolic rate (e.g. prostate cancer that have a low uptake).

Biochemistry:

  • Tumor markers (e.g. Colorectal cancer – CEA, Pancreatic cancer – Ca125)
  • Physiological changes that tumor may cause (e.g. Liver Ca – deranged LFTs)

Histopathology:

  • Tumor biopsy or from tissue that has been surgically resected.
  • Grading: assess level of cellular differentiation, with a greater differentiation reflecting a faster, more aggressive cancer.
  • Lymph nodes to assess for metastatic spread.
How well did you know this?
1
Not at all
2
3
4
5
Perfectly
5
Q

What is neoplasia? Give examples of benign and malignant neoplasms.

A

Neoplasia is new, abnormal growth of tissue.

It involves unchecked cell proliferation due to a combination of: a) genetic and b) environmental exposures. It results in overactive growth pathways and underactive growth suppression pathways.

Classification:
1. Benign:
Abnormal cell growth that lacks the ability to invade local tissue or to metastasize.
Examples (-oma):
- Adenoma (Benign neoplasm of glandular epithelium e.g. Liver/Renal)
- Fibroadenoma (benign neoplasm of breast)
- Leiomyoma (benign neoplasm of smooth muscle)
- Fibromas (fibrous tissue)
- Lipoma (fat tissue)
- Papilloma (finger-like projections emanating from an epithelial surface)
- Polyp (mucous membrane)

  1. Malignant:
    Abnormal cell growth that has the potential to invade surrounding tissue and to metastasize.
    Examples (-carcinoma or -sarcoma)

Carcinoma (malignant neoplasm from epithelial tissue)
Adenocarcinoma (malignant neoplasm of glandular tissue)
Melanoma (malignant neoplasm of melanocytes)
Rhabdomyosarcoma (malignant neoplasm of skeletal muscle)

Features:

Benign

  • Growth: Slow
  • Border: Well-demarcated
  • Histology: Well differentiated (similar to tissue of origin)
  • Metastases: Absent

Malignant

  • Growth: Progressive, eratic (slow to rapid)
  • Border: Poorly defined/irregular
  • Histology: Poorly differentiated and anaplastic (little/no resemblance to tissue of origin)
  • Metastases: Frequently present
How well did you know this?
1
Not at all
2
3
4
5
Perfectly
6
Q

What is a fistula? Give some examples.

A

Definition:
A Fistula is an abnormal anastomosis between two epithelialised surfaces.

Aetiology:

  • Congenital
  • Injury/surgical
  • Infectious
  • Inflammation/IBD (e.g. Chron’s, UC)
  • Neoplasia
  • Radiation

Classification:

  • Complete: internal and external openings (i.e. extends from skin to an internal body cavity)
  • Incomplete: external (skin) opening and no internal (organ) opening.

Examples:

  1. Gastrointestinal
    - Between organ to another organ e.g. enteroenteral, cholecysto-duodenal, gastrojejunal.
    - Between organ and external surface e.g. anal/anorectal, biliary, pancreatic.
  2. Urogenital
    - Between vagina and organ (rectum) e.g. rectovaginal (enterovaginal)
    - Between bladder and organ e.g. vesicorectal, vesicointestinal, vesicovaginal.
  3. Vascular
    - Between artery and vein e.g. arteriovenous fistula
  4. Respiratory
    - Between trachea and oesophagus e.g. tracheoesophageal fistula.
Complications:
Specific to location, but may result in:
- Recurrent infections, sepsis
- Bowel or bladder incontinence
- Abnormal discharge/bleeding
- Pain
How well did you know this?
1
Not at all
2
3
4
5
Perfectly
7
Q

What is a stoma?

A

Natural or artifically created opening between a hollow organ and the external environment.

Examples:
- Natural e.g. mouth, nostrils, anus
- Artificial e.g.
Ileostomy:
Connects ileum with external environment through the anterior abdominal wall (usually right-sided).
Produces large amounts of loose stool, requires changing 4-6x per day.

Colostomy:
Connects the large bowel with the external environment through the anterior abdominal wall (usually left sided).
Produces soft and formed stools with more flatulence, active once per day.

Gastrotomy:
Connects the stomach via tube for nutritional support or gastric decompression.

Tracheostomy:
Surgically created opening in the neck leading directly to the trachea done: a) to bypass an obstructed upper airway, b) to clean and remove secretions from the airway, c) prolonged mechanical ventilation, d) to easily and safely deliver oxygen to the lungs.

Classification:
- Temporary (reversed after 8-10wks)
Rest for distal bowel involved in inflammatory process (e.g. surgery, IBD, abscess)
Emergency relief of distal bowel obstruction (e.g. sigmoid volvulus, colorectal ca)
Protect distal anastomoses
- Permanent
Required when no distal segment is remaining (e.g. resected bowel/anus)
If unable to swallow due to neurological disorder (e.g. cerebral palsy, MS) or oesophageal obstruction (e.g. stricture, atresia, Ca)

Surgical methods:
End:
One lumen, usually colon, brought to abdominal wall and sutured into skin.

Loop:
Loop of bowel is pulled out and sutured onto abdomen and two openings are created for stool and mucous.
Typically used when reversal is planned.

Complications
Early:
-Stoma necrosis, dehiscence or retraction

Late:
- Stenosis, parastomal hernia +/- incarceration

Any time:
- Infection, parastomal bleeding, bowel obstruction, skin irritation, psychosocial impact

Miscellaneous:

  • Not painful due to the lack of somatic innervation of gut
  • Recommended to eat low fibre diet
How well did you know this?
1
Not at all
2
3
4
5
Perfectly
8
Q

What are the potential complications of a surgical procedure?

A

Anaesthesia:
Anaphylaxis
Infection at cannula or epidural/spinal site

General: 
Minor trauma to teeth/throat from endotracheal tube
Hypotension, MI, stroke
Post-operative nausea/vomiting 
Aspiration +/- pneumonia 
Malignant hyperthermia
Epidural: (into epidural space)
Hypotension 
Unintentionally high block 
Spinal: (into cerebrospinal fluid) 
Headache 

Perioperative:
Damage to adjacent structures (haemorrhage, leak of GI contents, sepsis, nerve injury)
Haemorrhage
Laparoscopic to open surgery

Immediate post-operative: (<24hrs)
Basal atelectasis
Haemorrhage
Oliguria (low urine output due to inadequate fluid replacement)
Shock (due to, MI, PE, septicaemia, haemorrhage)

Early post-operative: (24hrs-1m)
Pain
Infection (SSI, sepsis, pneumonia, UTI) 
Wound failure (dehiscence) 
Fluid/electrolyte disturbance (dehydration, fluid overload, hyponatremia, hyperkalaemia) 
DVT/PE 
Paralytic ileus 
Acute renal failure (ATN/nephrotoxins)
Urinary retention
Delirium
Constipation 
Pressure sores 

Late post-operative: (>1m)
Adhesions (bowel obstruction, abdominal pain)
Incisional hernia
Keloid formation
Recurrent infections
Recurrence of reason for surgery (malignancy)

How well did you know this?
1
Not at all
2
3
4
5
Perfectly
9
Q

What are the risk factors for a superficial wound infection?

A

Surgical site infection:
Within 30 days of procedure
Superficial Involves skin and subcutaneous tissue only
At least one of the following features:
Purulent discharge
Positive culture
Evidence of inflammation (pain, erythema, swelling, tenderness, warmth).

Risk factors:
Pre-operative (patient factors):
Immunocompromised state:
Pre-existing infection – HIV/AIDS, carrier of Staphylococcus
Primary immunodeficiencies - CVID
Medications – immunosuppressants, corticosteroids, rituximab
Metabolic – malnutrition, DM, obesity, smoking
Extremes of age

Perioperative:
Type of surgical wound:
Clean: no open viscus, site not inflamed or contaminated
Clean-contaminated: viscus open (GIT, GUT, Resp) but minimal spillage
Contaminated: viscus open with significant spillage
Dirty: active infection with purulent exudate, due to traumatic wound (with retained foreign body or faecal contamination) or ruptured viscus/abscess
Surgical method:
Aseptic technique – surgical scrub, skin prep, sterile instruments, OT environment
Open vs laparoscopic
Duration
ABX prophylaxis
Insertion of surgical drain, prothesis
Type of suture and dressing used

Postoperative:
Glycaemic control
Dressing and cleaning wound
Hygiene practices from health care professionals and visitors

How well did you know this?
1
Not at all
2
3
4
5
Perfectly
10
Q

How do you treat a superficial wound infection?

A

Prevention (optimising risk factors):
Pre-operative: Consider ABX prophylaxis, smoking cessation, medications
Perioperative: Sterile technique, skin preparation with antiseptic (betadine, chlorhexidine)
Post-operative: aseptic technique for regular wound dressings, shower after 48hrs

Mx:
Assess vitals
Wound swabs for culture (+/- blood culture if evidence of systemic involvement or sepsis)
Open wound (remove sutures/clips)
Irrigate and drain pus with 0.9% NaCl
Clean dressings (fluid-absorbent, anti-microbial) with regular changes

Abx:
Consider If evidence of spreading cellulitis
Ensure wound swab for MCS prior to commencing 
Pathogens: 
Staphylococcus Aureus (often MRSA) - 
Coagulase-negative Staphylococci
Gram-negative bacilli
Enterococcus
Streptococcus species 
Anaerobes 
Selection:

Gram positive
Mild: Flucloxacillin 500mg PO, 6-hourly

Gram negative (e.g. post GI/genitourinary) 
Mild:  Amoxycillin + Clavulanate 875 + 125mg PO, 12-hourly

Severe
IV Flucloxacillin + Gentamycin

Suspected MRSA/hypersensitivity to penicillins:
Vancomycin IV

How well did you know this?
1
Not at all
2
3
4
5
Perfectly
11
Q

What are the principles of antibiotic prophylaxis for surgical patients?

A

[NEED TO IMPROVE}

Surgical antimicrobial prophylaxis refers to the use of antibiotics for the prevention of surgical site infections and does not include preoperative decolonisation or treatment of established infections. The principles of antibiotic prophylaxis are:

  1. Right indication:
    The benefit of prophylaxis must be weighed up against the potential risks of antimicrobial use, including allergic reactions, antibiotic associated C.difficile and antibiotic resistance.
    Antibiotic prophylaxis is not indicated for clean non-prosthetic procedures, nor for minor surgeries. It is likely to be indicated for procedures where:
    a) The incidence of surgical site infection tends to be high, for example colorectal surgery.
    b) The consequences of infection are significant, for example surgery with implanted materials such as arthroplasty and cardiac valve surgery.
  2. Right antimicrobial:
    a) The choice of antimicrobial is ultimately influenced by the surgical procedure and associated risk factors. It should provide coverage of the expected microbiological flora at the incision site. For the majority of procedures a first generation cephalosporin such as cefazolin (primarily covers gram positive organisms) remains the preferred antimicrobial for prophylaxis.
  3. Right dose and duration:
    When indicated, a single dose of antibiotics is sufficient for most procedures. This dose may be influenced by patient related risk factors such as age, renal function and weight.
4. Right route of administration:
Parenteral administration (intravenous or intramuscular) is the preferred route for antimicrobial prophylaxis. However there are exceptions including oral antibiotics for transurethral resections of the prostate.
  1. Right timing of administration:
    Most guidelines recommend that antimicrobial prophylaxis is given 60-120 minutes before incision.
  2. Appropriate ABX prophylaxis is required to reduce postoperative infections without increasing antimicrobial resistance.
How well did you know this?
1
Not at all
2
3
4
5
Perfectly
12
Q

Why do surgical wounds fail?

A

Wound dehiscence is the failure of a wound to close properly, includes:

  1. Simple
    - Skin wound alone fails secondary to impaired healing
  2. Burst abdomen
    - The separation of abdominal wall closure with protrusion of the abdominal contents due to raised intra-abdominal pressure or from surgical technical failure (poor suturing).

Most common cause is SSI – so essentially anything that causes an infection may cause wound failure.

Risk factors:
Pre-operative (anything that leads to immunocompromised state)
Increasing age 
Steroid use 
Smoking 
Obesity or malnutrition 
Chemotherapy/Radiotherapy 
Perioperative:
Emergency surgery 
Abdominal surgery 
Duration >6hrs 
Poor wound closure (poor suturing, incorrect suture) 
Poor aseptic technique
Post-operative: 
Increased pressure: 
Infection of wound 
Haematoma
Abdominal distension (e.g. cirrhosis) 
Excessive coughing 
Heavy lifting 
Hypoxaemia 
Prolonged ventilation
Poor tissue perfusion (e.g. post-operative hypotension, COPD)
Trauma 

Clinical features:
Visible opening of wound
Bleeding, discharge (serosanguinous fluid – pinkish colour)

Mx:
Wound site swab and culture
Debride necrotic tissue
Consider prophylactic ABX (indicated ASAP if burst abdomen)
Re-closure
Surgical: Re-suturing the wound using deep retention suture
Non-surgical: saline-soaked gauze packing or negative pressure wound therapy

How well did you know this?
1
Not at all
2
3
4
5
Perfectly
13
Q

What is the difference between tissue obtained by fine needle aspiration and a core biopsy?

A

FINE NEEDLE ASPIRATE

Fine 22G needle aspirates cells/fluid from lesion for cytopathology

Results: 
Inadequate sample 
Benign
Suspicious 
Malignant (C1-5)
Advantages:
Simple, fast technique 
On-site interpretation 
Inexpensive 
Low risk of complication

Disadvantages:
Requires training in preparation of quality smears
Cancer can be missed if cells not obtained in tissue sample.
Can’t tell if cancer is invasive or not.

CORE BIOPSY

Large bore needle (varying shapes/sizes) collects large tissue sample with architecture preserved for histopathology. Usually performed with guidance.

Results:
Similar to FNA
Histological tissue types 
In situ vs invasive 
Reliable grading 
Receptor status (breast cancer - ER, PR, HER2)

Advantages:
Higher specificity and sensitivity
Definitive histological diagnosis and gives tissue fragments allowing for architectural features of tissue.
Gives borders and not just cells, allows for assessment of microcalcification in breast cancer.

Disadvantages: 
Not immediate result 
Complications (haematoma, haemorrhage, pain, discomfort) 
More invasive w/ local anaesthetic 
More expensive
How well did you know this?
1
Not at all
2
3
4
5
Perfectly
14
Q

Please look at these arterial blood gas results: pH 7.30, Pa O2 66mmHg, Pa CO2 26mmHg, base excess - 6.5, lactate 9.4. What is a summary description of these results?

A
Reference ranges for ABGs: 
pH 7.35-7.45 
PaO2 80-100mmHg 
PaCO2 35-45mmHg 
HCO3- 22-26mmol/L
Base excess -2 to +2 mmol/L 
Lactate 0.5-2mmol/L
pH 7.3 = 7.3 acidosis
PaO2 66 = hypoxaemia
PaCO2 26 = low (likely compensatory)
Base Excess -6.5 = reduced (metabolic acidosis)
Lactate 9.4 = Lactic acidosis (high)

This ABG shows metabolic acidosis with partial respiratory compensation. The low PaO2 and high lactate
suggest cellular hypoxia and subsequent anaerobic metabolism has occurred.

How well did you know this?
1
Not at all
2
3
4
5
Perfectly
15
Q

Please look at these arterial blood gas results: pH 7.30, arterial blood gas results: pH 7.30, Pa O2 66mmHg, Pa CO2 26mmHg, base excess - 6.5, lactate 9.4. What causes a blood gas profile of this sort?

A

Classified into Type A or Type B depending on whether there is impaired tissue perfusion. Biggest clinical concern is a septic patient.

Type A:
Inadequate oxygen delivery triggers anaerobic metabolism, and thus excess lactate as a by-product. This is in response to:
Increased oxygen consumption
E.g. exercise, seizures
Tissue hypoperfusion
E.g. Sepsis, Hypovolemic shock, hypotension, cardiopulmonary arrest, acute HF, mesenteric ischemia
Impaired oxygen-carrying capacity
E.g. anaemia, severe hypoxemia (respiratory failure), carbon monoxide poisoning

Types B:
No impairment of oxygen delivery, but carbohydrate metabolism is disordered resulting in excess lactic acid production.
B1: Underlying disease
E.g. ketoacidosis, malignancy (leukaemia, lymphoma), HIV infection
B2: Iatrogenic/medication
E.g. Alcohol, metformin, anti-retroviral, beta-agonists (epinephrine), paracetamol
B3: Inborn errors of metabolism
Enzyme defects e.g. pyruvate dehydrogenase deficiency

In summary, as this is associated with hypoxaemia, it is type A lactic acidosis.

For reference:

Equation: Pyruvate + NADH + H+ 🡨(lactate dehydrogenase)🡪 Lactate + NAD+

NAD+ is required for glycolysis to continue and produces 2ATP per mole of glucose.

Lactate is metabolised in the liver (60%) and kidney (30%)

How well did you know this?
1
Not at all
2
3
4
5
Perfectly
16
Q

What is a Deep Vein Thrombosis?

A

Deep Vein Thrombosis is a venous thromboembolism which involves the formation of a blood clot in a major deep vein. Usually in the lower limb, abdomen or pelvis.
They typically begin to form just above and below a venous valve.

Clinical features: DVTs can be asymptomatic with only 1⁄4 cases presenting with classical clinical features.
These can present with local features of venous engorgement and stasis. These include limb swelling, pain,
erythema, warmth on palpation, mild fever and tachycardia.

They can be classified as distal (i.e. in the calf – posterior tibial, anterior tibial or peroneal veins) or proximal (i.e. popliteal, femoral, profunda femoris, common femoral or iliac veins).

Aetiology is considered with Virchow’s Triad (stasis, endothelial damage and hypercoagulability).

Differentials include: cellulitis, muscle/tendon injury, mass compressing venous outflow.

Well’s Criteria:
- Active cancer (treatment/palliation within 6m)
- Bedridden recently >3d or major surgery within 4w
Calf swelling >3cm compared to the other leg
- Collateral (non-varicose) superficial veins present
- Entire leg swollen
- Localised tenderness along deep venous system
- Pitting oedema confined to symptomatic leg
- Paralysis, paresis, or recent plaster
- immobilisation of the lower extremity
- Previously documented DVT
- Alternative diagnosis to DVT as likely or more likely (-2 points)

How well did you know this?
1
Not at all
2
3
4
5
Perfectly
17
Q

Why is a DVT Important?

A

[NEED TO IMPROVE}

Can be prevented with prophylaxis & mitigation of risk factors

  • It can lead to pulmonary embolism (PE) which is a major complication – approximately 10% of symptomatic patients die within 1 hour
  • Amongst patients diagnosed with PE mortality is approx. 10% at 2 weeks and approx. 20% at 1 year
  • DVT is a major cause of preventable complication and death after surgery
  • It can lead to common long-term complications (e.g. post-thrombotic syndrome – pain, swelling, discolouration and pigmentation, varicose veins and rarely venous ulceration) + Ischaemia and necrosis.
How well did you know this?
1
Not at all
2
3
4
5
Perfectly
18
Q

What are some of the risk factors for hypercoagulability?

A

Inherited:
• Factor V Leiden (most common), Protein C and S deficiencies, antithrombin deficiency, prothrombin
gene mutation
o Factor V Leiden- clotting factor is no longer broken down by activated Protein C
o Protein C and S are Vit K dependent and cleave and neutralise V and VIII
o Antithrombin is a cofactor of heparin and inhibits thrombin

Acquired:
• Malignancy, oestrogen (HRT, OCP), dehydration, sepsis, polycythaemia, smoking, antiphospholipid
syndrome, recent trauma or surgery, obesity, pregnancy, immobility, heparin induced thrombocytopaenia, polycythaemia vera, cancer therapy (tamoxifen, thalidomide, lenalidomide), heart failure, IBD, nephrotic syndrome

How well did you know this?
1
Not at all
2
3
4
5
Perfectly
19
Q

How can the risks of DVT be reduced in a surgical patient?

A

Pre-operative:
Optimise RFs
Cease OCP 6w prior
Correct thrombophilia deficiency
Delaying surgery until resolution of hypercoagulability (pregnancy, malignancy)
Lifestyle modifications – weight loss, exercise, smoking cessation

Perioperative:
Intermittent calf compression (inflatable cushions wrapped around calves)

Post-operative:
Mechanical:
Early and frequent mobilisation
Elevation of the legs to increase venous return
Thromboembolic Deterrent Stockings (TEDs) to increase venous return
Intermittent pneumatic compression
Foot impulse device
Anticoagulation prophylaxis:
Indicated based on initialised VTE risk assessment
Low Risk 🡪 Nil
Ambulatory, Nil RF, <30m surgery
Moderate 🡪 Consider
High risk 🡪 Enoxaparin 40mg D SC 5-10days
Multiple VTE RF, major surgery (trauma, joints, abdominal, >45m)

How well did you know this?
1
Not at all
2
3
4
5
Perfectly
20
Q

What is Virchow’s Triad?

A

Definition: Virchow’s triad are the three factors that are believed to contribute to thrombosis, these are hypercoagulable sate, blood stasis (alterations in blood flow), endothelial injury.

Hypercoagulability 
Hereditary
Inherited thrombophilia
Factor V Leiden mutation (most common) 
Prothrombin gene mutation
Antiphospholipid antibodies 
Protein C and S deficiency
Acquired: 
Age
Pregnancy 
OCP/HRT
Malignancy 
Obesity 
Surgery/trauma (especially of lower extremity) 
Previous VTE or arterial thrombosis 
Immobilisation 
Smoking 
Stasis 
AF
Immobility or paralysis 
Venous insufficiency or varicose veins 
Venous obstruction from tumour, obesity or pregnancy 
Left ventricular dysfunction 
Endothelial injury 
Trauma or surgery 
Atherosclerosis (smoking, obesity, poor cholesterol) 
HTN
Venepuncture 
Prior DVT
How well did you know this?
1
Not at all
2
3
4
5
Perfectly
21
Q

You are the general surgery resident on call. You are asked to review a 70-year-old man who is 72 hours post elective, uncomplicated colonic surgery. He has developed an acute confusion having been well and behaving normally since surgery. What are you going to do?

A

Phone Call:
• In what way is the patient acutely confused?
• Is there depressed level of consciousness? What is the GCS?
• What are the vital signs (including glucose)?
• Is there an obvious reason for the patient’s behaviour?
• Is the patient aggressive?
• What has been tried so far?
• Are staff or patients at risk or actually injured?
• What staff are there to help now?

I would go to review the patient, while I am walking to their bedside, I would consider the important differentials which include:

  • Delirium – acute, fluctuating impairment with inattention
  • Dementia – chronic, progression impairment with intact consciousness
  • Psychosis – thought content/ perception issues, not necessarily confused
  • Receptive or expressive dysphasia – difficulties comprehending or verbalising responses to questions

Initial Management
• If unresponsive start with ABCDE and call for senior help early
• If aggressive and risk of harm call security – 1st priority is safety
• Use de-escalation strategies: verbal, physical (if necessary), medical tranquilisation (last resort)
• Determine if the patient is delirious with the use of the Confusion Assessment Method (CAM)
• Ask the patient if they are in pain and treat appropriately
• Consider the causes of delirium, starting with major treats to life:
o Hypoxia
o Hypotension
o Sepsis
o Intracranial mass lesion
o Seizures

History - take a history from the patient and collect a collateral history from relative and ward staff.
• Review - the patient notes, observations & medication chart
• Physical Examination - perform a selective physical examination:

• Physical Examination - perform a selective physical examination:
o Vitals and GCS – also perform Abbreviated Mental Test Score (AMTS) or Quick Confusion Scale
o HEENT – nuchal rigidity, photophobia, pupil size and symmetry, fundoscopy, otoscopy
o Neuro – focal neurological deficit (esp. cranial nerve or motor deficit)
o CVS – JVP, S3, pitting oedema, new murmur
o Resp – cyanosis, respiratory distress, wheeze, crackles
o Abdo – NG tube output, costovertebral angle tenderness, guarding, rebound tenderness, jaundice, hepatomegaly, asterixis
o GU – Urine output from catheter

• Investigation - initial investigations:
o FBC, UEC, BSL, CMP, LFT
o Blood cultures (if sepsis considered)
o Urinalysis and urine MCS
o ECG & CXR
o Consider ABG (if respiratory cause considered
• If no cause found, consider further investigations

• Management
Treat any possible underlying cause identified:

Non-Pharmacological
This is the main treatment
Aim to re-orientate the patient
Close observation by a nurse in a quiet, well lit room
Have a set routine and use regular staff
Have clocks visible and give the patient their glasses or hearing aids
Have family present

Pharmacological
Try to avoid using sedatives e.g. benzodiazepines
As a last resort: 
Haloperidol 0.5mg PO or IM, single dose
Risperidone 0.5mg PO, single dose
Olanzapine 2.5mg PO or IM, single dose
How well did you know this?
1
Not at all
2
3
4
5
Perfectly
22
Q

You are the general surgery resident on call. You are asked to review a 70 year old man who is 72 hours post elective, uncomplicated colonic surgery. He has developed a fever of 38.2 degrees C, having had no fever recorded since surgery. What are you going to do?

A

Ask caller to recheck obs, then head over to the patient. While you are walking to the bed, consider differentials:

DDx:
Wind (Day 1-2)
Atelectasis (ventilator use, poor deep breathing)
Pneumonia (ventilator use)

Water (Day 3-5)
UTIs (catheter, urinary retention)

Wound (Day 5-7)
SSI, cannula infection

Walking (Day 5+)
DVT/PE

Wonder drugs (Anytime)
Anaesthesia
Transfusion productions
Anti-inflammatory agents

Sepsis (Anytime)
Infected foreign body (prothesis)
Leaking anastomosis

Resuscitation:
Vitals, hydration status
ABCDE
Inform surgical registrar

Hx:
Surgical Hx (pre/peri/post) 
Onset, progression, associated features 
RF for infection – SSI, cannula, prosthesis, catheter, intubation, immunosuppression 
Medications, allergies 
O/E:
Possible infectious sites: 
Wound, cannula, catheter, prosthesis
Looking for inflammation, pus, fluctuant mass, cellulitis etc. 
Cardiorespiratory 
Ix: 
Initial 
Pathology: FBC, CRP, blood culture, U/A + MCS, wound culture 
CXR
ECG 
Specific
Dependent on cause: 
D-dimer, troponins, CT, joint aspiration (gout), LP (meningitis) 
Mx: 
Supportive 
IV fluids, analgesia, antipyretic
Cease unnecessary medications
Remove unnecessary attachments 
Treat cause
Infection – Abx, drain abscess
DVT/PE – anticoagulation 
Atelectasis – incentive spirometry, deep breathing (improves over time)
How well did you know this?
1
Not at all
2
3
4
5
Perfectly
23
Q

You are the general surgery resident on call. You are asked to review a 70 year old man who is 12 hours post elective, uncomplicated colonic surgery. He has developed an acute episode of hypotension, which has persisted for 10 minutes. What are you going to do?

A
DDx:
Hypovolaemia (↓ Preload) 
Haemorrhage
Inadequate fluid replacement 
Cardiogenic (intrinsic pump failure) 
MI, CHF exacerbation, arrhythmia, cardiomyopathy, valvular disease

Distributive (↓ Afterload)
Sepsis (increased capillary permeability)
Anaesthesia related (inhibits sympathetic system causing bradycardia and hypotension)
Anaphylaxis
Obstructive
PE
Tension pneumothorax

Resuscitation:
Vitals, hydration status
ABCDE
Inform surgical registrar

Hx:
Associated Sx 
Review:
Operation report 
Fluid Balance 
Medications (diuretics, antihypertensives, analgesics) 
PMHx (IHD, arrhythmias, DVT/PE)

O/E:
Vitals
SIRS, dehydration, poor perfusion
Evidence of infection (inflammation, pus, fluctuant mass, cellulitis etc.)
Cardiorespiratory
Abdominal (fluid, tenderness, peritonitis)

Ix: 
Initial:
FAST scan (free fluid)
Group and save +/- cross match 
Septic screen – FBC, CRP, cultures, lactate 
CXR
ECG
Mx: 
Initial 
IV Fluids +/- electrolytes 
? HDU referral
Treat cause 
Haemorrhage – OT
Cardiogenic – cardioversion, anticoagulation etc. 
Infectious – Abx
PE – Anticoagulation 
Tension pneumothorax – chest decompression with needle thoracostomy
How well did you know this?
1
Not at all
2
3
4
5
Perfectly
24
Q

You are the general surgery resident on call. You are asked to review a 70 year old man who is 24 hours post elective, uncomplicated colonic surgery. His urine output has dropped from an average of 60mls per hour (>1/2ml per kg) in the first 24 hours to 20mls per hour for the last 2. What are you going to do?

A

Note: Normal UO is >1-2ml/kg per hr

DDx:
Pre-renal 
Hypovolaemia (haemorrhage) 
Hypotension (sepsis)
Low CO (HF)
Renal artery stenosis (atherosclerosis)
Renal 
Hypoperfusion -> ischemia
Glomerular disease 
Nephrotoxins (NSAIDs, contrast, aminoglycosides)
Metabolic derangements (hypercalcaemia) 
Post-renal 
Obstruction (stones, pelvic mass)
Urinary retention (neurological inhibition, overdistension, constipation)  

Resuscitation:
Vitals, hydration status
ABCDE
Inform surgical registrar

Hx:
Sx
Fever, pain, confusion, dysuria etc.
AKI – weight gain, fatigue, anorexia, nausea, confusion, seizure
Hypovolaemia – SOB, weakness, light-headedness, syncope
Review
Operation report (bleeds, complications)
Fluid Balance
Medications (diuretics, antihypertensives, analgesics)
PMHx (HTN, DM, CHF, PVD, CKD, renal stones etc.)

O/E:
Catheter 
Blockage, Kink
Urine appearance 
Vitals 
Evidence of shock 
Abdominal 
Palpable bladder (obstruction, retention) 
Ballotable kidney 
Tenderness or peritonitis 
Cardiorespiratory  
Ix: 
Initial
FBC, UEC, eGFR, ABG, Group and Save 
Urinalysis 
Bladder scan 
Specific
CTKUB
FAST scan 
ECG
Mx: 
Initial 
IV Fluid challenge 
500ml over 10-30m
Assess for improvement in BP, UO and mal-perfusion symptoms 
Stop nephrotoxic medication 
Treat cause 
Hypovolaemia – fluids
Urinary retention – catheterise 
Consider HDU if deteriorating
How well did you know this?
1
Not at all
2
3
4
5
Perfectly
25
Q

What are the symptoms and signs of a PE?

A

Highly variable, may include:

Symptoms: 
SOB 
Pleuritic chest pain (shoulder pain if diaphragm is irritated by lower lobe PE) 
Cough +/- haemoptysis (if infarction)
Shock (if severe) – dizziness, syncope
Signs: 
Tachypnoeic 
Tachycardic
Elevated JVP
Parasternal heave
Loud pulmonic component of 2nd heart sound (P2) 
Crackles (pleural effusion)
Shock – hypotensive (SBP <90), altered LOC, pale/mottled skin, central cyanosis
How well did you know this?
1
Not at all
2
3
4
5
Perfectly
26
Q

How is a PE diagnosed?

A

Approach is based on assessment of pre-test probability via Modified Well’s Score:

Modified Well’s Criteria
Clinical signs/symptoms of DVT (+3)
PE is #1 diagnosis OR equally likely (+3)
Heart rate >100 (+1.5)
Immobilization at least 3 days OR surgery in past 4 weeks (+1.5)
Previous, objectively diagnosed PE or DVT (+1.5)
Haemoptysis (+1)
Malignancy with treatment within 6 months or palliative (+1)

Low risk (<2)
Consider D-dimer
Negative – PE excluded
Positive – consider CTPA (CT pulmonary angiography)

PERC (PE rule-out criteria)
All negative – PE excluded as <2% chance

PERC (+1 per criteria)
Age >50 
HR >100 
SaO2 on RA <95% 
Unilateral leg swelling 
Haemoptysis 
Surgery or trauma <4wks 
Prior PE/DVT
Hormone use

Moderate risk (2-6)
High sensitivity D-dimer
Negative – PE excluded
Positive – CTPA

High risk (>6) 
CTPA (D-dimer not recommended) 

If patient hemodynamically unstable despite resuscitation:
Consider bedside lower extremity compression U/S to diagnose DVT to allow initiation of therapy
Refer to PERT (PE response team) who will consider risk/benefit of initating therapy

Findings:
CTPA
Embolism appear dark against contrast in pulmonary vessels
RV enlargement
V/Q scan
2nd line choice if eGF <30 i.e. renal failure or allergy to contrast
Inhale helium and inject contrast to compare V and Q

Other Ix:
Pathology
FBC, ESR, CRP, Troponin 
Elevated BNP
ABG 
Respiratory alkalosis (hyperventilation)
If massive with severe shock can be metabolic acidosis 
CXR
Usually normal, may exclude ddx 
Non-specific atelectasis and effusion  
ECG 
Sinus Tachycardia (most common finding)  
S1Q3T3 pattern (non-specific ST segment and T wave changes) 
T wave inversions in anterior and inferior leads 
RBBB
Right axis deviation
How well did you know this?
1
Not at all
2
3
4
5
Perfectly
27
Q

How is a PE treated?

A

Supportive:
A – Intubation if in CV collapse/arrest
B – High Flow O2
C – IV fluids, vasopressors (adrenaline/dobutamine), analgesia

Specific:
Severe/life-threatening:
Thrombolysis
Indicated if sustained hemodynamically instable patients
Absolute CI is bleeding, recent stroke
Alteplase 10mg IV bolus followed by 90mg IV infusion over 2h
If bleeds – FFP and anti-fibrinolytics
Embolectomy
Catheter insertion via femoral/brachial) to reach clot
Anticoagulation
Initial:
Enoxaparin (LMWH) 1.5mg/kg SC OD OR 1mg/kg BD
Ongoing: (3-6m or more)
Rivaroxaban (NOAC) – start immediately after diagnosis
OR
Warfarin – continuing LMWH for 5 days until in therapeutic range (INR 2-3)
Venous filter
Placed in inferior vena cava for 3-6months in patient with absolute CI to anticoagulation

How well did you know this?
1
Not at all
2
3
4
5
Perfectly
28
Q

What is a pulmonary embolism?

A

Definition: Obstruction of the pulmonary artery or one of its branches by material that originated elsewhere in the body.

Origin:
80% Thrombus from lower limb 
Proximal DVT embolises to PE in 50%
Distal DVT embolises to PE in 5% 
Other: tumour tissue, fat, air, septic (endocarditis of tricuspid or pulmonary valves),

Classification:
Massive: acute PE with obstructive shock or SBP <90mmHg
Sub-massive: acute PE without systemic hypotension but with either RV dysfunction or myocardial necrosis
Low risk: none of the above features

Pathogenesis
As per DVT, Virchow’s triad of hypercoagulable state + stasis of blood + endothelial injury
Thus, similar RF

Pathophysiology:
PE are typically multiple, with lower lobes being involved in majority of cases
Effects are proportional to the rapidity and degree of obstruction
Physiological responses:
Due to obstruction, there is Increased pulmonary vascular resistance (PVR) which impedes RV outflow. This causes RV dilation as it attempts to maintain adequate pulmonary flow. As it fails to compensate hypotension ensues.
Obstruction results in increased alveolar dead space, i.e. ventilation with no perfusion. This results in the V/Q mismatch that triggers pulmonary vasoconstriction to optimise gas exchange. This results in hypoxaemia, stimulating respiratory drive resulting in hypocapnia and respiratory alkalosis.
In 10% of cases, small thrombi lodge distally in segmental vessels resulting in pulmonary infarction, that presents as pleuritic chest pain and haemoptysis.

How well did you know this?
1
Not at all
2
3
4
5
Perfectly
29
Q

What are the factors that increase the risk of formation of a Deep Vein Thrombosis?

A

Relevant factors on:

Modified Well’s Criteria
Immobilization at least 3 days OR surgery in past 4 weeks (+1.5)
Previous, objectively diagnosed PE or DVT (+1.5)
Malignancy with treatment within 6 months or palliative (+1)

PERC (+1 per criteria)
Age >50 
Surgery or trauma <4wks 
Prior PE/DVT
Hormone use
How well did you know this?
1
Not at all
2
3
4
5
Perfectly
30
Q

What are the important contra-indications for a patient to have an epidural catheter placed for analgesia?

A

Absolute CI:
Hypovolaemia (as it may precipitate hypovolaemic shock)
Raised ICP (early morning vomiting and headache, papilledema, altered LOC, dilated unresponsive pupil)
Infection at site
Allergy to anaesthetic
Coagulopathy
- INR >1.4
- Platelet count <100
- LMWH within the last 12hrs (prophylactic dose) or 24hrs (therapeutic dose)
- Clopidogrel (within the last 7 days)
Patient refusal

Relative CI:
INR 1.2-1.4
Sepsis
Abnormal anatomy of spine
Severe aortic or mitral stenosis
How well did you know this?
1
Not at all
2
3
4
5
Perfectly
31
Q

What are the common or important complications and side effects of an epidural anaesthetic?

A

Catheter Related
1. Epidural abscess – Back pain, infection, possible neurological deficit, if so, then MRI.
Mx: Incision + drainage + fluclox

  1. Infection – sore, exudate, redness, swelling, fevers.
    Mx: Fluclox
  2. Epidural haematoma – back pain, tenderness, possible motor block, of so, then MRI. Can lead to compression + ischaemia of spinal cord, leading to paralysis.
    Prevention: Know coag status of pts, check site prior to administraion of anticoagulants.
    Management: Notify anaesthetists, CT/MRI + surgical evacuation.
  3. Inadequate block/no block – can feel ice cold or pain sensation.
    Mx: Retry
  4. Dural puncture – postural occipito-frontal headache, neck stiffness, photophobia, tinnitus, hearing disturbance.
    Mx: Supine position, hydration, analgesia
  5. Total spinal – ascending motor loss with LOC

Drug Effects
1. Local anaesthesia toxicity
Initial: Light headedness, drowsiness, tinnitus, circumoral tingling, metallic taste.
Undetected: convulsions, hypoventilation, arrhythmias, hypotension, tachycardia, arrest
Prevention: Aspirate catheter to check for blood, give block slowly.
Mx: Stop infusion, MET call, IV fluids, O2, airways and ventilation support, bag + mask PRN, anticonvulsants.

  1. High block (Unintentional anaesthetic into CSF) – Respiratory depression, weakness, bradycardia, hypotension, arm neuropathy.
    Mx: Stop infusion, MET call, IV fluids, O2, airways and ventilation support, bag + mask PRN, anticonvulsants, adrenaline.
  2. Sympathetic block: Hypotension, bradycardia, vasodilation.
    Prevention: position in L lateral, administer slowly.
    Mx: Stop infusion, IV fluid bolus, elevate legs, adrenaline.
  3. Opioid effects: sedation, respiratory depression, vasodilation.
  4. Sub-arachnoid opioid: N/V, severe itch, drowsiness -> coma, bradypnoea -> apnoea.
    Mx: Stop infusion, MET call, IV naloxone, IV fluids, O2, airways and ventilation support, ICU.
  5. Urinary retention anaphylaxis: Confusion, pain, full bladder.
    Mx: Catheterisation
How well did you know this?
1
Not at all
2
3
4
5
Perfectly
32
Q

What is the treatment of hypotension that is occurring in a patient who has an epidural anaesthetic in place? The patient is 12 hours post laparotomy at which a left hemicolectomy was done?

A

Consider causes of hypotension:
• Hypovolaemia- haemorrhage, insufficient fluid replacement/maintenance
• Cardiogenic: MI, arrhythmia
• Obstructive: saddle PE, tension pneumothorax, cardiac tamponade
• Distributive- sepsis, side effect of epidural, anaphylaxis
ABCDE assessment;
• Trial of fluid challenge 500mL N Saline
• Anaesthetics consult- consider turning down the epidural or stopping
Once stable:
• Review patients fluid balance and medications
• Treat any underlying cause as well as fluid balance
• Elevate legs
• Administer oxygen
• If refractory- get ICU involved to administer vasopressors

Jessica Redmond 29
NB:
A transient drop in blood pressure is to be expected as the sympathetic blockage produces vasodilation.
This is usually compensated for by an increase in cardiac output. If the sympathetic blockage should reach
T4 level, where the sympathetic chain to the heart leaves the spinal column the compensatory mechanism
is blocked preventing increase in heart rate. This can result in profound hypotension, bradycardia, thready
pulse or anxiety. A common problem as Epidurals tend to unmask the patients volume status.

How well did you know this?
1
Not at all
2
3
4
5
Perfectly
33
Q

What, chemically, is in a 1L bag of ‘Normal saline’? What is the average daily sodium requirement for a 70kg man?

A
Normal Saline:
0.9% NaCl 
1L H2O
Na 154 mmol 
Cl 154 mmol 

Daily requirements:
2.5L of H2O (30ml/kg/day)
Na 1-2mmol/kg
K 0.5-1mmol/kg

Daily requirements for 70kg man:
2.1L (approx.2.5L) of H2O
70-140mmol/kg/day Na
70mmol K

How well did you know this?
1
Not at all
2
3
4
5
Perfectly
34
Q

What is the adult daily sodium requirement? What is the daily potassium requirement?

A

The average daily intake of sodium is 1-2mmol/kg/day. The average daily intake of potassium is 0.5-
1mmol/kg/day.

How well did you know this?
1
Not at all
2
3
4
5
Perfectly
35
Q

What is the adult daily potassium requirement? How do you usually prescribe this for IV infusion?

A

The average daily intake of potassium is 0.5-1mmol/kg/day.
Potassium is prescribed as additive to other fluids. It can be given with 5% dextrose and 0.9% saline
30 mmol KCl additive to IL N Saline or 10mmol in 100 mL

How well did you know this?
1
Not at all
2
3
4
5
Perfectly
36
Q

What is the adult water volume requirement per day? How would you prescribe this in IV fluid orders to someone who is nil by mouth?

A

The adult water volume requirement is 25-30mL/kg/day

That is a maintenance of of 1500-2500mL/day

How well did you know this?
1
Not at all
2
3
4
5
Perfectly
37
Q

Please write out a fluid prescripiton for 24 hours “maintenance” fluids for a patient who weighs 80kg, has no unusual insensible or measurable losses.

A

Note: maintenance glucose = 50-100mg/day 1% =10mg/mL

How well did you know this?
1
Not at all
2
3
4
5
Perfectly
38
Q

What, chemically, is in a 1L bag of “Hartmanns” solution?

A
Hartmann’s: 
1L H20
Na 131mmol/L 
K 5mmol/L
Cl 111mmol/L  
Ca2+ 2mmol/L
Lactate (HCO3-) 29mmol/L (as lactate gets first pass metabolised into bicarbonate)
How well did you know this?
1
Not at all
2
3
4
5
Perfectly
39
Q

What, chemically, is in a 1L bag of 1/5 Saline, 4% dextrose?

A
4% and 1/5th NS: 
1L H20 
Dextrose 40g 
Na 30mmol/L
Cl 30mmol/L

5% dextrose:
1L H20
Dextrose 50g

How well did you know this?
1
Not at all
2
3
4
5
Perfectly
40
Q

What are the common or important immunological complications of a blood transfusion?

A

Immunological complications:
• Febrile non-haemolytic transfusion reaction
• Acute Haemolytic transfusion reaction
• Urticarial reactions
• Anaphylactic transfusion reactions
• Transfusion related Acute Lung Injury
• Post transfusion purpura
• Transfusion associated graft vs host disease
• Delayed haemolytic transfusion reaction

Febrile Non-haemolytic transfusion reaction
Caused by release of cytokines by WBCs in a product that has not been leukoreduced.
Presentation: within 1-6 hours
Characterised by fever, usually with chills in the absence of systemic symptoms.
Diagnosis of exclusion
Mx is symptomatic e.g. paracetamol for fever, cease transfusion, antihistamines
Prevention: leucodepletion of blood products

Acute Haemolytic Reactions
Life threatening reaction within first 24 hours caused by intravascular haemolysis of transfused RBCs when major ABO incompatibility.
Can lead to acute renal failure, DIC and haemodynamic collapse.
Presentation: fever, chills, flank pain, oozing from IV sites, chest pain, pink serum or urine.
Classic triad = fever, flank pain and red urine.
Treatment:
Immediate communication to transfusion service
Aggressive hydration and diuresis- Normal saline

Urticarial Transfusion Reaction
Urticaria but no other allergic findings. Antigen-antibody reaction- antigens in plasma interact with recipient’s pre-existing IgE antibodies. Mast cells and basophils then
release histamine
Presentation = wheal and itch. No wheezing, angioedema, hypotension
Mx:
Can continue remainder of blood product if no wheezing, angioedema, hypotension
after antihistamine.

Anaphylactic Transfusion Reaction
Various mechanisms:
• Recipients with IgA deficiency and IgG anti-IgA antibodies
• Anhaptoglobinaemia
• Passive transfer of food allergen

Presentation = rapid onset. Angioedema, wheezing, hypotension. May occur in IgA deficient individuals.
Management: cease transfusion, adrenaline repeat every 5 minutes as necessary IM, antihistamines, vasopressors, airway management.

TRALI
Life threatening occurs when recipient neutrophils are activated by the transfused products in short time or have underlying cardiovascular disease. Neutrophils release inflammatory mediators and damage pulmonary capillaries.
Presentation: fever, chills, respiratory distress, hypotension.
Mx: supportive, may include intubation and mechanical ventilation.

Post-transfusion Purpura
Occurs after transfusion of any platelet containing produce. Caused by platelet-specific alloatbodies.

Presentation–> 5-10 days post infusion
-bleeding
-thrombocytoapenia
Mx: high dose IVIg for several days

Transfusion Associated Graft vs Host Disease
Donor lymphocytes recognise recipient cells as foreign, initiating GvHD-
(usually, donor lymphocytes destroyed by recipients immune system)
Risk factors:
-immunocompromised pts
-specific type of partial HLA matching between donor and recipient

Presentation- 4- 30 days post op
• Fever
• Erythematous, maculopapular rash –> TENS
• n/v
• Abdo pain often RUQ
• Produse diarrhoea
• Cough
• Ix: pancytopaenia- hypocellular marow, deranged LFT, electrolyte abnormalities
Dx is supported by skin biopsy and HLA phenoytping
Mx- no effective treatment- almost always universally fatal
Prevent by irradiating products to inactivate T cells

Delayed Haemolytic Transfusion Reaction
Haemolytic tractions that occur more than 24 hours after completing the transfusion.
Often, they occur days to weeks later. They are typically gradual and less severe and
often clinically silent.
Autoantibodies to minor antigens e.g. Rh, Kell ,Duffy, Kidd.
Presentation: 2-30 days post transfusion.
-falling Hct, anaemia
Fever- mild
Unconjugated bilirubin increase
Mx:
No treatment required in the absence of rapid haemolysis
Future avoidance of implicated antigen (must report to patient)

How well did you know this?
1
Not at all
2
3
4
5
Perfectly
41
Q

What are the common or important non-immunological complications of a blood transfusion?

A
Non-immunological complications:
• Infection/Sepsis
• Transfusion related circulatory overload (TACO)
• Primary hypotensive reactions
• Iron overload

Infection/Sepsis
Caused by transfusion of a product that contains a microorganism.
Presentation: fever, chills, hypotension
Broad spectrum antibiotics and haemodynamic support.

TACO
Pulmonary oedema due to volume excess or circulatory overload.

Diuresis, supplementary oxygen.
Presentation: respiratory distress, rales, SOB, orthopnoea, PND, hypertension, Abnormal CXR, hypoxaemia.

Prevention: non rapid infusion rates
Drop in blood pressure without other causes of hypotension. Exclude TRALI, sepsis, anaphylaxis.
Most often reported in platelet transfusions.

Iron Overload
In chronically transfused patients- e.g. sickle cell, thalassemia, myelodysplastic syndrome.

How well did you know this?
1
Not at all
2
3
4
5
Perfectly
42
Q

What interventional (non-pharmaceutical) options are there for treating chronic (not acute) coronary artery insufficiency?

A

Definition: Occurs due to insufficient blood flow through one or more coronary arteries usually due to atherosclerotic disease.

Classification: 
Stable angina 
Retrosternal chest discomfort
Exacerbated by exercise
Relieved by rest or GTN 
May be silent in diabetics, elderly and females 
Acute Coronary Syndrome 
Unstable angina 
STEMI
NSTEMI

Indications:
Patient with chronic stable angina in which:
Failure of medical therapy
Maximum does not improve symptoms
Maximum intolerable
Significant risk factors on non-invasive testing:
Smoking
Obesity
Dyslipidaemia
Co-morbid DM, HTN
ST changes on stress ECG
Patient categories with better survival post-surgical intervention
i.e.
Stenosis of Left Main
Triple vessel disease (RCA, LAD and circumflex)
Two vessel disease (proximal LAD and another)

Method:
Percutaneous coronary intervention (PCI)
Indications:
1-2 vessel disease
Suitable anatomically
Low risk CVD
CI:
Anatomically unsuitable (long, near bifurcations)
Procedure:
Insertion via femoral artery
Coronary guidewire inserted into stenotic area and balloon catheter is pushed above.
Inflation results in plaque compression and stretches stenosis to allow stent placement
Requires dual antiplatelet therapy + heparin post-op
Cx:
Thromboembolism – MI, stroke
Access site – haematoma, pseudoaneurysm, perforation
Contrast induced AKI
Restenosis (1/3 in 6 months)
Stent thrombosis
Advantages:
Minimally invasive (better for comorbidities i.e. pulmonary disease)
Cheaper
Faster recovery time
Less adverse events
Superior for acute STEMI
Disadvantages:
High re-stenosis rates
Not suitable for 3 vessel disease or LMCA
Not suitable for non-compliant patients i.e. anti-platelets
Potentially incomplete revascularisation
Coronary Artery Bypass Graft
Indications:
Failure of medical therapy
Three vessel disease or two vessel involving proximal LAD
Failed PCI or restenosis with angina
High risk CVD
CI:
Unfit for surgery
Procedure:
Internal vein used to bypass stenotic artery
Most commonly long saphenous vein, radial artery, mammary artery
Must continue strict lifestyle modification/therapy
Cx:
Thromboembolic events – MI, stroke
Haemorrhage
Pericarditis
Graft failure
Advantages:
More complete revascularisation
More effective for multi-vessels
Better long-term results – lower risk of recurrent angina
Mortality benefit in diabetic patients
Disadvantages:
More peri-operative complications – death, stroke, GA, surgical risks
Longer recovery and hospital stay

How well did you know this?
1
Not at all
2
3
4
5
Perfectly
43
Q

What are the indications for a pneumonectomy in a patient who has been found to have a primary non-small cell lung cancer?

A
Indications: 
Extent
Stages:
1-2 considered operable 
3A limited patients operable
3-4 inoperable

Patient preference
Fitness
Co-morbidities that CI surgical Mx – cardiac disease, CCF, end-stage renal disease
Recovery and rehabilitation
Psychosocial impact
Predicted post-operative FEV1 >80%
Requires adequate respiratory reserve to tolerate pneumonectomy and maintain quality of life

CI:
CI in patients with disease extending past the diaphragm:
Intra-abdominal
Contralateral hemithorax
Invading structures of the mediastinum – aorta, IVC, heart, oesophagus
Ribs
Malignant disease

Surgical options:
Wide resection + mediastinal lymph node sampling
Segmentectomy
Lobectomy
Sleeve resection
Involves removing infected lobe and primary bronchus and then the remaining lobes are re-joined to the remainder of the primary bronchus.

Lung cancer: 
Non-small cell cancer (NSCLC) 85%
Adenocarcinoma 40%
Squamous cell carcinoma 25%
Large cell (undifferentiated) 10%
Small cell lung cancer (SCLC) 10%
How well did you know this?
1
Not at all
2
3
4
5
Perfectly
44
Q

What are the principles of a heart lung bypass perfusion circuit (machine)?

A

Function:
Temporarily replaces the function of the heart and lungs during surgery to maintain systemic circulation and oxygenation.

Method:
Removes blood from the intravascular space, commonly RA or SVC + IVC.
This is filtered through a membrane oxygenator which both oxygenates blood and removes CO2.
It is then passed through a heat exchanger and then a filter to remove air bubbles.
The blood then returns to the body via the ascending aorta, which is pumped under pressure.

Method: 
Pre-operative 
Anticoagulation – high dose Heparin 
CPB circuit primed with crystalloid, heparin +/- mannitol 
Systolic BP is lowered
Prior to cannulation SBP 80-100
During bypass MAP 50-70 
Cardioplegic solution administered (K+/Mg2+, procain) – decrease myocardial oxygen demand

During
Lungs are deflated by turning off ventilator
BP maintained with vasodepressors and dilators
Induce slight hypothermia (28-34) – prevent ischemia

Post-operative
Air excluded from cardiac chambers and aortic clamp removed
May require cardioversion or pacing if arrhythmia or heart block
Machine gradually switched off
Cannulas removed and anticoagulation reversed with protamine sulphate

How well did you know this?
1
Not at all
2
3
4
5
Perfectly
45
Q

What are the advantages and disadvantages of using a mechanical valve to replace an aortic or mitral valve, rather than a tissue valve?

A

Mechanical

Advantages:
Durability 
Sterile – decreased risk of infection 
Availability 
Simple procedure
Disadvantages:
Thrombogenic – increased VTE risk 
Lifelong Warfarin 
Bleeding risk 
Sometimes audible when open/close
Endocarditis

Tissue

Advantages:
Reduced VTE risk
Reduced anticoagulation need – decreased bleeding risk

Disadvantages:
10-15-year lifespan 
May need replaced – mortality risk 
Higher rates of infection 
Difficult procedure 

Therefore, mechanical valves are normally used in younger patients and tissue valves in older patients.

How well did you know this?
1
Not at all
2
3
4
5
Perfectly
46
Q

What are the indications for surgery in a patient who has been stabbed in the chest?

A

There are 3 urgent considerations for surgery:

  1. Cardiac Tamponade
    a. Dx: FAST scan
    b. Initial MX ED = pericardiocentesis and catheter
    c. Definitive Mx – thoracotomy – release tamponade, control haemorrhage
    i. Indications: pts with pulseless ECG activity
  2. Significant haemorrhage
    a. >1-1.5L initial loss or >200mL/hour ongoing for 2-4hrs
    b. Suggests major vascular wall injury unlikely to stop without surgical
  3. Massive air leak
    a. One present during all respiration – suggests major bronchial injury
  4. Flail chest

Other Considerations for Surgery
Direct injury
Ruptured blood vessel – endovascular stent or open repair
Aorta – if unstable BP emergency surgical repair
Trachea/bronchus – tube thoracotomy then open repair
Diaphragm – laparoscopic repair
Oesophagus – open repair
Pulmonary laceration – open repair
Haemothorax
PC: reduced chest expansion/breath sounds, dull to percussion
Mx: Emergency thoracotomy
Pneumothorax
PC: persistent air leak from chest tube
Mx: thoracotomy

Overview of Chest Wall Injury
- Many penetrating chest wall injuries do not require surgery and can be managed conservatively with:
- Serial CXRs
- Simple chest tube thoracotomy
(15-30% of chest wall traumas will require surgery)

Approach to Penetrating Chest Injuries

  • Primary survey and resuscitation
  • Detect and immediately treat life-threatening conditions such as:
  • Tension pneumothorax
  • Massive haemothorax
  • Open pneumothorax
  • Cardiac tamponade
  • Flail chest

Rapid diagnostic testing:

  • CXR
  • FAST Scan
  • ABGs
How well did you know this?
1
Not at all
2
3
4
5
Perfectly
47
Q

You are the ED intern on duty. You are asked to see and treat a 20 yoa man who has broken his nose playing football 3 hours previously. It has not stopped bleeding. You can ignore all other issues other than those related directly or indirectly to do with his nose. How will you proceed to assess and treat his nosebleed?

A

Assessment:
Primary survey (ABCDE)
Basic manoeuvre: patient sits upright, bends at hips, and applies digital pressure on cartilage for >10mins.
History
PC: duration, severity, localisation (nose/throat), causative factor (trauma, coagulopathy, medications, environmental, iatrogenic)
PMHx: previous bleed and method of control
Evidence of anaemia
Examination
GI: nasal symmetry, deviation, tenderness, associated injuries
Identify source of bleeding:
Nasal speculum +/- suctioning
To reduce pain o/e consider 5% lignocaine with 0.5% phenylephrine nasal spray
Posterior bleeding:
Suggested by failure to identify anterior, bleeding from both nares, visualisation of blood in posterior pharynx
Refer to ENT for rigid endoscopy

Mx:
Pain management
Cauterisation
Apply silver nitrate stick 10-15s (reacts with mucosal lining to produce chemical burn)
Never cauterize both sides of septum to avoid perforation
Nasal packing
Anterior
- Rapid Rhino
–Nasal tampon with inflatable balloon coated in a compound that acts as a platelet aggregator, may remain in place for 3-4days
–Ribbon gauze soaked in paraffin paste or Kaltostat
Posterior (if bleeding continues despite anterior packing)
- Foley catheter
Inserted along floor of nasal cavity into the posterior pharynx then inflated to sit in nasopharyngeal space. Re-insert anterior packing.

As increased risk of hypovolaemic shock – insert IV cannula, send bloods (FBC, coagulation studies, cross-match), commence IV fluids
Surgical options (refer to ENT)
Arterial ligation (SPA – most common, external carotid, internal maxillary)
Embolization (useful if failed SPA ligation or unfit for general anaesthesia)

For reference:
Definition: bleeding from the nasal cavity and/or nasopharynx (common due to rich vascular supply)

Classification:
Anterior (90%)
Located in anteroinferior nasal septum
Formed by anastomoses of four arteries “GASS” – greater palatine, anterior ethmoidal, sphenopalatine, septal branches of superior labial

Posterior
Located in inferior lateral nasal wall, posterior to inferior turbinate
Formed by anastomoses of two arteries – Sphenopalatine and pharyngeal
Greater risk of airway compromise, aspiration and uncontrollable haemorrhage

A vs P
Division occurs at piriform aperture (pear-shaped bony inlet of the nose formed by the nasal and maxillary bones)

How well did you know this?
1
Not at all
2
3
4
5
Perfectly
48
Q

You are a GP. You are consulted by a 55 yoa man who has a hoarse voice. What features of his history must you establish before deciding on your next steps?

A
History:
• Duration of voice complaints and whether the symptoms are constant or intermmittent
• Characteristics of onset
o Sudden or gradually progressive
o Pattern- worse on voice use, worse in morning
• Potential triggering factors
o Vocal abuse
o Concurrent URTI
o Change in medications
o Exposure to known allergens or toxins
• Exacerbating and ameliorating factors
o Improvement with voice rest
o Fatigue with use
• Other head and neck symptoms
o Dysphagia
o Otalgia
o Odynophagia (pain on swallowing)
o Bleeding
o Throat pain
o Post nasal drip
• History of smoking and alcohol use
• Red Flags- (when not a/w acute RTI)
o SOB
o Stridor
o Cough
o Haemoptysis
o Throat pain
o Dysphagia
o Odynophagia
o Weight loss
• Use of meds affecting voice
o Steroids - inhaled
• History of reflux or sinonodal disease
• History of past surgery involving neck, base of skull or chest
o Thyroid
o Carotid
o Cervical spine
• History of trauma and endotracheal intubation
• Occupation, hobbies, habits impacting voice use
• Medical comorbidities that can affect voice – rheumatoid, tremor, hypothyroidism

Following Hx I would also like:
• Physical examination
• Referral for ENT for laryngoscopy +- FNA and imaging
• Any patient with hoarseness >3 weeks needs CXR
• Indications for referral
o SCALD
o S- smoker, stridor
o C- constant/persistent hoarseness, coughing up blood
o A- acute onset (not related to URTI or EtoH)
o L- Loss (of weight)
o D- Dyspnoea or dysphagia
• Early referral and detection essential
o 70-90% 5 year survival, worse prognosis in stages 3-4
Ddx
Causes of hoarseness may be considered in the following categories:
• Acute laryngitis, which is self-limited and related to acute respiratory illness or acute voice misuse
o Moraxella catarrhalis, haemophilus influenzae, pneumococcus isolated from larynx
o Acute vocal strain – such as screaming or protracted coughing= can result in submucosal
microtrauma of the vocal fold
• Chronic laryngitis, which is related to irritants, reflux, chronic infection (such as fungal), or habitual
vocal misuse- beyond 3 weeks
o Irritants
o Laryngopharyngeal reflux
o Muscle tension dysphonia
• Benign vocal fold lesions
o Polyps and nodules
• Malignancy
• Neurologic dysfunction
o Unilateral or bilateral paralysis
o Parkinsons
o MND
o MG
o Tremor
• Non-organic (“functional”) issues

Laryngeal anatomy:

Function:
Phonation (production of a primary vocal tone at the level of the vocal folds)
Respiration
Swallowing (laryngeal elevation, posterior deflection of the epiglottis, inhibition of respiration, closure of the vocal folds to prevent aspiration of ingested material)

Gross:
Extends from the epiglottis to the inferior aspect of the cricoid cartilage
Divided into supraglottis, glottis (location of true vocal cords), subglottis

Supply:
Above vocal cords: superior laryngeal artery (branch of superior thyroid)
Below vocal cords: inferior laryngeal artery (branch of inferior thyroid)

Innervation:
Motor: recurrent laryngeal (branch of vagus), except for the cricothyroid muscle supplied by external laryngeal

Sensory:
Above vocal cords: internal laryngeal nerve (branch of superior laryngeal)
Below: recurrent laryngeal

How well did you know this?
1
Not at all
2
3
4
5
Perfectly
49
Q

You are the ED intern on duty. Your patient is a 30 year old man who has had a sore throat for 3 days, which is getting worse. He has difficulty opening his mouth, ear pain on the right and can’t swallow his saliva. On examination he has a temperature of 39 degrees celcius, pulse 115bpm. Oral examination reveals a unilateral, left sided tonsillar swelling with a diffuse, oedematous bulge superior and lateral to the tonsil. The uvula is deviated to the right.
What is the diagnosis?
What is the cause of the problem?

A

Dx: Peritonsillar abscess

DDx:
Epiglottitis: rapidly progressive, younger children
Retropharyngeal abscess: neck pain and tenderness, minimal peritonsillar findings
Parapharyngeal abscess: bulging posterior to tonsillar pillar, rather than superior
Tonsillopharyngitis: bilateral swelling, no trismus, no palpable fluctuance
Peritonsillar cellulitis: U/S, response to Abx

Clinical diagnosis confirmed by collection of pus on drainage. If diagnostic uncertainty U/S or trial of Abx may help.

Q2. Pathogenesis
Infective
Preceding tonsillitis or pharyngitis that progresses from cellulitis to phlegmon to abscess
Most common pathogens include S.Aureus, B-haemolytic strepoccoci, S.Pyogenes, H.influenza
Non-infective
Due to obstruction of the Weber glands (salivary glands in soft palate), associated with smokers

For reference: Peritonsillar abscess

Definition: Peritonsillar abscess (quinsy) is a collection of pus located between the capsule of the palatine tonsil and the pharyngeal muscles in the superior pole.

Epidemiology:
Most common deep neck infection
20-40s, M>F, smokers

Hx:
Unilateral sore throat +/- ipsilateral ear pain
Fever
Muffled “hot potato” voice
Dysphagia with pooling/drooling of saliva, odynophagia
Trismus (inability to open mouth due to inflammation and spasm of the internal pterygoid muscle)
Distinguishes PTA from severe pharyngitis or tonsillitis
O/E:
Swollen, erythematous, fluctuant tonsil (note: bilateral PTA is rare)
Deviation of the uvula to the opposite side
Cervical lymphadenopathy

Ix:
CT – fluid collection with large and inflamed tonsil

Cx:
Retropharyngeal effusion or abscess
Septic thrombophlebitis of internal jugular vein
Extension into surroundings 🡪 airway obstruction

How well did you know this?
1
Not at all
2
3
4
5
Perfectly
50
Q

You are the ED intern on duty. Your patient is a 30 year old man who has had a sore throat for 3 days, which is getting worse. He has difficulty opening his mouth, ear pain on the right and can’t swallow his saliva. On examination he has a temperature of 39 degrees celcius, pulse 115bpm. Oral examination reveals a unilateral, left sided tonsillar swelling with a diffuse, oedematous bulge superior and lateral to the tonsil. The uvula is deviated to the right.
What is the diagnosis?
What is the management?

A

Dx: Peritonsilar abscess (quinsy)

Mx:
Resuscitation (ABCDE):
Airway - ?ET intubation
Breathing – pulse oximeter, O2 as required
Circulation – IV access, bloods (FBC, UEC, culture), IV fluids
Immediate:
Analgesia
Empirical IV Abx – benzylpenicillin or clindamycin + metronidazole
Definitive:
Needle aspiration OR incision and drainage WITH MCS (outpatient or in OT)
Emergency tonsillectomy
Significant airway obstruction, recurrent episodes, failure of drainage
On discharge:
Follow-up within 7 days
Return if worsening or recurrence of symptoms
Oral Abx 10-14days

How well did you know this?
1
Not at all
2
3
4
5
Perfectly
51
Q

You are on-duty in A&E. Your next patient is a 3 year old child with rapidly progressive airway obstruction, looking toxic, drooling, tachycardic and tachypnoeic, sitting up leaning forward looking worried. The child is not immunised.
What is the most important differential diagnosis?
What will you do?

A

The most important differential diagnosis is epiglottitis.
Epiglottitis describes inflammation of the epiglottis and adjacent supraglottic structures. Without
treatment, epiglottitis can progress to life-threatening airway obstruction. It is often caused by
Haemophilus influenza type B (which is part of the immunisation program thus often only seen in
unvaccinated children).
Maintenance of the airway is the first priority for patients with epiglottitis. In patients with signs of total or
near-total airway obstruction, airway control should occur prior to diagnostic evaluation.
• Defer attempts at visualizing the epiglottis (tongue blade or any other instrument) or invasive procedures
(eg, IV placement, phlebotomy, or any other painful or frightening intervention) until after airway
assessment and management
• Prepare to manage the airway and immediately involve airway specialists (anesthesiologist or critical care
physician and otolaryngologist) whenever available
Patient not able to maintain the airway: Attempt bag-valve mask ventilation
• Unable to oxygenate (pulse oximetry lower than high 80s or falling): Attempt endotracheal intubation by
rapid sequence intubation first but be prepared to establish a surgical airway (eg, needle cricothyrotomy or
surgical cricothyrotomy)*
• Able to oxygenate (pulse oximetry high 80s and steady or improving): Endotracheal intubation by the
most capable provider, preferably in the operating with an otolaryngologist present
Patient able to maintain the airway:
• Provide supplemental humidified oxygen and maintain the child in a position of comfort with the parent
present (eg, sitting on the parent’s lap on the stretcher)
• Keep the patient in a setting where the airway can be rapidly managed if necessary with capable
personnel and specialized airway equipment constantly available
• Soft-tissue radiograph of the lateral neck (portable if possible) may be helpful but necessary personnel
and equipment to manage an acute airway event must remain with the patient at all times during the
imaging process.
• Do not image patients with severe respiratory distress in whom it will delay definitive airway
management
Radiographic findings of epiglottitis: Enlarged epiglottis (“thumb” sign), loss of vallecular air space,
thickened aryepiglottic folds, and/or distended hypopharynx
Attempts at direct visualization are only appropriate in patients with no stridor or stridor without
significant distress, no increase in symptoms with agitation, and no cyanosis
In children <6 years of age with confirmed epiglottitis or older children who are toxic-appearing or have
>50 percent obstruction of the lumen by direct visualization, perform endotracheal intubation in the
operating room with an otolaryngologist present
Under GA: examine throat + swabs, intubate, take blood cultures
ICU
Start IV Abx – ceftriaxone or cefotaxime.

How well did you know this?
1
Not at all
2
3
4
5
Perfectly
52
Q

Lana is a 52 year old female who is day 1 post total thyroidectomy. You are the RMO in-charge of the ward and have been called by the NUM to assess Lana urgently. Lana is having difficulty breathing and her neck looks significantly swollen. What is the most likely diagnosis? What will you do?

A

Post-operative cervical haematoma (usually in paratracheal area) – rare but life-threatening complication.

Management:
Call senior + transfer to OT
Resuscitation (ABCDE):
Airway
If non-critical may perform in OT, if critical perform at bedside
Establish airway:
Relieve pressure on airway by removal of sutures and evacuation of haematoma. Keep pressure without occluding trachea or carotids.

Attempt intubation
If all else fails, then cricothyroidotomy
Breathing – immediate high flow O2
Circulation - establish IV access, control haemorrhage

OT
Re-opening and inspecting for location of bleed
Place drain to remove fluid collecting in bed of haematoma
Transfer to ICU for observation

For reference: Post thyroidectomy haematoma

Description:
Haematoma is a collection of blood within the tissue
Due to major vessel ligature, re-opening of cauterized veins, bleeding from traumatized sternocleidomastoid muscle.
Causes life-threatening airway obstruction from direct tracheal compression and lymphatic/venous congestion

Presentation:
Commonly 12-24hrs post-op
Progressive neck swelling
Signs of airway obstruction
Respiratory distress (dyspnoea, stridor, anxiety, tachycardia)
Hypoxia (tachycardia, sweating, irritability, confusion)
Bleeding along suture line or significantly into drain
Transient vocal cord paralysis

How well did you know this?
1
Not at all
2
3
4
5
Perfectly
53
Q

Heather is a 4 year old girl who had a tonsillectomy 7 days ago. She presents to the ED with bleeding per orally. She has bled around 2 cupfuls of fresh blood in 30 minutes. You are one of the emergency department doctors. Heather is in waiting room.
What is the diagnosis?
What is your management plan?

A

Dx: Secondary (>24hrs) post-op haemorrhage

Occurs when fibrin clot (eschar) separates between day 5-10 exposing vessels to local trauma.
There is significant vascular supply to the tonsil (primarily from tonsillar branch of the facial artery) which predisposes it to haemorrhage.

Mx:
Admit
Resuscitation (ABCDE):
Airway – position patient forward, encourage spitting blood into bowel, suction as required
Breathing – O2, OBS
Circulation – IV access, bloods (FBC, coagulation studies, G&S), IV fluids
Immediate:
Urgent ENT review
IV analgesia
NBM
24hrs monitoring
Definitive:
Examine oropharynx for source of bleeding – visualise greyish white eschar
Apply pressure to tonsillar fossa with gauze soaked in lignocaine and adrenaline
If non-resolving 🡪 OT:
Electrocautery or silver nitrate
Topical haemostasis (topical thrombin, fibrin sealant)
Suture ligation
Surgical embolization

How well did you know this?
1
Not at all
2
3
4
5
Perfectly
54
Q

45 year old male comes to your GP practice with a 2 month history of hoarse voice. He has recently developed a swelling on the left side of his neck. He is a heavy smoker and consumes substantial alcohol daily. You are the doctor examining him. What is the most likely diagnosis and examination findings and what are relevant investigations/referrals for him?

A

Dx: Squamous cell carcinoma (SCC) of the larynx

O/E:
General observation
Hoarseness or vocal change
Respiratory distress, noisy breathing
Cachexia, weight loss
Head and neck
Ear: otalgia
Oral cavity and oropharynx: masses or leucoplakia
Laryngeal inlet: lesions of vocal cords, tumours
Neck: cervical lymph nodes or masses (size, location, shape, mobility, firmness, tenderness)
Respiratory
Signs of distress: stridor, diminished breath sounds, obstruction

Refer to ENT for Ix
Urgent laryngoscopy
Visualise lesions and document vocal cord mobility
Imaging (assess local infiltration, LN involvement, presence of Mets or secondary tumours)
CT contrast neck and chest
Whole body PET/CT scan
FNA of neck mass or cervical LN (under U/S guidance)
Rigid direct laryngoscopy with biopsy (FNA or excisional) under GA

For reference: Laryngeal cancer

Background:
Laryngeal cancer approx. 20% of all cases of head and neck cancer
90-95% are SCC
Glottic (59%) > Supraglottic (40%) > subglottic (1%)
Metastatic patterns:
Larger (especially if located in supraglottis) spread lymphatically to the neck
Lungs most common site for metastases
Prognosis: If localised 5-year survival 77%

Risk factors:
M>F
>40y/o
40 pack year, EtOH
Chronic viral infection – HPV, HIV

Pathophysiology:
Chronic tobacco exposure irritates the epithelial surfaces causing pre-malignant changes of hyperplasia and dysplasia, pre-disposing the area to malignancy.
If alcohol is consumed as well, it more than doubles the risk compared to exposure to one factor alone.

Presentation:
Hoarseness >3wks (early in glottic, late in supraglottic and subglottic)
Dysphagia, odynophagia
Otalgia (referred ear pain via CN X – suggests advanced disease)
Cervical lymphadenopathy (poor prognosis – may be only sing of supraglottic lesion)
Stridor, dyspnoea
Haemoptysis, chronic cough
Unexplained weight loss, fatigue, night sweats

DDx:
Chronic Laryngitis
Polyps on the vocal cords
Reinke’s oedema 
Contact granuloma
HPV papilloma’s
Benign tumours (rare)
How well did you know this?
1
Not at all
2
3
4
5
Perfectly
55
Q

45 year old female comes to your GP practice with a 1 year history of a midline neck swelling. She also gets tremors and says that her heart races at times and has lost some weight that she is pleased about.
What are the relevant investigations that are related to this case?

A
Pathology:
Thyroid function tests (TFTs)
Decreased TSH 
Increased free T4, free T3
Thyroid autoantibodies (positive) 
Thyroid peroxidase (TPO) antibody 
TSH-receptor antibody 
Other: 
FBC (normochromic normocytic anaemia, mild leukopenia)
LFTs (elevated ALT and AST)
Hypercalcemia  

Imaging:
Radionuclide thyroid scan (Technetium-99m)
Normal or elevated uptake
Homogenous: graves
Heterogenous: toxic multinodular goitre
Decreased uptake: thyroiditis, exogenous thyroid hormone
Thyroid U/S +/- FNA
Indicated if suspicious nodules (i.e. macrocalcification, irregular border, increased vascularity)
ECG (document any arrhythmias or other changes)

For reference: Hyperthyroidism

Causes:
Autoimmune (Grave’s disease) – most common
Thyroid-stimulating antibodies (IgG) bind to TSH receptors causing excess thyroid hormone secretion. Over time the thyroid hypertrophies and becomes diffusely enlarged.
TSH-secreting pituitary adenoma
Toxic multinodular goitre
Thyroiditis
Exogenous intake of thyroid hormones

Clinical features:
Increased MR: weight loss, heat intolerance, sweating
CNS overactivity: tremor, nervousness, irritability, lethargy
Beta-adrenergic sympathetic overactivity: palpitations, tachycardia, arrhythmias
Eye signs: soft tissue oedema, exophthalmos, diplopia, ophthalmopathy

Thyroid storm (thyrotoxic crisis):
Precipitated by non-thyroid surgery, major trauma, infection, imaging with iodinated contrast
Features: insomnia, anorexia, vomiting, diarrhoea, sweating, fever, tachycardia

How well did you know this?
1
Not at all
2
3
4
5
Perfectly
56
Q

Liam was involved in a fight yesterday at the local pub and was slapped in the face. He had a sharp shooting ear pain following the slap and says that he cannot hear too well since then. He also says that he cannot pop his ear drums since that incident. He comes to your practice.
What is the most likely diagnosis and what will you expect from the examination point of view?
What will be your advice?

A

Traumatic perforation of tympanic membrane

O/E:
Anatomic inspection:
External ear: Haematoma, bruising, trauma, discharge
Otoscopy:
TM perforation (central or marginal – unsafe as allow skin invasion into middle ear)
Blood crust
Physiological assessment:
Hearing
Positive Rinne’s test (conductive: bone conduction > air)
Weber’s test (conductive: loudest on ipsilateral side to affected ear)
Facial nerve (CN VII) palsy
Symmetry, facial movements, taste

Advice:
Most perforations heal spontaneously within 8wks
Keep dry and clean to avoid infection (i.e. use plug when showering)
If signs of infection (pain/discharge) 🡪 Abx
Follow-up in 4-6wks
If not improved, refer to ENT for surgical management (tympanoplasty)

For reference: Rupture tympanic membrane

Causes:
Blunt force trauma to external ear canal (car accident, blow to side of head, explosion)
Penetrating trauma (cotton buds, iatrogenic during irrigation or FB removal)
Barotrauma (air travel, scuba diving)

Red flags:
Significant polytrauma
Battle’s sign (mastoid ecchymosis indicating skull base fracture)
Facial nerve palsy

Presentation:
Sudden severe pain (otalgia)
Serosanguinous drainage (fresh blood leakage)
Conductive hearing loss (sound does not reach the inner hear)
Tinnitus (ringing or buzzing in ears)
Fullness in the ears (aural fullness)
Disequilibrium/imbalance

How well did you know this?
1
Not at all
2
3
4
5
Perfectly
57
Q

You are working as rural relief resident in a remote town GP practice. A 12yo aboriginal boy is brought in by his mum to see you as he is complaining of poor hearing in his right ear that is getting gradually worse. He gives a history of constant offensive discharge from the same ear with frequent infections. You look into his ear and see some dry white crusting wax at the top of his ear drum.

A

Cholesteatoma (abnormal growth of epithelium in the middle ear that results in bone erosion and infections)

Actions:

Initial workup:
Otoscopy: (confirm diagnosis)
Attic (posterosuperior portion of TM) retraction containing keratin
Migration of keratin through perforated TM
White crust within or behind intact TM
Audiometry (assess hearing loss)
CT temporal bone (assess for extent and complications)
Ear swab of discharge

Mx:
Immediate: 
Analgesia (paracetamol/NSAID)
Topical Abx +/- wick
Definitive: 
Refer to ENT
Mastoidectomy
Canal wall up (closed) – heals better, but risk that disease will recur
Canal wall down (open) – decreases risk of recurrence, will need to wear earplug when swimming and have wax cleaned 
Tympanoplasty 

For reference: Cholesteatoma

Causes:
Eustachian tube dysfunction (from allergies, sinusitis, chronic middle ear infections)
Perforated TM due to infection of injury
Congenital remnant of skin trapped in middle ear
Implantation of keratinocytes following otologic surgery or traumatic blast injury

Presentation:
Hearing loss or tinnitus
Chronic offensive discharge
Otalgia, vertigo, facial nerve palsy

How well did you know this?
1
Not at all
2
3
4
5
Perfectly
58
Q

You are working in emergency and you see a 4yr old boy with fever, left otalgia and acute left facial nerve palsy.

a) What is the most likely diagnosis?
b) What condition could this develop into that causes swelling behind the ear and the ear to stick out/forwards?

VIVA II

a) What is the most likely diagnosis?
b) How would you manage this boy?

VIVA III

a) What is the most likely diagnosis?
b) What dangerous complications can result from this condition?

A

VIVA I
Dx: Acute Otitis media

The condition that this could develop into that causes swelling behind the ear and the ear to stick out/forwards is Mastoiditis – a bacterial infection of the mastoid cells surrounding the inner and middle ear, causing swelling behind the ear and the ear to stick out or forwards.

There will be oedema, erythema and tenderness over the mastoid process.

This is also managed with analgesia and antibiotics.

VIVA II
Dx: Acute Otitis media

Management:

  1. Acute pain relief with analgesia (usually paracetamol +/- ibuprofen)
  2. Antibiotics (usually penicillin/amoxycillin. Refer to local guidelines).
  3. As the patient will likely have an ongoing viral respiratory infection, the patient should also be encouraged to drink plenty of fluids and rest.
  4. ENT referral for surgical mx

VIVA III
Dx: Acute Otitis media

Dangerous Complications:
Intratemporal complications that can result include:
1. Hearing loss
2. Balance and motor problems
3. Tympanic membrane perforation
4. Cholesteatoma
5. Mastoiditis
6. Seventh cranial nerve palsy

Other complications include:

  1. Meningitis
  2. Epidural abscess
  3. Subdural abscess
  4. Brain abscess
  5. Lateral sinus thrombosis
  6. Cavernous sinus thrombosis
  7. Carotid artery thrombosis

For reference: Acute otitis media

Clinical features:
Ear:
1. Otalgia (ear pain) 
2. Postauricular tenderness
3. Decreased hearing
  • Physical examination will reveal a bulging, opacified tympanic membrane with decreased mobility.
  • The membrane may be white, yellow, pink or red.
  • Diagnosis is generally made with conventional otoscopy

Resp:
1. Usually ongoing viral respiratory infection

Neuro:
1. Facial nerve palsy

Systemic:

  1. Irritability/Lethargy
  2. Fever

GI:

  1. Anorexia
  2. Nausea/Vomiting

Protruding auricle/external auditory canal oedema

Ix (not routine):
CT -> bony destruction of mastoid air cells

Pathophysiology:
Preceding event (e.g. URTI) -> inflammatory response. 
Inflammatory oedema obstructs the isthmus in the eustachian tube -> accumulation of secretions in middle ear space -> viral/bacterial growth.

Pathogens:
Viral (most common): RSV, Rhinovirus, enterovirus
Bacterial: S.pneumoniae (50%), H.influenzae (45%), Moraxella, GAS

How well did you know this?
1
Not at all
2
3
4
5
Perfectly
59
Q

You are the surgical ward resident. A nurse calls you saying that a patient who had frontal sinus
surgery this morning had a fall in the toilet. They have also been bleeding intermittently throughout the
day. What will you do?

A

While I am still on the call, I would ask the nurse to check for any surrounding dangers, rechek the patient’s observations, and ensure their immediate stability.

I would urgently review this patient as there is a concern this patient may be hypovolaemic due to post-operative blood loss.

Whilst on my way to the patient's bedside, I will be considering differentials categorised by system:
Cardiovascular System
-Hypovolaemia
- MI
- Arrhythmia
- Fluid imbalance or acid/base imbalance
- Exacerbation of valvular pathology

Neurologic System

  • Vasovagal or micturition syncope
  • CVA
  • Delerium
  • Seizure
  • Also consider psychiatric aetiology

Respiratory System
- PE

Musculoskeletal System

  • Muscle weakness
  • Functional issue e.g. slippery surface/severe pain

Endocrine
- Hypoglycaemia

Once at the patient’s bedside, or rather toiletside, I would begin by ensuring their:
1. Airway is patent and assess for signs of bleeding
2. Breathing and Circulation are adequate
Look for chest rise, whilst listening and feeling for breath sounds with my ear and assess fluid status by checking pulse, cap refill and checking urine output.
3. Checking for disability. GCS (pupils, limb strength, motor/sensory assessment, focal deficit assessment)

  1. Checking exposure
    Examine for injuries sustained during fall (head/neck, fracture, bruises/lacerations)
Secondary survey:
Hx
- Mechanism of fall 
- Precipitants/preceding symptoms (straining, constipation, standing, dizziness, palpitations, SOB)
- LOC, time to recovery 
- Witnesses? 
- Subsequent pain/concerns

O/E

  • Cardiorespiratory
  • Neurological + cranial nerve
  • Surgical site? bleeding

Review:

  • OBS
  • Operation report
  • Progress notes
  • Fluid balance
  • Medications (particularly anti-hypertensives)
  • Consider ENT review

Ongoing monitoring:

  • OBS hourly for 4hrs
  • Observe for headache, nausea, vomiting or changes in behaviour
  • Observe for occult bleeding: flank pain, increased intra-abdominal pressure, swelling, bruising
  • Review falls risk (now high) and risk prevention strategies

For reference: Sinus surgery

Indications for frontal sinus surgery:
Chronic rhinosinusitis e.g. ethmoidectomy
Cerebrospinal fluid leak
Benign and malignant tumours of the frontal sinuses

Complications:
Open/external: osteomyelitis, ocular muscle injury, haemorrhage, CSF leak, meningitis
Endoscopic: Epistaxis, bruising, dental pain, brain abscess, haemorrhage, CSF leak, meningitis

Post-op advice:
Avoid straining or blowing nose for 10days
Do not recommence aspirin or NSAID until 2wks post-op
Avoid steroid nasal spray for 2wks
Nasal saline sprays every 2-3hrs
Sinus irrigation twice daily
Normal to have intermittent blood discharge first 3-5days after surgery

How well did you know this?
1
Not at all
2
3
4
5
Perfectly
60
Q

You are working in emergency and you see a 4yr old boy with fever, left otalgia and acute left facial nerve palsy. What is the most likely diagnosis?

What condition could this develop into that causes swelling behind the ear and the ear to stick out/forwards?

A

The most likely diagnosis is Acute Otitis Media
• May present with otalgia, irritability, decreased hearing, anorexia, vomiting, or fever, usually in the presence of an ongoing viral respiratory infection.
o Risk factors: male, young age, exposure to respiratory viruses (day care attendance, older siblings),
positive family history, aboriginal
• Physical examination will reveal a bulging, opacified tympanic membrane with decreased mobility. The membrane may be white, yellow, pink, or red.
• Diagnosis is generally made with conventional otoscopy. Additional tests might include pneumatic otoscopy or tympanometry to confirm the presence of an effusion.
• Treatment includes pain control with analgesics and might include antibiotics.
• Complications include perforation of the tympanic membrane and, rarely, mastoiditis, seventh cranial nerve palsy, or sigmoid sinus thrombosis.

The condition that could develop causing swelling behind the ear and the ear to stick out/forwards is acute mastoiditis.
• There is oedema, erythema, and tenderness over the mastoid process.
• Because the mastoid air cells are connected to the distal end of the middle ear through a small canal or antrum, most episodes of AOM are associated with some inflammation of the mastoid
• Also managed with analgesia and abx

How well did you know this?
1
Not at all
2
3
4
5
Perfectly
61
Q

You are working in emergency and you see a 4yr old boy with fever, left otalgia and acute left facial nerve palsy. What is the most likely diagnosis?

How do you manage this boy?

A

The most likely diagnosis is Acute Otitis Media
Management:
• Oral analgesia
o Pain control is central to the management
o Paracetamol/ibuprofen
• Antibiotic therapy with penicillin (amoxicillin)

How well did you know this?
1
Not at all
2
3
4
5
Perfectly
62
Q

You are working in emergency and you see a 4yr old boy with fever, left otalgia and acute left facial nerve palsy. What is the most likely diagnosis?
What dangerous complications can result from this condition?

A
The most likely diagnosis is Acute Otitis Media
Complications that can result include:
• Intratemporal complications
o Hearing loss
o Balance and motor problems
o TM perforation
o Choleastoma
o Mastoiditis
• Intracranial complications
o Meningitis
o Epidural abscess
o Brain abscess
o Lateral sinus thrombosis
o Cavernous sinus thrombosis
o Subdural empyema
o Carotid artery thrombosis
How well did you know this?
1
Not at all
2
3
4
5
Perfectly
63
Q

You are the surgical ward resident. A nurse calls you saying that a patient who had frontal sinus surgery this morning had a fall in the toilet. They have also been bleeding intermittently throughout the day.
What will you do?

A

Urgently go to review the patient, concerned that patient may be hypovolemic due to post-op bloodloss
ABCDE
Response – patient conscious or unconscious, alert or obtunded
Airway – patent?
Breathing – RR, SaO2
Circulation: HR, BP, mucous membranes, 2x large bore IV cannulas with resuscitation fluids if needed,
review fluid balance chart
Disability - GCS
Review obs chart, medications (anticoagulants?), operation report, patient notes
Hx of fall
• Mechanism of fall? Injuries?
• LOC?
• Post-ictal period?
• Dizziess
• anticoagulation
• Events leading up to fall – dizzy? Lightheaded? Palpitations?
• Amount of bloodloss
• Past hx of bleeding disorders, seizures, IHD
• Review: chart, operative report, perioperative progress notes, adequacy of fluid replacement, omission
of important meds?
Examination:
• Repeat obs
• Examine site of bleeding
• Surgical site examination
• Neurological exam – focal defecits? Global defecits?
• Cardiovascular exam – including check for orthostatic hypotension
DDx – Hypovolaemia, aortic stenosis, seizures, arrhythmias, vasovagal/micturition syncope, MI, CVA, PE

Ix:
B – FBC, UEC, Coags, Group and hold
O –
X – X-ray for fractures
E – ECG for arrhythmias, MI
S – CT head looking for site of bleeding/ ?CVA if suspected/ head trauma
Mx:
Attempt to bring about haemostasis – upright position, packing if needed
Provide fluid resuscitation and treat any injuries as required
Contact ENT surgeon who performed operation for advice- urgent ENT review
Complications of functional endoscopic sinus surgery:
• CSF leaks, orbital injury, intracranial haemorrhage, meningitis
Important causes of post-operative collapse or rapid deterioration:
Cardiovascular MI, sudden arrhythmia, fluid overload, PE, stroke, haemorrhage
Respiratory Failed reversal of anaesthesia, drug induced respiratory depression
Surgical/infective Hypovolaemic shock- blood loss, decompensation, systemic sepsis, SSI
Metabolic Electrolyte disturbance, BGL abnormality, adrenal insufficiency –> hypotension
drugs Drug reaction e.g. anaphylaxis

How well did you know this?
1
Not at all
2
3
4
5
Perfectly
64
Q

A 65 yoa lady has presented with a lump in her right cheek, centred near the angle of the mandible. What is the differential diagnosis?

A

DDx - VITAMIN CDEF

Neoplasm (most likely):
Parotid tumours 
Causes:
Benign (70%) - Pleomorphic adenoma 
Malignant – Mucoepidermoid carcinoma, SCC, acinus cell carcinoma
Hx: painless, firm, slow-growing mass 
Red flags: Red flags on Hx: Facial n. weakness, rapidly increasing size, ulceration of mucosa/skin 
Others: 
Lipoma (tumour of being adipose tissue) 
Non-Hodgkin Lymphoma 
Odontogenic 
Muscles of mastication 
Infective/inflammatory:
Parotid infection 
Bacterial (S.aureus, streptococci)  
Hx: sudden onset pain, swelling, purulence from duct, salivary stones 
Viral (Mumps, HIV, coxsackievirus) 
Hx: usually bilateral swelling, tenderness
Odontogenic infection and abscess
Hx: acute onset pain, swelling, trismus 
Lymphadenitis 

Obstructive:
Sialolithiasis (salivary stone)
Hx: chronic/recurrent pain and swelling, occurs with meals, recurrent infections
O/E: swelling, milking gland produces saliva, intraoral palpable stone

Autoimmune:
Sjogren’s syndrome
Hx: xerostomia, keratoconjunctivitis sicca, abnormal taste, intermittent swelling

Idiopathic:
Sialadenosis (non-specific bilateral enlargement of the salivary glands)

How well did you know this?
1
Not at all
2
3
4
5
Perfectly
65
Q

How would you go about investigating a 60 year old lady who presented to you having found a lump, about 2cm across, in her left breast?

A

Approach: “Triple Test”

Positive if any component is indeterminant, suspicious or malignant
Sensitivity (true positive) 99.6%
Specificity 62% i.e. false positive 38%

Hx and O/E
Imaging (mammography and/or ultrasound +/- MRI)
Non-excisional biopsy (FNA +/- core biopsy)

Hx:
HxPC
Site (1/2+, localised or generalised) 
Onset/duration (rapid – cyst, slow – carcinoma) 
Growth or change (fluctuate with menstrual cycle – fibroadenoma, fibroadenosis)
Previous lump
Associated symptoms
Pain/tenders (non-malignant) 
Nipple retraction or inversion 
Nipple discharge 
Skin changes (redness, swelling or puckering of the skin) 
Systemic features:
Fever, weight loss, anorexia 

PMHx/SHx
Previous breast problems (any previous Ix or results)
Obstetric history (RF for malignancy – late age at first pregnancy, low parity)
Menstrual history (RF for malignancy – early menarche, late menopause)
Obesity
Medications: the pill, HRT
FHx
Family history of breast/ovarian cancer – age of onset, bilaterality

O/E:
General inspection - patient sitting upright on side of bed
Asymmetry
Scars (small – lumpectomy, large diagonal – mastectomy)
Masses – size, location
Skin changes – erythema, puckering, peau d’orange (inflammatory breast cancer)
Nipple changes (retraction, discharge (green, blood-stained), scale)
Closer inspection - hands on hips (tenses pectoralis major), hand on head and lean forward
Palpation – hand behind head of breast you’re examining, including axillary tail and nipple
Location (quadrant of breast or clock face)
Size and shape
Consistency (smooth and round -benign, firm/hard – probable malignancy)
Edges (well-defined/irregular)
Mobility (mobile – benign, fixed – probably malignancy)
Fluctuance (fluid-filled – cyst)
Temperature
Overlying skin changes (erythema, ulceration, punctum)
General lymph node examination (axillary lymphadenopathy – probably malignancy)

How well did you know this?
1
Not at all
2
3
4
5
Perfectly
66
Q

What are the risk factors for breast cancer?

A
Non-modifiable:
Increasing age (peak incidence 70-80s)
Female 
Family or personal hx
BRAC-1 or BRAC-2 mutation (5-10%)
Menses <12y/o 
Late menopause >55y/o
Previous breast biopsies showing non-malignant abnormalities 
Modifiable:
Obesity (fat tissue produces oestrogen) 
Alcohol (limits liver’s ability to control the levels of oestrogen and can damage DNA cells) 
Smoking 
First pregnancy >30y/o
OCP/HRT
Radiation exposure  

Protective factors:
Regular physical activity
First pregnancy <20y/o
Breast feeding >12mths

How well did you know this?
1
Not at all
2
3
4
5
Perfectly
67
Q

Describe the options for adjuvant treatment for breast cancer.

A

Adjuvant therapy is given following primary surgical management to reduce the risk of local recurrence or prevent metastatic spread.

Radiotherapy
Indications:
Post breast-conserving therapy (lumpectomy)
Post mastectomy if high recurrence risk or nodal involvement
Delivered daily (except weekends) for 3-6wks via external beam to chest wall +/- local nodes
SE: Fatigue, skin reactions (sunburn), lymphedema, loss of appetite, predisposed to other cancers

Chemotherapy
Indications:
Lymph node involvement (regardless of size or menopausal status)
Premenopausal with invasive cancers (generally more aggressive)
HR -ve and HER2 +ve
Contraindications:
Less effective if node negative or post-menopausal
Non-invasive cancer
Combination: Anthracycline (doxorubicin) + Taxane (paclitaxel)
SE: nausea, vomiting, alopecia, neutropenia, fatigue, mouth ulcers, infertility

Hormone therapy
Indications:
ER +ve
Types:
Ovarian ablation (premenopausal)
GnRH agonists (supress ovarian function) or oophorectomy
Tamoxifen (premenopausal and low-risk postmenopausal)
Selective oestrogen receptor modulator that blocks peripheral action of oestrogen by binding to the receptor

SE: menopausal symptoms (vaginal dryness, hot flushes), clotting risk, endometrial cancer
Aromatase inhibitors (high-risk postmenopausal)
Prevents oestrogen synthesis in peripheral adipose tissues by blocking conversion of androgens to oestrogen by aromatase enzyme.
SE: osteoporosis, fractures, worsening menopausal symptoms
Delivered as daily oral medication for 5+ years post other treatments

Biological therapy
Indications: HER2 +ve
Trastuzumab (Herceptin)
Monoclonal antibody that targets HER2 receptors to downregulate action and initiate immune mediated destruction.
SE: flu-like, fever, anorexia, muscle/joint pain, diarrhoea
Delivered post-surgery and chemotherapy (must have or won’t be covered by PBS) at 3wk intervals for 1yr

How well did you know this?
1
Not at all
2
3
4
5
Perfectly
68
Q

What are the surgical options for a small confirmed cancer which is sited laterally in the breast, is not fixed to the skin, deep tissue or nipple? What are the advantages and disadvantages of the options?

A
  1. Breast conservation surgery
    Background
    Removal of tumour with margin of surrounding breast tissue + radiotherapy
    Lumpectomy (1cm margins), wide local excisions (2-3cm margins), quadrantectomy
Indications
Patient’s choice 
Single lesion, Stage I/II, <4cm 
Nil skin, nipple/areola involvement 
No node involvement 
CI: pregnancy, BRCA, SLE, lupus 
Advantages
Comparable survival 
Better cosmetic and psychological result
Less invasive 
Shorter recovery 
Disadvantages
Follow-up radiotherapy 
Higher rate of recurrence 
Anxiety/fear of recurrence 
May require additions surgeries (poor margin or recurrence)
2. Mastectomy
Background
Simple/total: breast + level I nodes  
Modified: breast + lvl I-III nodes + pec minor
Radical: modified + pec major   
Indications
Not a candidate for BCS
Multicentric disease
Positive margins after resection 
Radiation CI (pregnancy, recurrence of cancer) 
Advantages
Comparable survival 
Better cosmetic result if smaller breast 
Reconstruction potential  
Lower risk of recurrence 
No follow-up radiation if low risk
Disadvantages
Comparable survival 
Better cosmetic result if smaller breast 
Reconstruction potential  
Lower risk of recurrence 
No follow-up radiation if low risk
How well did you know this?
1
Not at all
2
3
4
5
Perfectly
69
Q

What are the important differential diagnoses for a breast lump?

A

Malignancy: 10%

Non-invasive/pre-cancerous:
Ductal carcinoma in situ (DCIS) 80%
Lobular carcinoma in situ (LCIS) 12%

Invasive:
Infiltrating ductal carcinoma (IDC) 75%
Infiltrating lobular carcinoma (ILC) 10%
Other: Medullary (5%), mucinous (colloid), inflammatory, tubular, Paget’s disease, papillary 
Metastases from another primary site 

Benign:
Cyst (blockage of duct resulting in fluid-filled mass)
Hx: premenopausal/perimenopausal
O/E: compressible, ballotable solitary mass

Fibroadenoma
Hx: asymptomatic, incidental finding, <40y/o
O/E: smooth, rubbery, mobile mass

Fibrocystic change or fibroadenosis
Hx: breast pain, increase with menstrual cycles
O/E: not well-defined mass, nodular breast tissue

Fat necrosis
Hx: obese, large breast, blunt trauma, injection of substance, breast surgery, radiation therapy
O/E: skin ecchymosis, hard irregular lump

Breast abscess
Hx: lactating, mastitis, fever, malaise, painful mass
O/E: fluctuant, tender, palpable mass

Galactocele (milk retention cyst)
Hx: breastfeeding

Phylloides tumour
Hx: 40-60, rapid enlargement breast
O/E: well-delineated, large breast mass

Intraductal papilloma (local epithelial proliferation in mammary ducts)
Hx: blood stained discharge 
Duct ectasis 
Hx: menopause, green nipple discharge
How well did you know this?
1
Not at all
2
3
4
5
Perfectly
70
Q

What is the influence of a patients age on the choice of imaging for breast cancer?

A

Indications:
>30 Mammography
<30 U/S or MRI if high risk (family hx, BRCA in first degree relatives, hx chest radiation)

Reasoning:
Denser breast tissue in younger women limits the sensitivity of mammography.
In patient <35 false-negative rate for mammography is >50%
MRI not affected by age or density of breast tissue
More sensitive, but not very specific which can result in unnecessary biopsies

How well did you know this?
1
Not at all
2
3
4
5
Perfectly
71
Q

A 26 yoa woman presents to the ED with a painful, hot, swollen left breast. She is 5 weeks post partum with her first child and has been breast feeding successfully. She was prescribed antibiotics for the problem 5 days ago by her GP but has been getting worse. She has no systemic previous medical history, takes no regular medication. On examination, she is febrile, and has a fluctant swelling in the left lower quadrant of her left breast with overlying erythema.
What is the diagnosis?
What is the treatment?

A

Dx: Lactational breast abscess

Mx:
Needle aspiration (+/- U/S guidance) or surgical incision/drainage.
Empirical abx (flucloxacillin) – modify based on culture of swab.
Pain relief – simple analgesia, ice pack.
Continue breast feeding/expressing milk – important for resolution of infection and relief of breast fullness.
? referral to lactation nurse consultant.

For reference: Lactational breast abscess

Definition: collection of infected fluid within the breast

Background:
20% breastfeeding women develop lactational mastitis, with 3% developing an abscess
Peak within first 6wks post-partum
Causative organism: S.aureus
Risk factors: hx mastitis, primiparity, maternal age >30, GA >41wks

Pathophysiology:
Milk stasis or milk overproduction coupled with bacterial invasion of the breast tissue through a traumatised nipple results in mastitis.
As the breast tissue becomes inflamed it further blocks milk ducts and allows for progression of the infection. Left untreated, it can cause tissue destruction resulting in an abscess.

Presentation
Hx: worsening mastitis unresponsive to abx
Breast pain, fever, flu-like (malaise, lethargy, myalgia, sweating, headache, nausea/vomiting)
O/E: Inflammation, fluctuant tender mass

How well did you know this?
1
Not at all
2
3
4
5
Perfectly
72
Q

How would you go about investigating a 16 year old girl who presented to you having found a lump, about 2cm across, in her left breast?

A

Dx: Fibroadenoma (most common lesion between 15-35y/o)

Reassure patient that most breast lumps benign, but must complete “Triple Test”

Triple testing is a combination of clinical examination, imaging and nonsurgical biopsy.

History:
HxPC
Site, onset, duration
Growth or change (fluctuate with menstrual cycle – fibroadenoma, fibroadenosis)
Previous lump
Associated symptoms (discharge, pain, tenderness, skin changes)
Systemic features (fever, weight loss, anorexia)
PMHx/SHx
Previous breast problems (any previous Ix or results)
Gynaecological, menstrural and obstetric hx
Medications – OCP
Hx cancer
FHx
Family history of breast/ovarian cancer – age of onset, bilaterality, BRAC genes

O/E:
Inspection: Asymmetry, scars, masses, skin changes, nipple changes/discharge
Palpation: Location, size, shape, consistency, mobility, temperature, tenderness/painless, skin changes
Lymph nodes

Ix:
U/S (well-defined, solid mass – fibroadenoma)
FNA or CNB (especially if red flags or suspicious imaging)

How well did you know this?
1
Not at all
2
3
4
5
Perfectly
73
Q

A 30 yoa woman presented with a painful lump in her left breast 5 days after childbirth. This is a photograph of the breast. What do you see?

A

Dx: Lactational mastitis +/- abscess

O/E:
Describe breast + quadrant
Nipple cracking (+/- retraction/inversion)
Localised area of inflammation (erythema, swelling)
Purulent discharge
Abscess punctum
Lactational localised to peripheral wedge-shaped areas of the breast (non-lactational central)

How well did you know this?
1
Not at all
2
3
4
5
Perfectly
74
Q

What is the most likely organism that might be causing an acute, lactation associated, breast abscess?

A

Staphylococcus Aureus

How well did you know this?
1
Not at all
2
3
4
5
Perfectly
75
Q

What is the management of an acute lactation associated breast abscess?

A

Needle aspiration (+/- U/S guidance) or surgical incision/drainage
Empirical abx (flucloxacillin) – modify based on culture of swab
Pain relief – simple analgesia, ice pack
Continue breast feeding/expressing milk – important for resolution of infection and relief of breast fullness
? referral to lactation nurse consultant

How well did you know this?
1
Not at all
2
3
4
5
Perfectly
76
Q

A 30-year-old woman presented with a painful lump in her left breast 5 days after childbirth. This is a photograph of the breast. What is the likely diagnosis and what is the management?

A

Dx: Lactational mastitis +/- abscess

Mx mastitis:
Flucloxacillin 500mg 6hrly for 5days
Effective milk removal 
Pain relief – analgesia, warm/cold compress
Increased fluid intake 

Mx abscess:
Needle aspiration (+/- U/S guidance) or surgical incision/drainage
Empirical abx (flucloxacillin) – modify based on culture of swab
Pain relief – simple analgesia, ice pack
Continue breast feeding/expressing milk – important for resolution of infection and relief of breast fullness
? referral to lactation nurse consultant

DDx:
Breast engorgement (occurs day 3-5 post-partum, requires frequent breast emptying)
Inflammatory breast cancer

How well did you know this?
1
Not at all
2
3
4
5
Perfectly
77
Q

A 50 year old woman is diagnosed with breast cancer. Her surgeon recommends a “lumpectomy” and sentinel node biopsy. The pathology report shows a 20 mm grade 3 ER positive, PR positive, Her-2 negative cancer, without lymphovascular invasion, and a negative sentinel node biopsy.

a) Describe three “adjuvant” treatments that would be potentially beneficial to her.
b) referring to the aims of each treatment and distinguishing between the aims of adjuvant systemic therapy and adjuvant local therapy.

A

For this patient I would recommend radiotherapy, hormone therapy and chemotherapy. I would not recommend biological therapy as she is HER2-ve and this would not be effective.

Radiotherapy
Indicated after breast conserving surgery
Aims to achieve local disease control by eradicating any residual disease and preventing its recurrence.
Hormone therapy (e.g. tamoxifen or aromatase inhibitors)
Indicated in hormone receptor positive cancers
Aims to reduce the level of circulating oestrogen and block hormonal stimulation of remanet cancer cells.
Chemotherapy
Indicated in HER2-ve grade 3 cancers >10mm
Aims to eradicate microscopic cancer cells that could recur as metastases
Most common regimen is Anthracycline + Taxane

Both local and systemic adjuvant therapies aim to improve survival by preventing disease recurrence. However, they can be distinguished by their mechanism of action. Local therapy acts directly on the breast to prevent recurrence of the primary tumour, systemic therapy acts through the whole body targeting metastatic spread or occult disease.

How well did you know this?
1
Not at all
2
3
4
5
Perfectly
78
Q

A 30-year-old man presents with a 3.2mm melanoma on his right arm. A wide excision and sentinel node biopsy is performed. One node is removed from the right axilla as part of the sentinel node procedure. It contains a 0.3mm deposit of melanoma. What are the biological arguments for and against proceeding to an axillary clearance?

A

For:
Staging and prognostic purposes
Determining appropriate adjuvant therapy
Reduce risk of further spread
Improves quality of life if bulky adenopathy in neck
Stage T3 or T4 (>2.0mm)
If macro metastases >2.0mm

Against:
No overall survival benefit (in comparison to observation alone)
May not prevent recurrence elsewhere
Approx. 80% of patients with positive sentinel node have no additional disease i.e. unnecessary surgery
Surgical risks – lymphedema, wound infections, nerve damage
Stage T1 or T2 (<2.0mm)
If micro metastases
Older patient, unstable patient, high peri-operative risk, life-shortening comorbidity

Definition: removal of all lymph nodes and possible tumour containing tissue from the armpit region.

Levels:
Lateral to pec minor
Up to medial border pec minor
Beyond medial border pec minor

How well did you know this?
1
Not at all
2
3
4
5
Perfectly
79
Q

A 53 year old man presents with a pearly nodule on his back that has, according to the patient “come up fairly quickly in the last 6 weeks”. It is bleeding when he rubs it with a towel. Q1. How should you mange this problem?

A

Dx: Nodular basal cell carcinoma

History:
HxPC:
Site, onset, duration 
Growth or change 
Association symptoms (pain, bleeding – after trauma) 
Risk factors: 
Previous skin malignancies 
Sun exposure (occupational/recreational) 
Exposure to radiation 

O/E
Inspection:
ABCDE approach: (more for pigmented lesions)
Asymmetry, irregular borders, colour, diameter >6mm, evolution
Characteristic features:
Pearly/waxy, central depression, central ulceration, raised border, telangiectasias
Closer inspection via dermatoscope
Full skin check

Mx:
Surgical:
Excision biopsy (gold-standard) with >4mm margins – cure rate approx. 90%
Curettage and cautery – approx. 90%
Non-surgical:
Radiation therapy – cure rate approx. 90%
Cryotherapy – approx. 85-90%
Photodynamic therapy – approx. 80%
Topical immunotherapy (e.g. imiquimod) – approx. 80%

For reference: Nodular BCC

Definition: Locally invasive skin cancer

Risk factors:
Sun exposure
Fair skin
Previous skin cancer

Clinical features:
Slow growing plaque/nodule 
Shiny/pearly nodule with smooth surface 
Variable size mm-cms 
Spontaneous bleeding or ulceration 
Rarely - rapid growing, invasive and/or metastatic to local lymph nodes
How well did you know this?
1
Not at all
2
3
4
5
Perfectly
80
Q

A 55 year old woman presents with a breast lump. Q1. What clinical and mammographic features together would allow you to be sure that it is benign?

A

Clinical features (alone not sufficient to differentiate):

Benign
Multiple lesions 
“Rubbery”
Mobile
Well circumscribed border
Bilateral, milky discharge
Fluctuating 
Malignant
Single lesion 
Hard
Immobile
Irregular borders
Unilateral, bloody, clear or coloured discharge
Skin changes
Hx of cancer or systemic symptoms 

Mammographic features:

Benign
Round/oval shape
Well circumscribed border
Low density
Large, coarse (‘popcorn-like’) or smooth, round calcifications
Malignant
Stellar shape
Irregular (spiculated) border 
High density
Clustered or linear branching micro-calcifications
How well did you know this?
1
Not at all
2
3
4
5
Perfectly
81
Q

A 45-year-old woman presents with a new breast lump that she has only just noticed. What single investigation might allow you to reassure her, and what finding on that investigation would be most reassuring?

A

Gold-standard Ix is the triple test, as no test alone has greater sensitivity or specificity.

In terms of a single most reassuring investigation, it would be a core biopsy for cytology and histopathology. Although, this usually requires imaging first.

Reassuring findings:
Histology of tissue specimen being void of malignant change (e.g. differentiated or irregular cells, abnormal proliferation)

How well did you know this?
1
Not at all
2
3
4
5
Perfectly
82
Q

A 36-year-old woman has a mastectomy and axillary clearance for a large “triple negative” breast cancer with multiple nodes involved. She is offered adjuvant chemotherapy which should provide a relative risk reduction of about 25%. Without treatment, this woman’s chances of 5-year survival are 20%. What are her chances of survival with the adjuvant chemotherapy? Explain how you arrived at this answer.

A

Relative risk reduction: relative/proportional risk reduction of an event happening in 2 groups with different interventions

Current risk of death in 5yrs is 80%, and chemotherapy offers a 25% reduction relative to that risk

i.e. new risk of death is 0.25*0.8 = 0.6 = 60%

So, if the new risk of death is 60% with chemotherapy, the new chance of 5yr survival with chemotherapy is 40%

How well did you know this?
1
Not at all
2
3
4
5
Perfectly
83
Q

A 50-year-old woman has a wide excision and sentinel node biopsy for a breast cancer. The pathology report shows a 30mm grade 3 ER positive, PR positive, Her-2 negative cancer, without lymphovascular invasion, and a negative sentinel node biopsy. Describe the adjuvant treatment options available to her. Refer to the different aims of adjuvant systemic therapy and adjuvant local therapy.

A

For this patient I would recommend radiotherapy, hormone therapy and chemotherapy. I would not recommend biological therapy as she is HER2-ve and this would not be effective.

Radiotherapy (via external beam)
Indicated after breast conserving surgery
Aims to achieve local disease control by eradicating any residual disease and preventing its recurrence.
Hormone therapy (e.g. tamoxifen or aromatase inhibitors)
Indicated in hormone receptor positive cancers
Aims to reduce the level of circulating oestrogen and block hormonal stimulation of remanet cancer cells.
Chemotherapy
Indicated in HER2-ve grade 3 cancers >10mm
Aims to eradicate microscopic cancer cells that could recur as metastases
Most common regimen is Anthracycline + Taxane

Both local and systemic adjuvant therapies aim to improve survival by preventing disease recurrence. However, they can be distinguished by their mechanism of action. Local therapy acts directly on the breast to prevent recurrence of the primary tumour, systemic therapy acts through the whole body targeting metastatic spread or occult disease.

How well did you know this?
1
Not at all
2
3
4
5
Perfectly
84
Q

How are anal fissures treated?

A

Conservative:
General measures:
Avoid straining
Mx constipation – Fibre, stool softeners e.g. Movicol, fluids
Sx analgesia
Warm anal bathing – stimulates somatoanal reflex
Topical ointment (1st line)
GTN – smooth muscle (sphincter) relaxation and vasodilation
CCB (diltiazem) – blocks smooth muscle contraction
Botulinum toxin injection (2nd line)
Sphincter relaxation (paralysis) for 8-12wks

Surgical:
Lateral internal sphincterotomy
Indicated post 6-8wks of conservative therapy
Method: Divide 1cm of the lower portion of the internal sphincter
Risks: incontinence of flatus or stools, infection

How well did you know this?
1
Not at all
2
3
4
5
Perfectly
85
Q

What is an anal fissure? How does it develop? What causes it to persist?

A

Definition: Longitudinal tear in the mucosa and skin of the anal canal distal to the dentate line. Approx. 90% occur in the posterior midline, whereas secondary causes more associated with lateral tears.

Aetiology:
Acute 
Primary: 
Passage of hard stools
Explosive diarrhoea
Vaginal delivery
Foreign body penetration 
Secondary: 
IBD
Malignancy
Infection (syphilis, HIV) 
Chronic
Approx. 40% develop into chronic form due to: 
Repeated tearing
“Fear of defecation” which aggravates constipation and straining 
Impaired healing
If extends into internal anal sphincter (involuntary smooth muscle) it can lead to spasm and reduced blood flow with ischemia. 
Exposure to faeces also slows healing 
Presentation: 
Pain post defecation
Tearing, ripping, burning character
Minutes to hours 
Hx constipation
Irritation or itchiness around skin 
Haematochezia 
Fissure or small skin tag (sentinel pile) at 6 or 12 O’clock position 
PR exam extremely painful
How well did you know this?
1
Not at all
2
3
4
5
Perfectly
86
Q

How would you go about investigating a previously well 63-year-old man who presented to you with a short history of rectal bleeding?

A
DDx: 
Colorectal Ca
Benign polyps
Haemorrhoids, fissures 
IBD
Diverticular disease
Infectious (gastroenteritis) 
Hx: 
PC
Bleeding
Onset, duration, previous bleeds, mucous, colour
Bright red – anorectal 
Dark red, clots – rectosigmoid 
Dark red mixed in stool – ascending/transverse 
Melena – Upper GI 
Associated Sx 
Pain, weight loss, fever, night sweats, changes in bowel habits
PMHx
Any of the DDx
Abdominal surgery Hx
Coagulopathies (incl. medications)
RF for Colorectal Ca
Polyposis syndromes (FAP, HNPCC, juvenile polyposis)
FHx
PMHx polyps or CRCa
Chronic UC or Crohn’s 
Diet poor in fruit and vegetables
Smoking/alcohol 
Long term NSAIDs

O/E
Vitals
Abdo (conjunctival pallor, palmar pallor)
PR (haemorrhoids, anal fissure, abscess, distal carcinoma or polyp)

All patients >45 with new onset rectal bleeding must be investigated due to high risk of CR Ca

Ix:
Pathology
FBC, UECs, coagulation studies, LFTs (metastatic disease)
Bedside rigid sigmoidoscopy
Visualised up to splenic flexure (75% of CR Ca)
Note that most PR bleeding with CR Ca will be distal to descending colon
Colonoscopy +/- biopsy
Other:
? Upper GI endoscopy
If CR CA suspected - CT colonography, abdominal CT

How well did you know this?
1
Not at all
2
3
4
5
Perfectly
87
Q

How would you go about investigating a previously well 63-year-old man who presented to you with a short history of anaemia?

A

DDx:
Microcytic
Decreased intake (lack of red meat ingestion)
Blood loss (CR Ca, haemorrhoids, IBD, menorrhagia, PUD)
Decreased absorption (surgery, Crohn’s disease)
Increased demand (puberty, pregnancy)

Normocytic
Increased reticulocytes i.e. Hyperproliferative
Haemorrhage
Haemolytic (thalassemia, sickle cell, autoimmune, infection, drugs)
Decreased reticulocytes i.e. hypoproliferative
Aplastic, leukaemia

Macrocytic 
Megaloblastic (hyper segmented neutrophils) 
B12 deficiency 
Folate deficiency 
Non-megalobalastic 
Alcohol 
Liver disease
Hx: 
PC
Source of bleeding? (GIT, bladder, menstural) 
Previous Hx
Diet (iron, folate, B12) 
PMhx
IBD, coeliac, GORD, malignancy
Coagulopathy
Surgical Hx (gastrectomy, bariatric)
Medications 
NSAIDs, steroids
Antiplatelet, anticoagulant 
Smoking, alcohol, drugs

O/E:
Vitals
Abdo (conjunctival pallor, SOB, mass, hepatomegaly etc.)
PR

Ix: 
Pathology 
FBC 
RBC <130 in M OR <120 W
Microcytic 🡪 iron studies
Normocytic 🡪 reticulocytes
Macrocytic 🡪 blood film + serum B12/folate 
Blood film/peripheral smear 
Cell morphology e.g. sickle cell
Iron, B12, Folate
UECs, LFTs
Other: 
If occult blood – FOBT, colonoscopy 
If haemoptysis/melena – upper GI endoscopy 
If cause unclear – bone marrow biopsy (hypo proliferative normocytic anaemia)
How well did you know this?
1
Not at all
2
3
4
5
Perfectly
88
Q

How would you go about investigating a previously well 63-year-old man who presented to you with a short history of a change in bowel habit to more frequent evacuation of loose stools with mucous?

A
DDx: 
IBS, IBD (especially UC)
CR Ca
Haemorrhoid, fissure, villous adenoma
Bacterial infection 
Obstruction, colostomy 
Hx: 
PC
Bowel habits – duration, frequency, Bristol stool chart 
Associated Sx – bleeding, abdo pain, n/v/a, weight loss, fever
Sick contacts, recent travel 
Previous Hx
PMHx
Any DDx
RF CR Ca
Polyposis syndromes (FAP, HNPCC, juvenile polyposis)
FHx
PMHx polyps or CRCa
Chronic UC or Crohn’s 
Diet poor in fruit and vegetables
Smoking/alcohol 
Long term NSAIDs
Previous Ix
Medications 
Abx – Clostridium Difficile 

O/E:
Vitals
Abdo + PR

Ix: 
Pathology 
FBC
UECs
Electrolyte disturbances from diarrhoea) – hypokalaemia, hyponatremia 
CRP, ESR
Nutritional deficiencies (B12, vitamin D) 
Stool culture + MCS
FOBT 
Consider: 
Rigid/flexible sigmoidoscopy 
Colonoscopy +/- biopsy 
Upper GI endoscopy +/- biopsy
Barium studies (IBD) 
Contrast CT (localise extent, assess mural and extra mural involvement)
How well did you know this?
1
Not at all
2
3
4
5
Perfectly
89
Q

What are the major risk factors for colorectal cancer?

A
Epidemiology: 
CR Ca 2nd commonest tumour, most common GI malignancy 
Approx. 1/12 Australians
M>F,
Peak: 55-75y/o
RF:
Modifiable 
Diet (high red meat, low fibre (fruit/vegetables)) 
Obesity 
Physical inactivity 
Smoking 
Heavy ETOH
Long term NSAIDs
Non-modifiable 
Age >50
FHx 1#
PMHx polyps or CRCa
IBD
Hereditary polyposis syndromes (FAP, HNPCC – lynch syndrome), juvenile polyposis)
How well did you know this?
1
Not at all
2
3
4
5
Perfectly
90
Q

What method is used for population screening for colorectal cancer in Australia?

A
Screening: 
Low Risk 
Asymptomatic, Nil FHx
Method: 
FOBT
Two stool samples on consecutive days to assess for blood 
50-74y/o every 2 years 
70% sensitivity (i.e. 30% false negatives) 
Moderate Risk 
1# relative OR 2x2# relative
Method: 
FOBT – 2yearly 
Colonoscopy – 5yearly or 10yrs before FHx diagnosis
High Risk 
3x1# relative, extensive premature adenomas 
Method: 
Refer to bowel cancer specialist 
? Flexible sigmoidoscopy or colonoscopy 1-2yearly >25y/o or 5yrs before affected family member 
Genetic screening 
Symptomatic 
Colonoscopy not FOBT
How well did you know this?
1
Not at all
2
3
4
5
Perfectly
91
Q

What is neoadjuvant therapy for rectal cancer?

A
Neoadjuvant:
Definition:
Treatment given prior to definitive surgery to shrink tumour making it more easily resectable
Initiations:
T3/T4
T1/T2 with nodal involvement 
Distal rectal tumour where APER is required 
Tumour that invades mesorectal fascia 
Benefits:
Higher sphincter preservation 
Improved resectability
Tumour regression
Therapy:
Conventional fractioned radiotherapy +/- chemotherapy (fluropyrimidine) 
Long course approx. 7-10wks 
All pts who receive neoadjuvant also require 4m of adjuvant chemotherapy 

Definitive:
Surgical resection +/- lymphadenectomy

Site of cancer
Type of resection
Anastomosis
Caecal, ascending or proximal transverse colon
Right hemicolectomy
Ileo-colic
Distal transverse, descending colon
Left hemicolectomy
Colo-colon
Sigmoid colon
High anterior resection
Colo-rectal
Upper rectum
Anterior resection (TME)
Colo-rectal
Low rectum
Anterior resection (Low TME)
Colo-rectal
(+/- Defunctioning stoma)
Anal verge
Abdomino-perineal excision of rectum
None
Palliative
For unresectable metastases or tumours 
Method:
Chemotherapy 
Obstructing tumours may be endoluminally stented with self-expanding metal stents or transanally ablated if rectal
How well did you know this?
1
Not at all
2
3
4
5
Perfectly
92
Q

Where are the common sites that metastatic disease from a colorectal primary are found?

A

Metastatic:
Background
Approx. 15-20% have metastasis at time of diagnosis
Location
Lymphatic spread
Lower rectum – internal iliac
Middle/upper rectum – pararectal, inferior mesenteric
Descending/sigmoid colon – inferior mesenteric
Transverse/ascending colon – superior mesenteric
Haematogenous spread
70% Liver (via portal venous system)
35% Lungs (via venous spread)
21% Peritoneum
Other: Bone, ovaries, kidneys, brain
Distal extension
Outer bowel wall direct contact – liver, greater curvature of stomach, duodenum, small bowel, pancreas, spleen, bladder
Rectal cancer – bladder, prostate, vaginal wall, sacrum

Staging Ix:
Assessment of metastases (liver, lung or para-aortic)
Thoracoabdominopelvic CT
CT PEP scan to evaluate equivocal lesions
Assessment of local extent
Colonic – CT adequate
Rectal – Pelvic MRI and TRUS
Assessment of synchronous tumours
Colonoscopy or barium enema to confirm no secondary tumour
Tumour marker
CEA not useful for diagnosis or staging but can indicate relapse if high at time of diagnosis and falls to normal after resection

How well did you know this?
1
Not at all
2
3
4
5
Perfectly
93
Q

What are the treatment options for liver metastases from a colorectal primary?

A
Pathological staging:
Dukes (approx. %5yr survival)
A – confined to bowel wall only (90%)
B – through bowel wall (70%)
C – any with +ve lymph nodes (50%)
D – any with metastases (<10%)
TNM
T1-4 – stages of invasion of bowel wall 
N0/1/2. No/up to 4/more than 4 lymph nodes involved
M0/1, Metastases not present/present
Confirming Metastases
Imaging
CT
FDG-PET
Biopsy
FNA
Laparoscopic needle biopsy 

Regional Mx:
Surgical resection (Approx. 50% 5YS)
Eligibility discussed by MDT
Patient factors – fitness for surgery, preference
Cancer factors
No extrahepatic metastasis
No portal vein, hepatic artery or bile duct involvement
Adequate hepatic function post resection (2/8 segments)
Radiofrequency ablation (14-55% 5YS)
Insertion of image guided probe percutaneously into tumour which emits high energy electrical currents effectively burning cancer
If not eligible for surgery
Selective internal radiation therapy (SIRT)
Injection of tiny radioactive beads providing specific radiation to liver
Trans-arterial chemoembolization
Chemotherapy injected directly into liver vessels
Based on principle that hepatic cancers preferentially receive blood supply from hepatic artery whilst normal parenchyma are supplied predominantly by the portal vein

Systemic Mx: 
Chemotherapy 
Neoadjuvant
Transitions 10-15% of unresectable to resectable
Adjuvant
Post-surgery for 6m 
Palliative 
10% 5YS, prolongs survival 5-20months
Decrease symptoms of jaundice, pain, nausea, fatigue
How well did you know this?
1
Not at all
2
3
4
5
Perfectly
94
Q

What is the Adenoma-Carcinoma Sequence?

A

Background:
Process by which mutational activation of oncogenes and inactivation of tumour suppressor genes result in cancer.
In CR Ca, this is the process in which benign polyps undergo dysplastic change to become malignant adenocarcinomas.
Process takes approx. 5-10years

Pathophysiology: 
Pathway of mutation
Chromosomal instability (85%)
Microstallite instability (15%) 
Genes:
Oncogenes: KRAS
Tumour suppressor genes: APC and p53 
Progression
Normal mucosal cells of intestine continuously lost into lumen due to apoptosis and exfoliation, and continually replaced by proliferation at the crypt base. 
Subsequent to mutations it progresses from normal to low-grade dysplasia to high-grade dysplasia to carcinoma

CR Ca:
Predominant type adenocarcinoma
May occur as a polypod, ulcerating, stenosing, or infiltrative tumour mass
Majority (75%) lie on left side of colon and rectum

How well did you know this?
1
Not at all
2
3
4
5
Perfectly
95
Q

What is Crohn’s Disease?

A

Definition: Chronic immune-mediated inflammatory disorder that can affect any part of the GIT. Most commonly small/large bowel, particularly terminal ileum and right colon.

Epidemiology:
>incidence the further from the equator
Peak 15-30y/o and 55-80y/o
Idiopathic

RF:
Smoking

Sx:
RIF Abdo pain + diarrhoea (+/-blood/mucus) + weight loss
Constitutional: malaise, fever, n/v/a
Extra intestinal: conjunctivitis, iritis, gallstone formation

O/E:
Abdo
Inspection: wasting, anaemia, red fat tongue, oral ulcers
Palpation: tenderness, RIF mass (when omentum wraps around inflamed terminal ileum)
PR: nodules, perianal fissures

Ix:
Diagnosis: Colonoscopy (most sensitive) + Biopsy
Consider:
Pathology
FBC, ESR, CRP, Iron/B12
Decreased Hb and albumin (protein-losing enteropathy)
Stool MCS + culture – exclude infection (e.g. C.Difficile)
Antibody tests to distinguish from UC
Imaging
Abdo CT
Assess strictures, disease location, wall thickening, complications
Small bowel contrast study (barium enema or from duodenum via NG tube)
Narrowing of lumen, fibrosis, nodularity, cobblestones, ulceration

Gross Appearance:
Scope
Pink (preserved mucosa) between Yellow/white (ulcers) – “Skip lesions”
Fissures extending from epithelium into muscle wall – “Cobblestone appearance”, abscess, fistula, strictures
Transmural inflammation – thickened bowel wall
Fat wrapping (fingers of mesenteric fat wrap around the bowel due to chronic inflammation)
Histological
Non-caseating granuloma (approx. 50% of biopsies)
Lymphoid aggregates
Transmural inflammation
Submucosal oedema
Fibrosis

Cx:
Fissures
Fibrosis – Strictures, bowel obstruction
Fistulas 
Adhesions 
Perforation – Peritonitis

Fistula’s:
Approx. 35% affected
Commonly involve small bowel
Mx:
Regression with aggressive medical mx
Resection of affected bowel segment + closure of other viscera
Note: high risk of leak due to impaired healing process in CD

Perianal disease: 
Approx. 20% 
Involves fissures, abscesses and fistulas 
Presentation: 
Devastatingly painful 
Bleeding 
Mx:
Fissures: Sitz baths, stool softeners, analgesics
Abscess: incision and drainage under GA
Fistula: 
Surgical
Setons (rubber band threat through fistula to keep it open and allow drainage preventing infection)
Fistulotomy 
Bowel resection 
Abx cover 

Medical Mx:
Anti-inflammatory agents
5-ASA (Sulfasalazine and Mesalazine) – first-line acute and long-term
Corticosteroids (prednisolone, budesonide) – acute exacerbations
Immunomodulators
Azathioprine, 6-mercaptopurine
Infliximab (anti-TNFalpha antibodies)
Dietary modification
Elemental diet to reduce inflammatory factors
Liquid/low fibre diets if at risk of obstruction
Supplemental calories, iron and vitamins

Surgical Mx: 
Approx. 50% end up with surgery
Indications:
Acute complications – fistula, obstruction, abscess, fulminant colitis, toxic megacolon
Failure of medical therapy 
Cancer 
Growth retardation
How well did you know this?
1
Not at all
2
3
4
5
Perfectly
96
Q

What is Diverticular Disease?

A

A diverticulum is an outpouching of a hollow or fluid filled structure.
Diverticular disease is defined as clinically significant and symptomatic diverticulosis due to diverticular
bleeding, diverticulitis, segmental colitis associated with diverticula, or symptomatic uncomplicated
diverticular disease.
Diverticular disease is extremely common, affecting more than 50% of people over age of 70.
Common sites affected:
Potential defects can occur between longitudinal muscle bands (teniae coli) where mucosal vessels
penetrate the wall from the outside
• Sigmoid colon most commonly affected
• Right sided diverticular- rare but often ddx for appendicitis
• Isolate caecal diverticula - common in Western countries- susceptible to perforation
The pathophysiology is likely related to functional hypersegmentation - lack of dietary fibre –> high
intraluminal pressure –>mucosa herniate through muscle layers at weak points of intestinal wall.
Aetiology: dietary fibre deficiency, obesity, red meat consumption, NAIDS, coffee and alcohol, steroids,
decreased physical activity, altered colonic wall structure

How well did you know this?
1
Not at all
2
3
4
5
Perfectly
97
Q

What is Diverticulitis? How Does it Present Clinically?

A

Diverticulitis is the inflammation of a diverticulum. It be acute or chronic, uncomplicated or complicated.

Jessica Redmond 81
Usually presents with continuous LIF pain, altered bowel habits, nausea and vomiting, flatulence and
bloating, and is systemically unwell with fever, tachycardia leucocytosis in symptomatic acute diverticulitis.
It can also present with sign of perforation (peritonism, distended tympanic abdomen, reduced/absent
bowel sounds), and signs of fistula (PV discharge, UTI)
CT is imaging of choice.
Management involves bowel rest, antibiotics (amoxicillin, met, gent, tazocin) and surgical intervention for
complications in particular such as perforation, fistula, abscess.
Uncomplicated
-bowel rest
-abx (cipro + met)
-high fibre diet
-colonoscopy 6 weeks post to exclude colon cancer

Complicated
Empirical Abx - taz or ceftriaxone and met
Mx complications - surgical - colonic resection etc.

How well did you know this?
1
Not at all
2
3
4
5
Perfectly
98
Q

What are the Major Complications That can Result From Diverticulitis?

A

The major complications of diverticulitis are:
• Diverticular abscess
o Pericolic abscess often presents with persistent pain and tenderness, swinging pyrexia, incomplete
obstruction, or even sepsis
o Dx made on CT
o Abx if <4cm, or >4cm drained percutaneously under radiological guidance

• Fistula
o Can occure when inflammed diverticlum lies close to another hollow viscus
o Vesicolic fistula = pneumaturia and severe UTI
o Colo-ileal fistula = diarrhoea
o Colovaginal - PV discharge
o Surgical intervention
• Bowel obstruction
o May occur due to relative luminal narrowing from pericolonic inflammation or compression from
abscess

• Free perforation
o Often from abscess
• Segmental colitis

• Acute PR haemorrhage although not common in diverticulitis as much as diverticulosis

How well did you know this?
1
Not at all
2
3
4
5
Perfectly
99
Q

What are haemorrhoids and how are they classified or described?

A

Haemorrhoids are normal vascular structures in the anal canal, arising from a channel of arteriovenous
connective tissues that drains into the superior and inferior haemorrhoidal veins. They can enlarge and
thus become symptomatic. The cardinal features of haemorrhoidal disease include bleeding, anal pruritus,
prolapse, and pain due to thrombosis.

Constipation and pregnancy are the most common triggers for their development . Straining raises intra-
abdominal pressure which obstructs venous return, causing venous plexuses to engorge, they are dragged

distally by hard stool.
Haemorrhoids are classified as either internal or external, or mixed. External haemorrhoids are distal to the
dentate line in the anal canal and are covered by skin. Internal haemorrhoids are proximal to the dentate
line, and are classified into 4 grades based on extent of prolapse.
Classification of haemorrhoids:
Grade I: never prolapse- protrusion limited within anal canal
Grade II: prolapse during defecation and then return spontaneously
Grade III: piles remain outside anal margin unless replaced digitally.
Grade IV: irreducible and may strangulate

Jessica Redmond 82
Symptoms and signs of haemorrhoids:
• Attacks can last days-weeks, intermittent symptoms
• May bleed from stool trauma during defecation
• May prolapse and thrombose- acute severe pain
• Atrophy- by thrombosis and fibrosis –> skin tags at anal margin
• Strangulated haemorrhoids

How well did you know this?
1
Not at all
2
3
4
5
Perfectly
100
Q

How are Haemorrohids Treated?

A

Treatment is generally conservative, medical or surgical.
Conservative: (all)
• High fibre diet to reduce constipation
• Squatting or feet up to reduce straining
• Avoid straining and excessive time on toilet
• Moist gentle wiping
• Adequate fluid intake
Medical:
Creams, suppositories and topical preparations
• Grade 1- hydrocortisone cream short term for pruritus and pain
• Anaesthetics - benzocaine rectal ointment
• Stool softeners to decrease straining
• antispasmodics
Surgical
• Sclerotherapy - grade 2+
o Indication: First degree haemorrhoids that don’t regress with dietary, and secondary
o Outpatient
o Injection of sclerosing agent with proctoscope
o Injected into submucosal tissue (NOT the haemorrhoid itself) fibrotic action rapidly obliterating the
haemorrhoidal vessels.
o Usually repeated on 2-3 occasional 4-6 weeks apart
o not for people with nut allergy
• Banding - grade 2+
o Cone around base of haemorrhoid to constrict the vessels
o Uses anoscope
o Ischaemic necrosis occurs distal
o l to bands and they fall off
• Haemorrhoidectomy (grade 4+, external)
o Indications 3rd degree haemorrhoids
o Mass excised with overlying skin and mucosa. Secondary intention healing
o Stapled - sometimes for large

How well did you know this?
1
Not at all
2
3
4
5
Perfectly
101
Q

What are the Indications for Surgery in a Patient with Ulcerative Colitis?

A

• Colectomy indications:
o Patients with persistent symptoms
o Chronic disabling symptoms of intractable diarrhoea, urgency, recurring anaemia, failure to maintain weight and nutrition
o Children with failure to thrive
o Patients with increased cancer risk
o Patients with longstanding colitis
o Patients with dysplastic or adenomatous polyps
Most patients undergo a restorative proctocolectomy with ileal pouch anal anastomosis

How well did you know this?
1
Not at all
2
3
4
5
Perfectly
102
Q

What are the Indications for Elective Surgery in a Patient With Crohn’s?

A
  • Intra-abdominal, retroperitoneal, abdominal wall abscess
  • Symptomatic fibrotic stricture causing intestinal obstruction
  • Enteric fistula
  • SB or CRC from longstanding Crohn’s
  • Intractable disease
How well did you know this?
1
Not at all
2
3
4
5
Perfectly
103
Q

What are the Indications for Emergency Surgery in a Patient With Inflammatory Bowel Disease?

A

For emergency or urgent indications, a total abdominal colectomy with end ileostomy is typically
performed.
• Colectomy indications:
o Toxic megacolon
o Perforation
o Major haemorrhage
o Urgent rx for fulminant cases failing to respond to intensive medical management.

The rectum is not resected but left as a defunctionalised Hartmann’s pouch with or without a mucus fistula.
In patients who develop significant rectal bleeding after total abdominal colectomy, completion
proctectomy with an ultralow Hartmann’s pouch or transanal suturing of the bleeding rectal ulcers can be
performed to achieve haemostasis. After the patient is stabilized, a completion proctectomy with
reconstruction can be performed at a later time.

How well did you know this?
1
Not at all
2
3
4
5
Perfectly
104
Q

You are the ED intern on duty. You are asked to see a 20 yoa man who has presented with perianal
pain that started 24 hours ago and has got worse despite appropriate analgesia. The pain is worse when
he walks or sits.
This has not happened before. He has no other relevant history. On examination you find a fluctuant,
warm, red 3cmx3cm swelling at the anal margin. What is the likely diagnosis?

A

The most likely diagnosis is a perianal abscess.
Perianal abscess - originates from an infected anal crypt gland
• Perianal abscesses transverse distally in the intersphincteric groove into the perianal skin where they
present as a tender fluctuant mass
• If undrained, a perianal abscess can expand into adjacent tissues
(e.g. ischiorectal space, supralevator space) and progress to sepsis
Epidemiology: mean age 40 years (but any age from 20-60), M:F = 2:1
Pathogenesis: A perianal abscess usually originates from an infected anal crypt gland– the glands penetrate
the internal sphincter and end in the intersphincteric plane.
An abscess develops when an anal crypt gland becomes obstructed permitting bacterial growth and
abscess formation
An anal fistula is a connection between two epithelial lined structures, and connects the anal abscess to
the skin of the buttocks
Clinical features:
• Severe pain in anal/rectal area
• Pain may be constant or associated with walking/sitting
• Constitutional symptoms such as fever, malaise
• Purulent discharge if abscess has started to drain spontaneously
• Erythema, lump
• May be no findings of inspection- abscess felt upon DRE
Approximately half of anal abscesses will result in the development of a fistula from the
inciting anal gland to the skin
DDx:
• Supralevator abscess, ischiorectal abscess, intersphincteric abscess
• Anorectal fistula- epithelialsed track between the abscess and perirectal skin or an adjacent organ e.g.
bladder

Jessica Redmond 84
• Internal haemorrhoid
• Pilonidal abscess
Ix:
B –FBC, abscess culture if starting antibiotics, blood cultures if septic
O –PR exam
X –
E
S –CT if suspicious of nonpalpable abscess
Mx:
• Incision and drainage
• Antibiotics -especially in valvular heart disease, immunosuppression, cellulits, diabetes
How well did you know this?
1
Not at all
2
3
4
5
Perfectly
105
Q

You are the ED intern on duty. You are asked to see a 20 yoa man who has presented with perianal
pain that started 24 hours ago and has got worse despite appropriate analgesia. The pain is worse when
he walks or sits.
This has not happened before. He has no other relevant history. On examination you find a fluctuant,
warm, red 3cmx3cm swelling at the anal margin.

What is the likely diagnosis?

What aetiological factors are associated with this condition?

A

Perianal abscess
• Most result from infection of the anal glands with colonic bacteria (cryptoglandular infections)
o The anal canal has 6-14 glands that lie in the plane between the internal and external anal sphincters
o Ducts from these glands pass through the internal sphincter and drain in to the anal crypts at the
dentate line
o This glands may become infected when the crypt is occluded by impacted faeces, oedema (from anal
trauma e.g. passage of hard stool) or as a result of inflammation e.g. Crohns
Aetiological factors/ risk factors:
• Male (M:F = 2:1)
• Crohn’s disease (1/3 of patients)
• Hard stools
• Age 20-60 (mean 40)

How well did you know this?
1
Not at all
2
3
4
5
Perfectly
106
Q

What is Ulcerative Colitis?

A

Ulcerative Colitis is an inflammatory disorder of the mucosa and submucosa of the large bowel. It is
characterised by recurrent acute exacerbations and intervening periods of quiescence or chronic low grade
activity.
Epidemiology
• Typically a young adult with a history of several weeks of frequent loose stools, later streaked with
blood and mucous.
• Attack often starts with attack of gastroenteritis or traveller’s diarrhoea that fails to settle
Aetiology
• Unclear

Jessica Redmond 85
Pathophysiology
• the disease always involves the rectum and extends proximally
• Can processed to backwash colitis in pancolitis patients
• Colonic mucosa becomes acutely inflamed –> neutrophil accumulation –> crypt abscesses
• Sloughing of mucosa –> small superficial ulcers
• Dysplastic changes occur over time –> prolonged or repeated episodes of inflammation can cause
progression to adenocarcinoma
o 5% after 10 years of colitis
o Aggressive
o On average occurs 20 years earlier than general population
• Lead pipe colon - fibrosis from longstanding colitis causing smoothing of haustrations an shortening of
colon
Clinical Features
• Acute attacks:
o Loose blood stained stools streaked with mucous
o Diarrhoea severity corresponds to extent of inflammation
• May pass 20+ stools/day
• Preceded by abdominal cramps
o Fulminant colitis
• Severe dehydration and electrolyte imbalances and blood loss
• Subgroup of patients with severe ulcerative colitis who have more than 10 stools per day,
continuous bleeding, abdominal pain, and distension, and acute, severe toxic symptoms
including fever and anorexia.
o Weight loss
o Extra-colonic features tend to flare up in parallel with colitis, except liver-related ones.
o Tenesmus
• O/E
o PR exam, proctoscopy and sigmoidoscopy
Extra-colonic features:
• Anaemia
o Normochromic normocytic- typically chronic and non specific
• Arthropathy
o Sacroillitis/ankylosing spondylitis, enteropathic arthropathy
• ~20% cases
• Anterior uveitis
o ~10% cases
• Primary sclerosing cholangitis
o Progressive fibrosis of intrahepatic biliary system –> cirrhosis, progressive liver failure, jaundice,
death
o Other liver presentations: chronic active hepatitis, bile duct carcinoma
• Erythema nodosum or pyoderma gangrenosum (purulent skin ulcers)
Complications:
• Toxic megacolon
o Colon dilates massively and patchy necrosis eventually occurs. The patient is systemically ill with
high fever, marked tachycardia and dehydration.
o Perforation and fatal peritonitis required emergency colectomy
o Causes:
• Inflammatory: UC, Crohn’s
• Infectious: bacterial (C diff, salmonella, shigella, campylobacter), parasitic, viral,
pseudomembranous colitis

The diagnosis of toxic megacolon should be considered in all patients presenting with abdominal
distension and acute or chronic diarrhoea. The diagnosis is clinical, based upon the finding of an
enlarged dilated colon on abdominal imaging accompanied by severe systemic toxicity.
o Medical therapy is the first line of treatment for patients with IBD and toxic megacolon and

includes fluid resuscitation and correction of laboratory abnormalities, administration of broad-
spectrum antibiotics, intravenous corticosteroids, complete bowel rest, bowel decompression with

a nasogastric or long intestinal tube, and a surgical consultation.
o The operative procedure of choice is a subtotal colectomy with end-ileostomy and is reserved for
patients who do not improve on medical management.
Investigations
• B: FBC, UEC, LFT, CRP, ESR
o Raised inflammatory markers, anaemia, hypoalbuminemia
• O: stool samples: MCS, OCP, C Diff
• X: barium enema usually not in acute disease, AXR monitoring dilatation as a sign of toxic megacolon -
likely do a CT tbh
• E
• S: proctoscopy, sigmoidoscopy, biopsy, colonoscopy in non acute setting

Ddx
• Crohn’s disease
• Radiation colitis
• Infectious colitis
• Diverticulitis
• Ischaemic colitis
• Vasculitis
• IBS
Management
Management depends on the severity of an individual attack, amount of colon involved, extent of chronic
symptoms and risks of long term complications.
Medical
• 5-ASA
o Mild attacks of proctitis/procto-sigmoiditis treated with local 5-ASA suppositories or enemas
o Mild attack of pan colitis with orals
o Avoid aspirin and NSAIDs as exacerbate despite being related
• Corticosteroids
o Suppositories or enemas used for local rx of rectal inflammation
o Short course high dose oral for severe exacerbations, IV if seriously ill
• No evidence for bowel rest and TPN
• Immunosupressives
o Ciclosporin and azathioprine
• settles on steroids but flares as weaned off steroids
• Maintenance therapy recommended in all patients with left sided colitis, pancolitis, extensive colitis.
• Supportive:
o Avoid anti-diarrhoeals e.g. loperamide - precipitate toxic dilatation
o High calorie and protein diet to counteract losses
Surgical
• Only needed in ~20% patients
• Colectomy indications:
o Urgent rx for fulminant cases failing to respond to intensive medical management
o Acute cases which progress to toxic megacolon, perforation or major haemorrhage
o Chronic disabling symptoms of intractable diarrhoea, urgency, recurring anaemia, failure to
maintain weight and nutrition
o Children with failure to thrive
o Patients with longstanding colitis who develop dysplasia or malignancy
• Total colectomy is curative
• Three main options:
o Subtotal colectomy with ileostomy
• Safest in emergency situation in sick patient on high dose corticosteroids
• Patient left with inflamed rectal stump
• Can change to other option when patient is well
o Proctocolectomy with permanent ileostomy
• Includes removal of rectum
• Generally recommended for elderly patients with sphincter preserving procedures are
inadvisable
o Restorative proctocolectom- ileoanal pouch
• Sphincter preserving operation avoiding permanent ileostomy
• Pouch from terminal ileum anastomosed into upper anal canal
Health maintenance:
• Immunisations
o Live vaccines contraindicated in immunosuppressed people
• Screening for cancer
o CRC, skin, cervical - skin because therapies
• Screening for anxiety and depression
• Prevention and monitoring of bone loss
o Increased risk, especially if on glucocorticoids
• Lab monitoring of complications of IBD
o Anaemia, b12 and folic acid deficiency
• Medication side effect monitoring

How well did you know this?
1
Not at all
2
3
4
5
Perfectly
107
Q

This lesion is found at colonoscopy in the proximal colon. What is it? How should it be treated? Is
follow up or surveillance needed?

A

If Polyp:

Classified as:
• Non-neoplastic
o Hamartomatous – glandular epithelium supported by smooth muscle cells that is contiguous with
the musularis mucosa – associated with Peutx-Jeghers syndrome
o Serrated (neoplastic or non-neoplastic)
o Submucosal (neoplastic or non-neoplastic)
• Neoplastic – adenoma or carcinoma

Management
• Polypectomy during colonoscopy

Surveillance
• If only 1-2 adenomas ≤10mm on baseline colonoscopy = lower risk – follow-up colonoscopy in 5 years
• If advanced adenomas (≥10mm, villous histology or high-grade dysplasia) or multiple adenomas (between
3-10) at baseline colonoscopy = higher risk – follow-up colonoscopy in 3 years
• 3-6 month and 12 months if sessile and laterally spreading

How well did you know this?
1
Not at all
2
3
4
5
Perfectly
108
Q

This 22 year old female patient presents with vague intermittent right sided abdominal discomfort
and some mild diarrhoea. What abnormality is evident here in the terminal Ileum and right colon at
colonoscopy?

What histopathological findings are anticipated in the biopsy findings of this lesion? What are possible
diagnoses?

A

?Likely Crohn’s:

Abnormality
• Cobblestoning
• Linear ulcers
• Granulomas
• Stricture
Likely histology:
• Transmural inflammation- lymphoid aggregates
• Granulomas - nonnectrotising
• Cobblestoning
• Linear ulcers/fissures
• Skip lesions
• Submucosal fibrosis
Possible diagnoses:
• Crohn's disease
• Ulcerative Colitis with backwash ileitis if pancolitis
• NSAID enteropathy
• Vasculitis
• Eosinophilic gastroenteritis
• Sarcoidosis
How well did you know this?
1
Not at all
2
3
4
5
Perfectly
109
Q

This lesion is found at colonoscopy in the sigmoid colon of an 85 year old male admitted acutely
with a near complete large bowel obstruction.

What is the lesion?

What initial treatment is most appropriate for this patient whose CT scan shows a large number of bilobar hepatic metastases?

What are the major risks of this approach?

A

CRC
Initial treatment
If the metastases are potentially resectable for cure, then an aggressive surgical approach is warranted for
both the primary and metastatic sites with the aim of curing the patient. If there are five or more
simultaneous, potentially resectable hepatic metastases (unless all are located in one
lobe), bilobar involvement, or if disease is borderline resectable due to location, initial chemotherapy
followed by reassessment and delayed resection is probably a better strategy than upfront surgery.
There are two main options for this bowel lesion. Acute resection or stenting to decompress the bowel

Jessica Redmond 90
followed by resection if deemed appropriate for the patient at a later stage. I believe the best option is
stenting. Stenting is largely palliative
Major risks of approach:
Risks of neoadjuvant chemo:
-failure to work
-toxicity
-risk of becoming unfit for surgery
-
Risks of liver resection:
• haemorrhage
• Intra-abdominal sepsis
• Liver failure
• Bile leak
• Liver steatosis
• Mortality
Risks of stenting:
- perforation
- failure to decompress
- tumour ingrowth and re-obstruction
- stent migration
- bleeding
- risks of anaesthetic
--------
This patient's bowel obstruction needs to be managed.
This 85 yo man may not be able to survive surgery, and metastases means most likely need palliative
intervention.
To manage the bowel obstruction:
There are two main options for this bowel lesion. Acute resection or stenting to decompress the bowel
followed by resection if deemed appropriate for the patient at a later stage. I believe the best option is
stenting.
Risks of stenting:
- perforation
- failure to decompress
- tumour ingrowth and re-obstruction
- stent migration
- bleeding
- risks of anaesthetic

With integration of surgery and chemotherapy, long-term survival can be achieved in as many as 50
percent of cases. For patients with borderline resectable metastatic disease, downstaging with neoadjuvant
chemotherapy may permit later successful resection. Even patients who are not candidates for a curative
resection can benefit from surgical palliation for symptoms of obstruction and bleeding from the
primary tumor.
4 rules for considering liver metastases resectable:
1) primary cancer is resectable,

Jessica Redmond 91
2) no extra-hepatic metastases,
3) at least 2 of 8 liver segments will remain preserving adequate hepatic function,
4) no involvement of major structures (portal vein, hepatic artery)
There is the possibility of neoadjuvant chemotherapy to allow for stage resection of liver metastases,
however, it is likely that if there is not enough preserved hepatic function from resection, than this would
not be possible, and definitely not curative.
On the other hand, if there are five or more simultaneous potentially resectable hepatic metastases (unless
all are located in one lobe), bilobar involvement, or if disease is borderline resectable due to location, initial
chemotherapy followed by reassessment and delayed resection is probably a better strategy.
As such, the focus would be on palliative control as opposed to curative control. Although this decision
likely needs to be made post-neoadjuvant chemotherapy
If 2 stage metastases resection:
There are two main options for this bowel lesion. Acute resection or stenting to decompress the bowel
followed by resection if deemed appropriate for the patient at a later stage. I believe the best option is
stenting.
Can do palliation

How well did you know this?
1
Not at all
2
3
4
5
Perfectly
110
Q

What is the obvious abnormality on this X-ray and in this operative photo?
In the presence of symptoms of large bowel obstruction what initial treatment options exist?
What can be done to prevent recurrence?

A
?volvulus
Initial treatment of large bowel obstruction
• Flexible sigmoidoscopy to detorse twisted segment
• Rectal tube
Prevent recurrence:
• Endoscopic detorsion
If acutely unwell:
• O2
• IV fluids
• Blood transfusions if anaemic
• NG decompression
• Pre-operative Abx
• If perf- laporatomy

Volvulus? – Mx: Endoscopic decompression of sigmoid volvulus followed by elective resection for sigmoid
volvulus because of high rate or recurrence (up to 50%) with endoscopic decompression alone
• Urgent intervention needed due to risk of perf
• Rectal tube in situ for 24 hours with repeat AXR
• Endoscopic detorsion
• ?resection of sigmoid to prevent recurrence
• the literature indicates that sigmoid resection, done either as a Hartmann’s procedure when a
gangrenous colon is discovered at laparotomy or as a one-stage resection with primary anastomosis in
the setting of a viable colon, has the lowest rate of recurrence

Colorectal tumour? – Mx: Stenting
• May be ‘bridge to surgery’ i.e. provide time to correct fluid and electrolyte balance prior to operation or be
palliative

How well did you know this?
1
Not at all
2
3
4
5
Perfectly
111
Q

This is a photo of a patient with a loop de-functioning colostomy. What is the abnormality seen here?
What is the mechanism?
What treatment options are there?

A

Parastomal Hernia

Most frequent complication - 50% pts

Type of incisional hernia.

Stoma creation introduces an abdominal wall defect, the trephine, for which no healing is expected.
A parastomal hernia forms as the trephine is continually stretched by the forces tangential to its circumference.

Mx:
Asymptomatic: conservative management
Prosthetic mesh repair for acute obstruction or elective repair

Mucocutaneous separation

Separation of stoma from peristomal skin.
Leakage and skin irritation.

Mx:
Surgery

Stomal Necrosis

Early post operative period- venous congestion or arterial insufficiency from tight fascial opening.

Mx: Surgery

Dermatitis
Exposure to damaging effluent can produce severe skin breakdown in the area exposed to the drainage; this type of skin damage is characterized by severely denuded skin along the inferior aspect of the stoma and is most common among ileostomy patients and patients with problematic stomas.

Mx: Steroids

Stomal retraction
Stomal retraction is defined as a stoma that is 0.5 cm or more below the skin surface within six weeks of construction, typically as a result of tension on the stoma.

Mx:
Use of a convex pouching system and the use of a belt or binder. If these measures fail to provide a secure pouch seal, surgical revision may be needed.

Stomal Prolapse
Prolapse is the telescoping of the intestine out from the stoma and can occur with any type of stoma. Large abdominal trephine, increased intraabdominal pressure, and a redundant loop of bowel proximal to the stoma.

Mx:
Mannaged conservatively with cool compress and/or application of an osmotic agent (eg, table sugar or
honey) to reduce edema, followed by manual reduction of the prolapse and application of a binder with a prolapse over-belt to keep the bowel recued into the abdomen, or by pouching modifications to accommodate the prolapsed bowel when reduction
cannot be established or maintained.

How well did you know this?
1
Not at all
2
3
4
5
Perfectly
112
Q

What is the lesion visible here at colonoscopy and what has been done?
What is the possible histology?
What additional endoscopic manoeuvre is needed before completion of the colonoscopy?

A

?polyp - polypectomy
Histology- adenoma, adenocarcinoma

?adenoma – by definition histology is dysplasia
• Most colorectal cancers are caused by adenomas but only 5% of adenomas progress to cancer
• Development of adenoma to cancer usually takes 7-10 years
• Advanced adenoma is any adenoma with high-grade dysplasia, size >10mm, an adenoma with a villous
component
• Adenomas can be classified as tubular, villous or tubulovillous
• Mx: Polypectomy via snare excision

How well did you know this?
1
Not at all
2
3
4
5
Perfectly
113
Q

This patient presents with anal discharge and pain.
What is the likely nature of this lesion?
What does the initial management involve?
How is it usually treated?
What is the role of surgery?

A

Anal cancer
The lesions is likely a squamous cell carcinoma
Staging consists of biopsy of the primary tumour, CT chest abdomen and pelvis and PET
scans.
Chemoradiotherapy is the first line treatment, with radiation therapy administered to the
sites of the primary disease and lymphatic spread, usually with prophylactic irradiation of
clinically negative groins.
Local excision is an option for patients with T1 tumours less than 1 cm in size with followup
prophylactic chemoradiotherapy. Surgery also has a place in recurrent or persistent
disease after chemoradiotherapy. Although prognosis is poor abdominoperineal resection
offers the potential for long term survival.

NB- tumours above the dentate line, similar to rectal cancers, drain to the mesorectal and
internal iliac nodes whereas tumours below the dentate line may also spread to the
superficial inguinal and external iliac nodes.

Note: May also be a fistula. Initial management is surgery, and the mainstay of treatment is a fistulotomy.

How well did you know this?
1
Not at all
2
3
4
5
Perfectly
114
Q

This patient presented with severe anal pain after lifting a heavy piece of furniture. He feels this
perianal swelling but does not present to his GP for a couple of days.
What is the diagnosis?
What is the treatment?

A

Prolapsed internal haemorrhoid.
Management:
Medical
Grade 1 hydrocortisone cream short term for pruritis and pain

Anaesthetics- benzocaine rectal ointment
Stool softeners to decrease straining
Antispasmodics
Surgical:
Sclerotherapy- injection into the submucosal tissue of sclerosing agent with proctoscope to
obliterate the haemorrhoidal vessels
Banding- Cone around the base of the haemorrhoids to constrict the vessels
Haemorrhoidectomy- Indications include 3rd degree haemorrhoids. The mass is excised
with the overlying skin and mucosa, followed by secondary intention healing or staples for
larger haemorrhoids.

NB- external haemorrhoids are painful whereas internal haemorrhoids are not painful

How well did you know this?
1
Not at all
2
3
4
5
Perfectly
115
Q

This man had had an abscess drained in the perianal region.

Q1: What is the common presentation of this problem?

Q2: What is the first line of management?

Q3: What bacteria are usually cultured?

Q4: What is the usual cause of this?

A

Q1: What is the common presentation of this problem?
Constant and severe perianal pain and local tenderness

Q2: What is the first line of management?
Surgical incision and drainage

Q3: What bacteria are usually cultured?
Staphylococcus or E.coli

Q4: What is the usual cause of this?
Usually begin as acute purulent infections of the anal glands in intersphincteric spaces and
obstruct the ducts.
Obstruction- constipation, trauma or inflammation (Crohn’s disease or UC)

How well did you know this?
1
Not at all
2
3
4
5
Perfectly
116
Q

This patient had had an endoscopic mucosal resection (EMR) of a colonic polyp.

  1. What are the complications of this procedure?
  2. What is the likely Pathology here?
  3. What would normally occur if there is found to be a tiny focus of completely excised, well differentiated invasive cancer in the specimen that is far from the
    resection line?
A
  1. What are the complications of this procedure?
    Bleeding- 2-11%
    Perforation
    Failure to completely resect polyp
  2. What is the likely Pathology here?
    Adenoma
  3. What would normally occur if there is found to be a tiny focus of completely excised, well differentiated invasive cancer in the specimen that is far from the
    resection line?
    Frequent surveillance.
    If advanced adenomas (>1cm or high grade dysplasia) or multiple adenomas (between 3 and 10) at baseline
    colonoscopy = high risk = colonoscopy in 3 years.
    12 month if sessile and laterally spreading
How well did you know this?
1
Not at all
2
3
4
5
Perfectly
117
Q

This is a photo of the anal canal taken at colonoscopy by retroflexion of the scope.
Q1. What does this show?
Q2. How do these present clinically?
Q3. How are they treated?

A
Q1. What does this show?
?haemorrhoids
Anal rectal carcinoma
Fissure
polyp
Anal cancers

Q2. How do these present clinically?
Haematochezia (45% of patients), anal pain, sensation of an anal mass, pruritis, anal
discharge, on DRE they may have a localised firm or hard ulcer/ growth with an irregular
surface and edge, possibly surrounded by woody induration.

Q3. How are they treated?
The lesion is confirmed by biopsy
1. Chemoradiotherapy
2. Abdominoperineal resection

How well did you know this?
1
Not at all
2
3
4
5
Perfectly
118
Q

This patient presents with perianal itching.
Q1. What are the lesions you can see in the perianal area?
Q2. What is the responsible agent that causes this condition?
Q3. What treatments are available?

A

Q1. What are the lesions you can see in the perianal area?
Anogenital warts/ perianal warts/ condylomata acuminata

Other causes:

  • Faecal soilage
  • Herpes
  • Mucous leakage
  • Diarrhoea
  • Haemorrhoids
  • Skin disorders - contact dermatitis from soaps, shower gel, cream, talc etc
  • Fungal infections (candida)
  • Genital warts:
  • > Chlamydia trachomatis, syphillis, gonorrhoea

Q2. What is the responsible agent that causes this condition?
HPV. Types 6 and 11 are normally the causative agents of anal warts. Transmitted sexually.

Q3. What treatments are available?
All patients should be offered treatment although although warts may resolve.
Topical agents- podophyllin (anti-mitotic)
Trichloracetic acid (caustic causing chemical coagulation of protein) Immunomodulators such as imiquimod

Cryotherapy
Surgical excision under local or general anaesthetic (only practical option when large numbers are present). Care must be taken to prevent the complication of anal
stenosis.

How well did you know this?
1
Not at all
2
3
4
5
Perfectly
119
Q

How do you go about assessing a 40 year old woman who presents with a lump in
her lateral neck?

A

The assessment of a neck lump requires a thorough history, examination and further investigations:

History
Characteristics of the mass- duration, growth pattern, absence or presence of pain.
Masses that are present for years with little changes are more likely to be benign neoplasms (e.g benign salivary gland tumours, schwannomas or paragangliomas)
Rapidly expanding masses raise concern for infectious processes or rapidly growing lymphomas.
Masses that fluctuate over time and increase with viral illness are often congenital cysts.
Symptoms- pain, voice change, hoarseness, dysphagia and otalgia.
Pain is often associated with rapid growth but can also be related to direct neural invasion in the setting of certain malignancies.
Voice change, hoarseness, dysphagia and otalgia may indicate cervical lymph node metastasis from an underlying aerodigestive malignancy.
Presence of fever, night sweats, or weight loss- suspicious of lymphoma.

Tobacco, alcohol and drug use
HIV status
Occupational exposures
Recent travel history

Examination:
Mass location
Pre auricular and angle of jaw- likely related to the parotid gland.
Posterior triangle- high index of suspicion for malignancy
Supraclavicular masses- metastasis from below the neck, especially the lungs and GIT
Characteristics of the mass- size, shape, consistency, tenderness, mobility and colour
Discrete, firm, mobile or rubbery but not rock hard masses are usually lymph nodes
Rock hard masses that are fixed are likely malignancy
Soft ballotable masses are usually congenital cysts
A firm lateral neck mass that moves side to side but not up and down indicates
involvement with the carotid sheath such as a carotid body tumour or schwannoma.
Pulsatile mass or bruit suggests a vascular lesion
An immobile midline neck mass that elevates with swallowing indicates a thyroid source

Cranial nerve exam
Abdominal exam
GIT exam
Investigations
FBC, ESR/CRP, EBV serology when adenopathy is diffuse, HIV serology in at risk patients
Ultrasound
CT
MRI?
PET?
FNA or core biopsy in suspicious lesions
Consider excision biopsy

NB- A neck mass in an adult over the age of 40 should be considered neoplastic and
potentially malignant until proven otherwise.

120
Q

What are the main types of Thyroid cancer and what is their prognosis?

A

There are 5 main types of thyroid cancer:

  1. Papillary thyroid cancer- Papillary thyroid cancer is the most common type of thyroid cancer. It develops from follicular cells and usually grows slowly. Papillary thyroid cancers can often spread to the lymph nodes. They are usually curable.
  2. Follicular thyroid cancer- develop from follicular cells and usually grow slowly. They rarely spread to the lymph nodes. They are usually curable.
  3. Hurthle cell cancers- Arise from the Hurthle cells, a specific type of follicular cell, and are more likely to spread to the lymph nodes than other follicular thyroid cancers.
  4. Medullary thyroid cancer- develop in the C cells. Medullary thyroid cancers can be controlled if diagnosed before lymphatic spread occurs. Medullary thyroid cancers are genetic in 25% of cases.
  5. Anaplastic thyroid cancers- This type is rare, accounting for about 1% of thyroid cancer. It
    is fast growing, poorly differentiated thyroid cancer. As anaplastic thyroid cancers are so fast growing they have a poor prognosis.
121
Q

What are the treatment options for a patient who presents with a toxic goitre
(hyperthyroidism associated with an enlarged, overactive thyroid)?

A

Thionamides (carbimazole and propylthiouracil) are used in patients with thyrotoxicosis and a goitre due to Grave’s disease.

Patients with multinodular goitre will also respond to thionamide medication, but definitive treatment with surgery is generally preferred.

Radioactive iodine is another option and reduces goitre size by about 50% in the majority of patients over 6-12 months. It can be used in patients with toxic adenoma. However radioactive iodine carries a significant risk of causing hypothyroidism over time and so patients require annual TSH follow up.

Surgery is appropriate for patients who have troubling compressive symptoms or who fail to respond to medical management. Surgical treatment is with a thyroidectomy, which involves partial or total removal of the thyroid gland.

122
Q
This patient has become aware of a skin abnormality for years that has developed
hair growth during adolescence.
Q1. What is the abnormality?
Q2. What is it's significance?
Q3. What treatment is needed?
A

Supernumerary nipple?- commonly arise along the two vertical milk lines that originate in
the axilla, descend through the region where nipples are typically situated and terminate at
the groin.

NB may also be a melanocytic naevi

Q2. What is it’s significance
Supernumerary nipples are typically asymptomatic however may enlarge during puberty,
swell and become premenstrually tender and undergo lactation. Supernumerary nipples
can also undergo similar disease to normal breast tissue including fibroadenoma,
adenoma, cyst, abscess, mastitis and breast cancer.

Q3. What treatment is needed?
Usually no treatment is required. Surgical removal can be considered for cosmetic purposes or if there is discomfort from lactation or tenderness.

123
Q

This is the abdominal photograph of an 80 year old woman who presents with a
painful lump in the right groin. She has had it for 24 hours.
Q1. What could it be? She has had generalised colicky abdominal pain from about 12 hours and some vomiting.
Q2. What is the most likely diagnosis here?
Q3. What is the most appropriate treatment strategy?

A
  • Femoral hernia that has become incarcerated and strangulated and causing a small bowel obstruction.
  • Inguinal hernia
  • Lymphadenopathy
  • Spigelian hernia (a hernia through the spigelian fascia, which is the aponeurotic layer between the rectus abdominus muscle medially and the semilunar line laterally)
    Q2. What is the most likely diagnosis here?
    Incarcerated femoral hernia causing a mechanical bowel obstruction.
    Q3. What is the most appropriate treatment strategy?
    Resuscitation and urgent surgery to resect and infarcted bowel and mesh repair the abnormality that lead to the hernia.
124
Q

This lady had a swelling on the back of her lower neck. It has been present for
years. It had slowly enlarged over that time and is producing no other symptoms.
Q1. What is the most likely diagnosis?
Q2. What tests other than physical examination are likely to be helpful?
Q3. What are the common complications of operative removal of this?

A

Q1. What is the most likely diagnosis?
Lipoma- benign neoplasm of adipose tissue
Differential diagnosis- epidermoid/ sebaceous cyst (has a punctum), liposarcoma, abscess, malignancy.

Q2. What tests other than physical examination are likely to be helpful?
Ultrasound- diagnostic
MRI- a lipoma will appear as homogenous and superficial. MRI is normally done to rule out
differentials such as liposarcoma.
Biopsy- shows histopathological features of a lipoma.

Q3. What are the common complications of operative removal of this?
Operative removal of lipoma is needed if there is pain, for cosmetic reasons or if there is concerns over diagnosis.
Complications of surgery include scarring, seroma (pocket of clear serous fluid),
haematoma, wound infection, nerve injury

125
Q

A 65 year old woman presents to the ED with vomiting, abdominal distention and colicky pain. She has never had an operation. She has a CT scan.
Q1. What does this show?
Q2. What does the surgical registrar do the minute he sees this scan?
Q3. What does he find? What is the treatment?

A

Q1. What does this show?
Likely shows a hernia

Q2. What does the surgical registrar do the minute he sees this scan?
The surgical registrar should attempt to reduce the hernia

Q3. What does he find? What is the treatment?
It is most likely irreducible. Therefore treatment should be as per SBO and hernia repair.

Initial:
Primary survey and fluid resuscitation
Correct electrolyte imbalances
NG tube decompression of GI tract
Pain management
Preoperative antibiotics
FBC, UEC, group and hold, coags

Definitive:
Surgical repair

126
Q

What is the massive transfusion protocol?

In what circumstances should it be used?

A

The massive transfusion protocol is a protocol used in critically bleeding patients anticipated to require a massive transfusion in order to maintain adequate tissue perfusion and oxygenation, A massive transfusion is defined as a replacement of the entire blood volume in 24 hours or 50% of the blood volume in 4 hours.

The massive transfusion protocol should be activated for patients with any of the following:

  • Actual or anticipated transfusion of 4 units of red blood cells in less than 4 hours + haemodynamically unstable + anticipated ongoing bleeding.
  • Severe thoracic, abdominal, pelvic or multiple long bone trauma.
  • Major obstetric, gastrointestinal or surgical bleeding
127
Q

A 75 year old lady was admitted with abdominal pain and vomiting, this Xray was taken.

Q1. What are the possible causes of her small bowel obstruction?

Q2. What are the priorities in managing her in the emergency department?

A
Q1
Adhesions
Hernia
Crohns disease
Malignancy- adenocarcinoma
Volvulus
Benign mass- adenoma, lipoma
Intra-abdominal abscess

Q2
Primary survey and appropriate resuscitation
AXR and erect CXR
Surgical consult- urgent if potentially strangulated bowel or very dilated as there is a risk of perforation and peritonitis.
Correct electrolyte abnormalities and losses from vomiting.
Decompression with an NG tube since the patient is vomiting.
Pain management

128
Q

Q1) What is this structure and what operation is being attempted?

Q2) What symptoms would this patient have presented with?

Q3) What is the modern management of this condition?

A

Q1)
Gallbladder + cholecystectomy

Q2)
Symptoms of cholecystitis
Clinically it presents as prolonged (>4 hours) severe RUQ pain that may radiate to the right shoulder or back with fever, abdominal guarding, tachycardia, nausea and vomiting, anorexia and a positive Murphy’s sign.

Q3)
Management is with analgesia, IV fluids, NBM, broad spectrum antibiotics (not required in simple biliary colic) and a laprascopic cholecystectomy in the same admission.

129
Q

How do you categorise causes of abdominal pain?

A

Causes of abdominal pain are best categorised based on location, by dividing the abdomen into 9 anatomical location. Pain in each location has common causes.

Can also mention Surgical sieve: VINDICATE or VITAMIN CDEF

130
Q

Where is the Appendix Found?

A

The appendix is a blind-ending tube arising from the caecum at the meeting point of three taeniae coli, just distal to the ileocaecal junction.
The appendix base thus lies in the right iliac fossa close to Mcburney’s point (two thirds of the way along a line from the umbilicus to the ASIS).
Apart from the true retrocaecal appendix (appendix lies retroperitoneally posterior to the caecum) the organ probably floats in a broad arc about its base. In about 30% of cases it lies over the pelvic brim.
It is Retrocaecal and pelvic.

131
Q

When should you commence antibiotics in a patient who has symptoms and signs of appendicitis?

A

Although several studies have shown that uncomplicated appendicitis can be treated with antibiotics alone there is not enough evidence to currently recommend that approach.
Therefore antibiotic therapy should only be started once the surgeon has decided to operate.
Ideally the antibiotics should be given 1-2 hours before the surgery commences however in practice they are usually given in the operating theatre.
For uncomplicated appendicitis (appendicitis without localised ischaemia, perforation and the development of appendiceal abscess or generalised peritonitis) antibiotics may be discontinued post operatively. For complicated cases antibiotic treatment should be continued for at least 5 days post op.
Antibiotic choice is usually with triple therapy (ampicillin + gentamicin + metronidazole)

132
Q

What are the important differential diagnoses to consider when managing a 17 year old woman who has the symptoms and signs of appendicitis?

A

Ectopic pregnancy- presents with missed menstrual period, right lower quadrant pain or pelvic pain, generally with some degree of vaginal bleeding or spotting. Cervical motion tenderness may be present on pelvic exam. Investigations include beta HCG hormone levels, which will be raised in the urine and blood and ultrasound which will reveal a mass in the fallopian tubes.

Ovarian torsion- Presents with right lower quadrant pain and may present with a mass in the right lower quadrant. Investigation is with ultrasound, which will show an ovarian cyst and decreased blood flow/ ischaemia.

Pelvic inflammatory disease- Occurs when bacteria from the vagina spread to the cervix or uterus. Presents with bilateral lower quadrant tenderness, usually within 5 days of the last menstrual cycle with purulent discharge from the cervical os. Investigation is with an endocervical swab which may show chlamydia trachomatis.

Ruptured Graafian follicle- occurs mid menstrual cycle (12-16 days post menstruation) and causes a brief period of lower abdominal pain not usually associated with nausea, vomiting or fever. The tenderness is usually diffuse.

  • Pyelonephritis/ cystitis- UTI
  • Right sided ureteric stones
  • Pancreatitis
  • Cholecystitis
  • Crohn’s disease
  • Mesenteric adenitis
  • Gastroenteritis
  • Meckel’s diverticulum
133
Q

You (the candidate) are an Emergency Department resident. How would you proceed to manage a 22 year old female patient who presents to you with a 24 hour history of Right iliac fossa pain?

A

ABCDE:
History:
Socrates of pain
Nausea and vomiting (75% of patients with acute appendicitis)
Anorexia (almost always associated with appendicitis)
Migration of pain from periumbilical to RIF
Changes in bowel habits (uncommon with acute appendicitis)
Recent cold/ flu like symptoms (mesenteric adenitis)
Sexual activity (STI and ectopic pregnancy)
Menstrual history
Ongoing GIT issues such as IBD
Active or past malignancy
Past appendectomy

Examination:
Vital signs
Abdominal exam including signs of peritonism, McBurney’s sign (tenderness and rebound tenderness at McBurney’s point [1/3 of the way from the ASIS to the umbilicus on the right side]), Rovsing sign (pain in RLQ after compressing LLQ), psoas sign (pain on flexion of the right leg)
Reduced bowel sounds
Pelvic examination
Investigations:
FBC, UEC, bHCG, CRP
Urine MCS and dipstick
Erect CXR if considering bowel perforation
? abdo/pelvis USS

This information can then be used to assess the Alvarado score for appendicitis.

Initial management for an appendicitis;
NBM
IV fluids
Analgesia
Monitor vital signs
Antibiotics according to local protocols if the decision to operate is made
Call surgical registrar
134
Q

What is the indication for Imaging in Suspected Appendicitis?

A

Imaging is not essential for the diagnosis of appendicitis, the diagnosis is made clinically. Imaging is indicated when the diagnosis of appendicitis is unclear and the patient isn’t too unwell such that delaying surgery may lead to perforation.

An abdominal and pelvic ultrasound scan is performed mostly in female patients when the history is unclear and clinical findings are equivocal. In obese patients the use is limited due to body habitus.

CT abdomen and pelvis may have some place in imaging in acute appendicitis. However CT should not be used in younger populations due to the radiation exposure or in female patients who may be pregnant. It is more classically used in older patients who present with a possible appendicitis as the incidence is lower in this population and common differentials, such as cancer, are potentially more life threatening.

135
Q

Why does the pain and discomfort of appendicitis classically ‘migrate’ from generalised abdominal pain to become localised in the RIF?

A

Acute appendicitis classically begins with poorly localised, colicky central abdominal visceral pain, lasting a few hours. This results from smooth muscle spasm and stretching as a reaction to appendiceal obstruction. The pain is poorly localised due to the relatively small number of nociceptors. The pain also travels along non-myelinated C-fibres, which are slow conducting and cause a dull, poorly localised pain.
As inflammation advances over the ensuing 12-24 hours it progresses through the appendiceal wall to involve the parietal peritoneum, which is innervated somatically by a larger number of nociceptors containing a-delta fibres. Consequently the pain typically becomes localised to the right iliac fossa.

NB- Non myelinated C fibres run with autonomic nerves and terminate in the superficial dorsal horn, transmitting signals to the brain through the dorsal column to the limbic area.
A-delta fibres run in somatic nerves, terminate in the deeper dorsal horn and transmit signals up the faster spinothalamic spinal tract, which terminates in the thalamus and cortex.

136
Q

What are the indications for a laparoscopic approach to appendicectomy?

A

The indications for laprascopic approach to appendicectomy are:
Patients who are clearly unwell and in need of a diagnosis if unclear.
Women of menstruating age to allow for visualisation of other organs.
Children.
Obese patients to reduce the risk of wound infection from a large incision.

Advantages
Diagnostic as well as therapeutic
Quicker return to work post op
Less post op pain
Fewer adhesions
Better cosmetically
Fewer wound infections
Technical ease in obese patients
Ability to visualise other organs- particularly pelvic organs
Disadvantages:
Higher rate of intra-abdominal abscess
Longer operative time
Higher operative costs
Technically more demanding on the surgeon
137
Q

What are the Classical Symptoms and Signs of Acute Appendicitis?

A

Acute appendicitis classically begins with poorly localised, colicky central abdominal visceral pain, generally lasting less than 72 hours. Anorexia, nausea and vomiting (1-3 episodes of vomiting) and pyrexia (fever of less than 38˚C) often occur at this stage.
As inflammation advances it progresses through the appendiceal wall to involve the parietal peritoneum. The pain typically becomes localised to the right iliac fossa. Signs of local peritonitis can be elicited such as tenderness, guarding and rebound tenderness. Specifically, McBurney’s sign (pain and guarding on palpation of McBurney’s point), Rovsing sign (pain elicited in the right lower quadrant in response to palpation of the left lower quadrant), Obturator sign (internal rotation of flexed right thigh causes pain in RLQ) and Psoas sign may be present.

This classic picture is seen in less than half of all cases, largely because localising symptoms and signs vary with anatomical relations of the inflamed appendix and the vigour of the bodies defences.

NB: If the inflamed appendix lies in the pelvis near the rectum it may cause local irritation and diarrhoea. If it lies near the bladder or ureter inflammation may cause urinary frequency, dysuria and pyuria (the presence of pus in the urine). An inflamed retrocaecal appendix produces none of the usual localising symptoms or signs but may irritate the psoas major muscle causing involuntary hip flexion and pain on extension. A high retrocaecal appendix may cause pain and tenderness below the right costal margin. An inflamed appendix near the fallopian tubes causes pelvic pain suggestive of an acute gynaecological disorder.
If left untreated the inflamed appendix may become gangrenous (tissue death due to loss of blood supply) after 12-24 hours and perforate, causing peritonitis unless sealed off by the omentum. The whole abdomen becomes rigid and tender and there is marked systemic toxicity.

138
Q

What is a sentinel node biopsy in the management of breast cancer?

A

A sentinel lymph node is defined as the first lymph node to which cancer cells are most likely to spread from a primary tumour. In breast cancer the lymphatic drainage is generally to the axillae (can be internal mammary and supraclavicular nodal groups). Lymphatic spread nearly always follows a predictable pattern with a sentinel node the first to be affected, so the standard diagnostic procedure is a sentinel lymph node biopsy.
The operation is performed after a diagnosis of breast cancer is made. The node is identified using a radioactive isotope bound to albumin, injected next to the tumour 12 hours before surgery and a blue dye injected into the periareolar area at the start of the surgery. Radioactivity is detected at surgery and the visibly blue nodes are excised and examined. If the nodes are positive an axillary lymph node clearance can be completed and is the standard approach for ned []? positive patients.
NB inflammatory breast cancer is a contraindication to sentinel lymph node biopsy

139
Q

What is a Hernia?

A

A hernia is a protrusion, bulge or projection of an organ through the body wall that normally contains it, such as the abdominal wall. They are typically classified by aetiology and location.
Hernias can be reducible (contents of the sac returns spontaneously or with manual pressure), irreducible (contents can not be returned with manual pressure) or strangulated (blood supply to the contents of the hernia is compromised causing significant pain).
Some examples of hernias include inguinal (direct and indirect), femoral, epigastric and umbilical.

140
Q

What is the anatomical difference between an inguinal hernia and a femoral hernia?

A

In terms of the anatomical difference between inguinal and femoral hernias it is important to note that inguinal hernias can be direct or indirect. Inguinal and femoral hernias are both considered to be groin hernias. The inguinal hernias occur in the inguinal canal superior to the inguinal ligament whereas femoral hernias occur in the femoral canal, inferior to the inguinal ligament.

  • Indirect inguinal- These are the most common type of hernia in males and females (5x more common than direct) and are 7x more common in males (due to persistence of the processes vaginalis in the testicular descent). They are usually congenital abnormalities where abdominal contents, such as mesenteric fat, small bowel loops or mobile colon segments, protrude through the internal inguinal ring, follow anterior to the spermatic chord in males, and exit through the external inguinal ring into the scrotum in males and into the labia majorus in females.
  • Direct inguinal- Occur through Hesselbach’s triangle (protruding through transversals fascia on the posterior wall of the inguinal canal) which is formed by:
  • Inguinal ligament inferiorly
  • Inferior epigastric vessels laterally
  • Rectus abdominus medially
  • Direct hernias occur as a result of weakness in the floor of the inguinal canal, often due to connective tissue abnormalities or acquired causes of increased intra abdominal pressure (COPD, bladder outflow obstruction, chronic constipation. They are more common in men over 50 and often occur bilaterally
  • Femoral hernias develop in the empty space of the femoral canal in the medial aspect of the femoral triangle, entering through the femoral ring, inferior to the inguinal ligament. They are more common in women due to wider bony pelvis’ and are more likely to strangulate as the borders are tough and not easily extendible. They will also cause compression of the femoral vein. The femoral canal (the opening of which is the femoral ring where the femoral hernias protrude through) has four borders:
  • Medial border- lacunar ligament
  • Lateral border- femoral vein
  • Anterior border- inguinal ligament
  • Posterior border- pectineal ligament, superior ramus of the pubic bone and the pectineus muscle
141
Q

On examination, what is the difference between an inguinal hernia and a femoral hernia?

A

The key is the position of the hernia in relation to the inguinal ligament. Inguinal hernias are above the inguinal ligament whereas femoral hernias are below the inguinal ligament.
Initially in the exam the patient is standing and the groin/labia majora is checked for the hernia. The patient is then asked to cough or valsalva with the hand over the bulge.

An inguinal hernia is above the inguinal ligament, is likely reducible, is better felt on cough, often descends laterally and above the pubic tubercle and is more likely indirect if it is in the scrotum or labia majora. When placing a finger over the pubic tubercle hernias felt against the side of the finer are likely direct and hernias felt against the tip of the finger are likely indirect.
A femoral hernia is below the inguinal ligament and medial to the femoral artery, is rarely reducible and is likely strangulated.

142
Q

What is the importance of deciding whether a hernia is inguinal or femoral?

A

It is important to decide whether a hernia is inguinal or femoral because the approach to management is different for each. Watchful waiting may be an option for asymptomatic or minimally symptomatic inguinal hernias, but this is not recommended in femoral hernias as they are more likely to strangulate.

The surgical options for hernia repair include either an open repair or laprascopic repair with stitching of the protrusion point and placing of a mesh. In a femoral hernia the approach may be from above (doesn’t interfere with inguinal structures) or below (the usual approach as compromised bowel can be resected) the inguinal ligament.

143
Q

What is the difference between an incarcerated or irreducible hernia, and a strangulated hernia?

A

An incarcerated/ irreducible hernia is entrapped. The hernia is trapped in the hernial sac and hence can not be reduced back into the abdominal wall
In a strangulated hernia the blood supply to the contents of the hernia have ceased due to compression of the hernial orifice. Initially, lymphatic and venous channels are obstructed thus there is oedema and venous congestion, and if arterial pressure and tissue pressure are equal arterial flow ceases and tissue necrosis ensues due to ischaemia. When bowel is involved the patient could present with peritonitis, and will be in severe pain.

NB- Femoral hernias are more likely to strangulate as the surrounding structures are not as extensible as in an inguinal hernia.

144
Q

A 75 yoa woman presents to the emergency department with a 24 hour history of vomiting and cramping abdominal pain. She has not passed flatus or stool for 24 hours. She has no relevant previous medical history. On examination, she has a distended abdomen, and a tender, red coloured lump in the right groin. What is the most likely diagnosis and what should be done to help this patient?

A

The most likely diagnosis is a mechanical bowel obstruction secondary to a strangulated groin hernia, likely femoral due to the age, gender and strangulation.
This is a surgical emergency and the patient requires immediate work up for theatre:
ABCDE
IV access and consider fluid resuscitation if the patient is hypotensive
Catheterisation
FBC, UEC, group and hold, coags, lactate
NBM
Perioperative antibiotics
Analgesia
Contact general surg consultant/ registrar for consultation and definitive treatment
Surgical repair of the hernia by either a laprascopic or open approach.

145
Q

What are the causes of a lower GI bleed?

A

Lower GI bleeding refers to blood loss of recent onset originating from a site distal to the suspensory ligament of Treitz (connects to both the third an fourth parts of the duodenum, as well as the duodenojejunal flexure). The causes of lower GI bleeding may be grouped into several categories including:

  • Anatomical- diverticulosis
  • Vascular- angiodysplasia (small vascular malformation of the gut), haemorrhoids, ischaemia
  • Inflammatory- Inflammatory bowel disease, radiation colitis, other colitis (infectious, antibiotic associated, ischaemic, colitis of unclear etiology)
  • Neoplastic- polyp or cancer
  • Other- Anal fissures, rectal ulcers, postpolypectomy (removal of polyps)
146
Q

What are the signs and symptoms of a lower GI bleed?

A

Signs and symptoms include:
Haematochezia (passage of maroon or bright red blood or blood clots per rectum). In general anatomic and vascular causes of bleeding present with painless, large volume blood loss
Rarely melena in a right sided lower GI bleed.
Abdominal pain- may be caused by IBD, ischaemia and colitis
Rectal pain- anal fissures
Constipation/ diarrhoea- colitis and IBD
Weight loss- IBD, cancer
Anaemia- more common with occult blood loss and may lead to dyspnea, palpitations, fatigue, conjunctival pallor
Signs of haemodynamic instability in severe bleeds- tachypnoea, tachycardia, hypotension

147
Q

What is the management of a large LGI bleed?

A

Initially ABCDE should be performed with particular interest in circulation. Haemodynamic status should be initially assessed with intravascular volume resuscitation started as needed.
Upper GI bleed should be excluded with upper endoscopy
Colonoscopy should be the initial investigation and should be performed within 24 hours of patient presentation after adequate bowel preparation.
Endoscopic haemostasis therapy should be provided to all patients with high risk stigmata of bleeding including active bleeding, non bleeding visible vessels or adherent clot. Endoscopic haemostasis therapy involves mechanical, thermal, injection of adrenaline or a combination and is most often guided by the aetiology of bleeding.
Radiologic interventions (tagged red blood cell scintigraphy [technetium], CT angiogram and angiography) should be considered in high risk patients with ongoing bleeding who do not respond adequately to fluid resuscitation and who are unlikely to tolerate bowel preparation for colonoscopy.
In a patient who is acutely unstable surgical intervention with a laparotomy may be required and can also be used when other means of controlling an established source of bleeding have failed.
For an unidentified source of bleeding a subtotal colectomy (removal of the entire colon) and ileostomy may be necessary
Specific management may be necessary depending on the cause of the LGI bleed. Acute colitis can be managed with antibiotics if there is an infectious source or hydrocortisone if the source is IBD. 90% of diverticulosis bleeding will resolve spontaneously, however 10% will require a blood transfusion.

148
Q

What is a Meckels Diverticulum?

A

A Meckel’s diverticulum is a true diverticulum, meaning it contains all layers of the small bowel wall (mucosa, submucosa, muscular layer and serosa), found in the small bowel and particularly the middle to distal ileum.
It is the most common congenital abnormality of the GI tract, resulting from the incomplete obliteration of the vitelline duct during the fifth week of foetal development.
Patients are often asymptomatic and majority of symptomatic patients present before the age of 2. Signs and symptoms include GI bleeding, obstruction and Meckel’s diverticulitis (clinically indistinguishable from appendicitis).
The rule of twos is the classic description of the essential features:
Occurs in about 2% of the population
Male to female ratio of 1:2
Is about two feet (60cm) above the ileocaecal valve
Is about two inches long
2 types of heterotopic mucosa- gastric and pancreatic
Approximately 2% of the population develop a complication over their lifetime (typically before the age of 2)

149
Q

What symptoms can be caused by a Meckels diverticulum and why?

A

Meckel’s diverticulum are often clinically silent, however they can present as abdominal pain, symptoms of GI bleeding and bowel obstruction

GI bleeding:

  • Most common presentation in children
  • GI bleeding is caused by ulceration of the small bowel due to acid secretion by ectopic gastric mucosa within the diverticulum.
  • This usually occurs distal to the diverticulum such as in the perianal region.

Obstruction

  • Intussusception (a process in which a segment of intestine invaginate into the adjoining intestinal lumen)
  • Volvulus (the small bowel twists around its mesentery)
  • Torsion of the diverticulum
  • Littre hernia (protrusion of a Meckels diverticulum through the abdominal wall)
  • Meckel’s diverticulitis

Acute abdominal pain:

  • Diverticular inflammation
  • Bowel obstruction
  • Perforation of the bowel leading to peritonitis
  • Entrapment of enterolith (a mixed concretion formed in the GIT) causing obstruction, bacterial growth and inflammation (similar process to acute appendicitis)
150
Q

What are the causes of an obstruction of a luminal structure (e.g the bowel, the ureter, biliary tree, or blood vessel)?

A
Extramural:
Adhesions
Hernia
Volvulus
Intrabdominal abscess
Endometriosis
Prostatic hypertrophy
Tumours
Congenital bands
Intramural:
Congenital malformations
Neoplasms
Strictures
Intramural haematoma
Post op ileus

Atherosclerosis
Vasoconstriction
Vasospasm

Intraluminal:
Intussusception
Gallstones
Faeces
Foreign body
Parasites
Kidney stones

Thrombosis
Emboli

151
Q

What are the Signs and Symptoms of an Upper GI Bleed?

A

The main causes of an upper GI bleed are:
Peptic ulcer- epigastric or right upper quadrant pain
Severe or erosive gastritis/ duodenitis
Severe or erosive oesophagitis
Oesophageal varices- jaundice, fatigue, anorexia and abdominal distension
Angiodysplasia
Mallory-Weiss tears- emesis, retching or coughing
Tumours (benign or malignant)
In 10-15% of patients no cause can be identified

General signs and symptoms include:
Haematemesis- either fresh blood or “coffee ground” (a sign the blood has been in the stomach) blood
Melena
In severe cases signs of hemorrhagic shock/ haemodynamic instability- tachypnoea, tachycardia, hypotension, diaphoresis
Signs of anaemia

152
Q

What is the Treatment of an Upper GI Bleed?

A

Low risk patients should be admitted to the ward, made NBM, observed for continued bleeding or signs of haemodynamic instability and may receive an elective endoscopy with definitive management if necessary

High risk patients:
Initially resuscitation with a large calibre IV access and fluid resuscitation if the patient is tachycardic or hypotensive.

Call blood bank and give a blood transfusion if there is a large volume haematemesis or if there are still signs of haemodynamic instability after fluid resuscitation.

Bloods: FBC, UEC, LFT, coagulation and cross match
Alert HDU/ICU
Alert appropriate surgical team
Vitals monitoring
NG tube
Cease all NSAIDs and give IV PPI.

Gastroscopy (within 12 hours of first bleed) to determine the site of the bleeding and for management. If no obvious lesion is identified further imaging such as angiography/ small bowel capsular imaging/ enteroscopy (endoscopy of the small bowel). If a major bleed is identified on gastroscopy, therapeutic endoscopic interventions such as endoclipping, heater probe and injection therapy are available.

Surgery (laparotomy) if endoscopic treatment fails or for patients who suffer 2 or more re- bleeds.

153
Q

What do you do as the intern if you think someone is having an acute UGI bleed?

A

ABCDE
Assess for haemodynamic instability/ shock by checking vital signs
Give high flow oxygen
Call surgical registrar/ consultant
Call ICU
Venous access 2x large bore cannulas and fluid boluses x2
Blood transfusion if patient still shocked after fluid boluses
Catheter to monitor urine output
Considering call for rapid response if the patient is still unstable
NBM
Bloods- FBC, UEC, coags, LFTs, Group and hold
Correct electrolyte and clotting abnormalities

History;
How much bleeding and over what time period
Associated symptoms such as haematemesis, melena, dysphagia, pain
Use of NSAIDs (peptic ulcers)
Alcohol history including drinking just prior to bleeding and vomiting
Cirrhosis or liver disease (oesophageal varices)
Smoking

Examination:
Vitals
Cardio exam
Gastrointestinal exam
Looking for signs of liver disease
Differentials:
Peptic ulcer disease- (duodenal or gastric)
Reflux oesophagitis
Gastritis/ duodenitis
Oesophageal varices
Mallory-Weiss tears
Vascular malformations
Aortoenteric fistulas (post aortic surgery)
Neoplasm
154
Q

What are the risk factors contributing to the aetiology of a peptic ulcer?

A
Heliobacter pylori infection
NSAID use
Smoking
Increasing age
Personal history of peptic ulcer disease
Family history of peptic ulcer disease
Patient in intensive care
155
Q

What are the initial investigations in the ED for a perforated ulcer?

A
Bloods:
FBC- leukocytosis (infection)
CRP- inflammation or infection
UECs- kidney function and SIRS
LFTs
ECG- MI may present as epigastric pain
Serum gastrin levels- Zollinger-Ellison syndrome (gastric screwing tumours or hyperplasia of the islet cells in the pancreas causes overproduction of gastric acid, resulting in recurrent peptic ulcers)

Erect chest x-ray: 75% of PPU have free air under the diaphragm.
Abdominal X-ray
- Rigler’s sign- appearance of gas on both sides of the bowel/ double wall
- Football sign- a large volume of free gas resulting in a round black area
- Gas outlining soft tissue structures such as the liver edge or falciform ligament
Serum lipase/amylase- checking for the differential of acute pancreatitis

CT scan:
Diagnostic accuracy of 98%
Free air anteriorly just below the anterior abdominal wall
Visible falciform ligament when air is present on both sides
Exclude acute pancreatitis

156
Q

What are the physical signs suggestive of a perforated ulcer?

A

Physical signs are generally those suggestive of peritonitis due to the gastric contents spilling out into the abdomen:
- Involuntary guarding
- Rebound tenderness
- Board like rigidity
- Epigastric tenderness
Mild fever, pallor, tachycardia and hypotension
Absent bowel sounds

157
Q

What are the symptoms suggestive of a perforated peptic ulcer?

A
Sudden onset of severe epigastric pain that becomes generalised and is worse on movement. It may also radiate to the back and shoulders
Abdominal distension
Brownish bloodstained vomiting
Nausea
Constipation
Fever
158
Q

What is the treatment for a perforated duodenal ulcer?

A

Conservative therapy has been shown to be effective in 40-80% of perforated peptic ulcer disease:
Nasogastric tube
Intravenous fluids
Gastric acid suppression with a PPI
Antibiotics- triple antibiotic therapy, usually with ampicillin (gram positive), gentamicin (gram negative) and metronidazole (anaerobes)
Repeated clinical assessment

In patients who become clinically unstable or who are shown to have a leakage with gastrograffin, dye study surgery is indicated:

Laparotomy is the most common form of surgery and will involve an abdominal cavity washout with saline solution and closure of the perforation by suturing a vascularised flap of omentum over the defect.

Post surgery patients will have to undergo H.pylori eradication with triple therapy for 7 days:
PPI
Amoxicillin
Clarithromycin

Prompt resuscitation
Analgesia
Nasogastric tube
Urinary catheter
Omeprazole and triple therapy
Surgery
159
Q

How do you (as a surgical or ED intern) get a patient with a perforated ulcer ready for theatre?

A

Resuscitation ABCDE
Known relevant history and examination findings
NBM
Catheterisation
Contact the relevant people including the surgical team, anaesthetic team, operating theatre, blood bank
Medical management- analgesia, IV fluids, IV PPI, antiemetics, pre op antibiotics
Consent patient and give the booking slip to the NUM

160
Q
  1. You are the general surgery resident on call, you are asked to review a 70 year old man who is 72 hours post knee replacement on the orthopaedic ward. He has abdominal pain and distension, he has not passed stool or wind since the first day post surgery, he started vomiting this evening. He is usually well, with no chronic health problems found in his pre-surgery investigations. What are you going to do? What is the most likely diagnosis?
A

This patient likely has a pseudo-obstruction of the colon (Ogilvie’s syndrome).
Differentials include opiate induced pseudo-obstruction, paralytic ileus, toxic megacolon and bowel obstruction.
As the resident I would:
Start with a primary survey and adequate resuscitation replacing losses from vomiting and from lack of food
NBM
NG tube since the patient is vomiting
Decrease meds that decrease colonic motility and give analgesia with paracetamol/ ibuprofen and minimal opioids if necessary
History- op report, SOCRATES of pain
Exam- Abdominal and PR exam- bowel sounds can help distinguish between a functional and mechanical obstruction (bowel sounds are present in 90% of cases of Ogilvie’s syndrome)
Bloods- FBC, UEC, LFTs
Serial abdominal radiographs and consider abdominal CT
Neostigmine if there is greater than 12cm of dilatation of colonic distension and conservative management has failed for 48 hours. Neostigmine is a anti cholinesterase inhibitor to increases parasympathetic nervous system activity. However it can cause the side effects of bradycardia, hypotension, asystole, seizures and bronchoconstriction.
Colonic decompression if neostigmine fails or is contraindicated

161
Q

What are the common skin cancers?

A

The common skin cancers are:
Basal cell carcinomas
Squamous cell carcinomas
Melanomas

162
Q

What features of a mole on the skin would make you concerned that it had become malignant?

A

ABCDE of melanoma suspicion
Asymmetry- irregularity in shape
Border irregularity
Colour change- an increase in pigmentation, red halos around the lesion, multiple colour and even uneven distribution of colours are all important prognostic factors.
Diameter- > 6mm in diameter is often concerning
Evolution- increase in size, bleeding, ulceration or crusting, spreading of pigmentation, satellite lesion (smaller lesions near the edges of a principal lesion) or pain and itching are poor prognostic signs.

163
Q

What are the principles of initial treatment of a mole on the skin that you are concerned may have undergone malignant change?

A

Investigations:
Diagnostic biopsy- Excision biopsy is always preferred as it always gives you more information about invasion but shave and punch are done as they are more convenient. For an excision biopsy of a melanoma there should be 1mm margins.
Sentinel node biopsy- indicated for a melanoma with depth greater than 1mm
Imaging CT chest abdomen pelvis for staging and looking for metastatic spread. Note that there is a 98% chance of the CT not showing anything and a 2% chance of the CT showing metastasis, which have a very poor prognosis anyways.

Management:
Surgery- wide local excision of primary lesions with margins
- BCC and SCC- 1-2mm margin
- Carcinoma in situ (no spread past the basement membrane)- 5mm margins wide excision
- Invasive (below the epidermis) melanoma- wide excision with 1cm margin
Resection of metastases
Lymph node clearance if sentinel node biopsy is positive
Adjuvant treatment- radiotherapy and chemotherapy (relatively ineffective) or immunotherapy

164
Q

What is a volvulus of the colon?

A

A volvulus is a closed loop obstruction of the large bowel, often of the sigmoid or the caecum. It occurs when a large loop of bowel distended with gas and faeces, twists in on its mesentery, leading to a closed loop obstruction.
Presentation is more common in the elderly and there is an association with chronic neurological conditions such as Parkinson’s disease and MS and with antipsychotic medications.
This can rapidly lead to strangulation, infarction and then perforation and generalised peritonitis.
Classic clinical features include:
- Constipation
- Nausea and vomiting
- Abdominal distension and pain
- Tympanic abdomen
- Peritonitis if perforated
- Shock, tachycardia and severe pain if there is ischaemia
Abdominal radiographs will show a large, dilated loop of the colon, often with a few gas-fluid levels and commonly a collapsed rectum. Specific signs include coffee bean sign. CT may also be performed.

Rectal tube insertion is successful in treating 90% of cases.

The mortality rate is 20-25%, with the most common complication being bowel ischaemia.

165
Q

How is a sigmoid volvulus treated (after initial history, examination and investigation)?

A

The goal of treatment of sigmoid volvulus is to reduce the sigmoid volvulus and to prevent recurrent episodes
A flexible sigmoidoscopy should be performed first line to detours the volvulus, thereby restoring blood supply. An additional advantage is that it allows for an assessment of the viability of the colon.
Visualisation of a dilated proximal segment filled with gas and stool or a sudden expulsion of air and stool indicates successful reduction of the volvulus. Decompression should then be confirmed with AXR.
A rectal tube should also be left in place with the proximal end beyond the area of torsion as it lessens colonic distension and reduces the chance of recurrence.

The surgical approach for sigmoid volvulus includes sigmoidectomy and primary anastomosis. It is indicated in recurrent episodes of the sigmoid volvulus. Hartmann’s procedure (proctosigmoidectomy with closure of the anorectal stump and end colostomy) may also be used.

166
Q

This 18 year old man has this lump in his groin. What is the most likely diagnosis? What are the differential diagnoses?

A

The most likely diagnosis is an indirect inguinal hernia.
The differential diagnosis include:
- Direct inguinal hernia- bulging of abdominal contents through the posterior wall (transversalis fascia) of the inguinal canal.
- Femoral hernia- Abdominal contents migrate through the abdominal canal. Rarely has a cough impulse, rarely reducible and more likely to strangulate
- Inguinal lymphadenopathy- The inguinal nodes drain the lower limb, abdominal wall below the umbilicus, anal canal, scrotal skin and penis
- Saphena varix- dilatation of the long saphenous vein superficial to the deep facia before entering the femoral vein
- Femoral artery aneurysm- Dilatation of the common femoral artery. This is classically found in males >65yoa and is classically expansile and pulsatile
- Psoas abscess- Classically seen in TB
- Hydrocele
- Spermatocele
- Varicocele

167
Q

This 18 year old man has this lump in his groin. It is a hernia. What is most likely to be in the hernia?

A

Inguinal hernia:

  • Omentum
  • Small intestine
  • Peritoneum (if small)
  • Extra-peritoneal fat (if small)
  • Large intestine (less common)
  • Appendix (less common)

Femoral hernia:

  • Small bowel
  • Omentum
168
Q

What are the common methods of repairing an inguinal hernia in a fit 18 year old?

A

Laprascopic surgery- mesh repairs are required in laprascopic procedures and normally involve approaching the hernia from the posterior aspect.
Open surgery- a tension free mesh placed anterior to the hernial defect is used to plug and patch the hernia. A non-mesh open repair is also possible where a patch of external oblique aponeurosis may be used to patch the defect.

Indications for laprascopic approach:
Bilateral hernias
Recurrent hernias
Femoral hernia
Prevention of post repair neuralgia
Patients with physically active jobs or lifestyles as laprascopic repair reduces pain and has a quicker return to normal activity
Indications for open approach:
Prior pelvic surgery
Infection
Complicated hernia (i.e strangulated)
Large scrotal hernia (greater than 3cm)

The indications for repair are; for a symptomatic hernia, a femoral hernia, or a strangulated or incarcerated hernia
The contraindications for surgery are elective repair in pregnancy in asymptomatic hernias and in active infection.

169
Q

What are the common complications of an inguinal hernia repair, and what are the serious complications of this operation?

A
Common:
Seroma
Haematoma
Bruising
Superficial wound infection
Serious complications:
Urinary retention
Bladder injury
Persistent groin pain
Testicular complications such as pain, ischaemic orchitis, testicular atrophy, transection or obstruction of the vas deferens
Deep wound and mesh infections
Recurrent hernias
Mesh migration and erosion into bladder, spermatic chord or gastrointestinal structures
170
Q
  1. What is the most significant abnormality in this photo (the photo is a lump in the neck of a 70 yoa man). Where, anatomically, is it, what is the differential diagnosis?
A

The neck can be divided into the anterior and posterior triangles:
Posterior triangle- bordered by the trapezium muscle, sternocleidomastoid muscle and the clavicle. It can be further divided int the occipital triangle (superior to the posterior belly of the omohyoid muscle, and then subclavian triangle, inferior to the posterior border of the omohyoid muscle.
Anterior triangle- made up of the submental triangle, carotid triangle, submandibular triangle and the muscular triangle.

Differential diagnosis of neck lumps:
Inflammatory
Reactive viral lymphadenopathy
Bacterial lymphadenopathy
Parasitic lymphadenopathy

Neoplastic
Metastatic head and neck cancer
- Posterior triangle- nasopharyngeal cancer
- Upper jugular- oral cavity, oropharynx or laryngeal cancer
- Virchow’s node- tracheobronchial, oesophagus, stomach carcinoma
Thyroid masses
- Diffuse goitre- Grave’s disease and Hashimoto’s disease
- Toxic multinodular goitre
- Thyroid cancer
Salivary gland neoplasms
- 80% of neoplasms arise from the parotid gland with 80% of those tumours being benign (mostly pleomorphic adenomas)
- 50% of submandibular neoplasms are malignant
Paragangliomas- tumours in the peripheral nervous system. Occur in the superior part of the neck (just inferior to the ear) near the carotid artery.
Schwannomas- tumour of the Schwann cells that line the peripheral nerves
Lymphoma- common in children with Hodgkin’s lymphoma
Lipoma and benign skin cysts

Congenital

  • Brachial cleft cyst- epithelial cyst, arising on the lateral part of the neck from a failure of obliteration of the second branchial cleft in embryonic development
  • Thyroglossal duct cyst- an embryonic remnant that forms due to failure of closure of the thyroglossal duct extending from the foramen cecum in the tongue to the thyroid
  • Vascular anomalies
  • laryngocele- congenital anomalous air sac communicating with the larynx
  • Teratoma- tumour containing fully developed tissue such as bone, hair or muscle
  • Dermoid cyst- saclike growth containing structures such as hair, fluid, bone or sebaceous glands)

NB: A neck mass in an adult should be considered neoplastic, and potentially malignant, until proven otherwise. This likelihood is further increased if there is a smoking or alcohol history.

171
Q

What specific history and examination will you perform for a 70 yoa man who presents with a lump in the neck that is clearly not in the thyroid or other midline structures?

A

History:

  • Duration- masses present for years without change are likely benign neoplasms (benign salivary gland tumours, schwannomas or paragangliomas)
  • Growth pattern- rapid expanding masses are likely due to an infectious process or rapidly growing lymphoma
  • Absence or presence of pain
  • Voice changes, hoarseness, otalgia, dysphagia may indicate cervical node metastasis.
  • Other features- fever, night sweats, weight loss
  • Risk factors- smoking, alcohol, illicit drug use, occupational history and exposures, recent travel

Examination:
- Location
- Characteristics of mass- size, shape, consistency, tenderness, mobility and colour
• Neck masses that are due to reactive lymph nodes are usually discrete, mobile, firm, rubbery, but not rock hard
• Rock hard masses increase the chance of malignancy
• Soft, ballotable, mobile masses are usually congenital cysts
- Cranial nerve exam
- Abdo exam looking for masses and organomegaly
- Examination of the oropharynx and oral cavity

172
Q

This man has complained about a swelling at his umbilicus.

Q1. What can you see and what is the diagnosis?

Q2: What is the likely contents?

Q3. What is the risk of serious complications with this hernia as it stands?

Q4. What would be the correct approach if this were to become acutely painful and incarcerated?

A

I. Q1. What can you see and what is the diagnosis?
Most likely umbilical hernia
An umbilical hernia is a defect of the anterior abdominal wall fascia that occurs when the umbilical ring fails to close. The defect allows protrusion of a peritoneal sac that is covered by skin and may contain intrabdominal content, such as omentum and bowel.
Ddx epigastric hernia
II. Q2. What is the likely contents?
Mainly omentum and and pre peritoneal fat and also small bowel and colon
III. Q3. What is the risk of serious complications with this hernia as it stands.
Incarceration and strangulation and bowel obstruction and perforation
IV. Q4. What would be the correct approach if this were to become acutely painful and incarcerated?
ABCDE with adequate resuscitation and open surgical repair with mesh

173
Q

This is the laparoscopic view of the lower abdomen.
I. Q1. What is visible here?
II. Q2. How can the bowel be injured at Laparoscopy

A
  1. This is the laparoscopic view of the lower abdomen.
    I. Q1. What is visible here?
    II. Q2. How can the bowel be injured at Laparoscopy and what are the consequences?
    1/3 of bowel injury occurs during abdominal access. Other mechanisms include dissection, electrocoagulation (diathermy), tissue grasping, placing of pneumoperitoneum needles.
    Bowel injury is the 3rd leading cause of death, following anaesthesia and major vascular injury, after a laprascopic procedure. Apart from mortality, other consequences include peritonitis, bowel necrosis and perforation.
174
Q

What is biliary colic?

A

Biliary colic is when a colic (sudden pain) occurs due to a gallstone temporarily blocking the cystic duct. The pain is severe, located in the RUQ, commonly occur post-prandially, may radiate to the back or right shoulder blade and it typically rises to a plateau over a few minutes then continues unrelentingly. Note that the pain does not have the strikingly intermittent brief peaks of other forms of colic (e.g ureteric). These patients may be in agony until the pain resolves spontaneously after several hours or after opiate analgesia. Vomiting is often associated with the attack and the patient feels exhausted and sore for the next few days or so. There is commonly a history of similar episodes. There are few positive findings on examination; the patient is afebrile, but there may be some local tenderness due to gallbladder distension. If the attack lasts more than 24 hours acute cholecystitis is the more likely diagnosis.
Other differentials to consider include perforated peptic ulcer.

175
Q

What causes cholangitis?

A

Cholangitis is an infection of the biliary tree, most commonly caused by biliary obstruction, bile stasis and dilatation of the distal common bile duct. This allows for bacteria in the duodenum to enter the common bile duct. The most common infecting organisms are gram negative organisms such as E.coli, Klebsiella and enterobacter and gram positive organisms such as enterococcus.
Causes of the obstruction are biliary calculi, benign stenosis, malignancy and stent placement.

176
Q

What are the clinical features of cholangitis?

A

In its less severe form cholangitis will present with Charcot’s triad of intermittent right upper quadrant pain, swinging pyrexia and jaundice.
In the more severe, life threatening form known as toxic cholangitis patients will present with Reynolds’s pentad of Charcot’s triad + hypotension and confusion/ altered mental status.

Investigations include FBC (for WCC) inflammatory markers (CRP), LFTs (raised bilirubin)

The bile duct should be drained urgently, preferably by endoscopic sphincterotomy.

177
Q

What is cholecystitis?

A

Acute cholecystitis is acute gallbladder inflammation and is one of the major complications of cholelithiasis/ gallstones. It develops in up to 10% of patients with symptomatic gallstones. In most cases (90%) it is caused by complete cystic duct obstruction usually due to an impacted gallstone in the gallbladder neck or cystic duct, which leads to inflammation within the gallbladder wall. In 10% of cases bile inspissation (thickening), due to dehydration, or bile stasis, due to trauma or severe systemic illness, can block the cystic duct, causing an acalculous cholecystitis (cholecystitis without evidence of cystic duct obstruction).
Clinically it presents as prolonged (>4 hours) severe RUQ pain that may radiate to the right shoulder or back with fever, abdominal guarding, tachycardia, nausea and vomiting, anorexia and a positive Murphy’s sign. It is often associated with episodes of fatty foods approximately 1hr before onset.
Suspected with fever, Murphy’s sign or leucocytosis. Diagnosed by abdominal ultrasound
Management is with analgesia, IV fluids, NBM, broad spectrum antibiotics (not required in simple biliary colic) and a laprascopic cholecystectomy in the same admission.

NB: gallstones typically contain 20-80% cholesterol and calcium carbonate, phosphate, bilirubin and other bile pigments

178
Q

What techniques are available for treating common bile duct stones? Describe the technique briefly.

A

Choledocholithiasis is the presence of gallstones in the common bile duct. They are mostly asymptomatic, but can present with RUQ pain, nausea and vomiting. Stones may pass conservatively but often do not if they are larger than the size of the common bile duct. In this case there are two main techniques to remove them:
ERCP- Sphincterotomy of sphincter of Oddi to allow the passage of the stones. Papillary balloon dilatation has also been tried to preserve the sphincter however this technique has less supporting evidence. The ERCP should be followed by cholecystectomy.
Surgical- Largely only occurs if the stones are found intraoperatively during a laprascopic cholecystectomy. If found intraoperatively an intraoperative CBD exploration can be done or an intra-operative ERCP.

179
Q

What are the potential complications of an ERCP?

A

ERCP related complications can be divided into two main groups:
General complications common to all endoscopic procedures:
- Medication reactions and anaphylaxis
- Oxygen desaturation
- Cardiopulmonary accidents
- Haemorrhage or perforation caused by passage of the endoscope
Selective complications specific to pancreatobiliary instrumentation:
- Pancreatitis- caused by physical irritation of the pancreatic duct. Amylase should be tested 24 hours post-ERCP and levels greater than 3x normal are suggestive of acute pancreatitis. This occurs in approximately 3% of patients undergoing ERCP.
- Infection- cholangitis leading to subsequent septicaemia. The most common causative organisms are E.coli, Klebsiella, haemolytic streptococci, Pseudomonas, Enterococcus and Staphylococcus.
- Perforation of the pancreatic duct, bile duct or duodenum- reported in less than 1% of patients.
- Haemorrhage- postsphincterotomy bleeding has been reported in up to 2% of patients

180
Q

What are the symptoms of biliary colic? How can it be differentiated from acute cholecystitis clinically?

A

Biliary colic is usually caused by the gallbladder contracting in response to a fatty meal, pressing a stone against the gallbladder outlet or cystic duct. This then results in increased intra-gallbladder pressure and pain. The pain is felt in the RUQ, however, unlike acute cholecystitis the pain is visceral in origin, without gallbladder wall inflammation, so peritoneal signs are absent. In addition patients with biliary colic are afebrile with normal laboratory studies. As the gallbladder relaxes the stones often fall back from the cystic duct and as a result the pain reaches a crescendo over a number of hours and then resolves spontaneously, generally in less than 6 hours.
Cholecystitis has symptoms similar to biliary colic but will also present with fever, tachycardia, leucocytosis, Murphy’s sign and rebound tenderness and guarding of the RUQ.

181
Q

What are the common causes of painless obstructive jaundice in a 65 yoa person?

A

Jaundice is a yellow colour of the skin, mucous membranes or eyes. The yellow comes from bilirubin, a byproduct of red blood cell haemolysis. Obstructive jaundice is a specific type of jaundice where symptoms develop due to a narrowed or blocked bile duct or pancreatic duct, preventing the normal drainage of bile into the intestines. Painless obstructive jaundice may be caused by:

  • Pancreatic carcinoma near the head of the pancreas
  • Pancreatic cysts
  • Ampulla of vater carcinoma
  • Biliary stricture (often secondary to chronic to alcoholism)
  • Cholangiocarcinoma (cancer of the bile duct)
  • Autoimmune causes such as primary sclerosing cholangitis
  • Drugs
182
Q

How might a person with pancreatic cancer present to their GP?

A
Pancreatic cancer rarely causes symptoms. Symptoms often only present once the cancer is large enough to affect nearby organs or once it has metastasised. Signs and symptoms include:
-	Intractable upper abdominal pain which is described as:
•	Severe and continuous
•	Deep and gnawing
•	Often nocturnal and poorly relieved by analgesia
•	Partially relieved by sitting forward
•	Radiating to the back (60% of cases)
-	Weight loss- 80% of cases
-	obstructive jaundice- 50% of cases
-	Nausea and vomiting
-	Diarrhoea
-	Anorexia
-	Dark urine
-	RUQ mass or epigastric mass
-	Hepatomegaly
-	Cachexia
-	Gastric outlet obstruction
-	Steatorrhoea (the excretion of abnormal quantities of fat with the faeces)

Note that courvosier’s sign is that a palpable gall bladder in the setting of jaundice is unlikely to be due to gallstones.

183
Q

Why does a tumour of the head of pancreas cause jaundice? What are the Haematological consequence of biliary obstruction?

A

Pancreatic tumours present with jaundice due to compression of the common bile duct causing a cholestatic pattern.
Haematological consequences:
LFT derangement in an obstructive pattern- high alkaline phosphatase and high GGT as both of these substages are produced by the biliary epithelium (alkaline phosphatase also produced by the bones)
Elevated conjugated bilirubin

Liver failure causes haematological consequences including decreased clotting factors especially vitamin K clotting factors as the pancreas helps absorb vitamin K, decreased albumin, decreased platelets, protein C and S

184
Q

Why or how do gallstones cause pancreatitis?

A

Gallstones can cause pancreatic duct obstruction at the point of the ampulla of Vater. This causes reflux of pancreatic enzymes into the pancreas which can lead to inflammation and self digestion of the pancreas which will care interstitial oedematous pancreatitis.
Findings on liver function tests, particularly ALT levels greater than 150 units per litre, have a positive predictive value of 95% in diagnosing gallstone pancreatitis.

185
Q

What are the common causes of pancreatitis?

A

Idiopathic (10-20%)
Gallstones (50%) and genetic (cystic fibrosis)
Ethanol (30%)
Trauma
Steroids
Mumps/ malignancy
Autoimmune
Scorpion stings/ spider bites
Hyperlipidaemia/ hypercalcemia/ hyperparathyroidism
ERCP
Drugs (tetracyclines, furosemide, azathioprine, thiazides)

Note:

  • Clinical findings consistent with acute pancreatitis- acute onset of persistent, severe epigastric pain (pain may also be in the RUQ or more rarely confined to the left side), radiating to the back (in 50% of cases) and partially relieved by sitting up or bending forward, with associated nausea and vomiting (90% of cases), and occasionally dyspnoea
  • Laboratory findings- serum amylase or lipase (more commonly used in practice) greater than three times the upper limit of normal
  • Imaging (CT, MRI or ultrasound) criteria suggestive of acute pancreatitis
186
Q

How do you assess the severity of an episode of pancreatitis when the patient is first being admitted to the hospital?

A

There are various prognostic scores that have been developed to assess disease severity in acute pancreatitis. The Glasgow system is a simple prognostic system that uses age and 7 laboratory values collected during the first 48 hours following admission for pancreatitis. It is applicable to both biliary and alcoholic pancreatitis. A point is assigned for each breakpoint that is met at any time during the first 48 hours of admission. A score of greater than 3 is suggestive of severe pancreatitis.

PaO2 < 60
Age > 55
Neutrophils > 15
Calcium < 2
Renal function high serum urea
Enzymes- AST or ALT > 200 and LDH > 600
Albumin < 32
Sugar > 10
187
Q

What is the initial management of pancreatitis?

A

Fluid resuscitation with 2 large bore IV cannulas (with isotonic crystalloid solution such as Hartmann’s solution)!!!! (very important)
Oxygen supplementation
NBM/NG tube- oral intake can be restarted in mild pancreatitis once pain and nausea and vomiting have resolved.
Antiemetics- for example with ondansetron
Analgesia

Patients with biliary pancreatitis should have an ERCP within 24 hours of presentation.

188
Q

What are the potential late complications of severe, acute pancreatitis, late being 2 or 3 weeks after the start of the episode?

A

Pancreatic necrosis
Bile duct or pancreatic duct obstruction
Pseudocyst (fluid filled cavity not lined by epithelium) formation in the pancreas.
Diabetes mellitus
Chronic pancreatitis
Pseudoaneurysm- pancreatic enzymes in the blood causing breakdown of blood vessels. The most commonly affected vessels are the splenic and gastroduodenal arteries.
Enteric fistula formation
Lung complications- pleural effusion and pulmonary oedema

189
Q

What is this investigation/procedure? What does it show? Describe 3 complications of this intervention.

A
It is an ERCP.
It shows the common bile duct, biliary tree and gallbladder
Complications include:
-	Pancreatitis
-	Bleeding
-	Sepsis
-	Perforation

ERCP is indicated for therapeutic intervention, whereas MRCP should be used for diagnosis

ERCP with sphincterotomy and stone removal are valuable therapeutic interventions in choledocholithiasis with jaundice, dilated common bile duct, acute pancreatitis, cholangitis.
In patients with pancreatic or biliary cancer ERCP can also be used for palliation with plastic or mental stenting of the bile duct to relieve obstruction.

190
Q

What are the potential early complications of severe, acute pancreatitis, early being in the first 24 to 72 hours of the episode?

A

ARDS- pancreatic enzymes cause leakage of pulmonary capillaries
Shock
Peripancreatic fluid collection forming into a pseudocyst
Infection- necrotising pancreas or pancreatic abscess
Multiple organ failure
AKI
Abdominal compartment syndrome- increased intraabdominal pressure reducing blood flow to abdominal organs and leading to multi-organ failure
Respiratory compromise
Haemorrhage- adjacent inflamed stomach/duodenum, ruptured pseudocyst, peptic ulcer, pseudoaneurysm.

191
Q
  1. This is an ERCP on a patient who has had a bile duct injury at Laparoscopic Cholecystectomy.

Q1: What does this show?
Q2: What might this patient present with?
Q3: What are the features of Cholangitis?

A

Q1: What does this show?
A bile leak

Q2: What might this patient present with?
Fever, chills, nausea, vomiting, RUQ pain, distension, General discomfort, jaundice

Q3: What are the features of Cholangitis?
Charcot’s triad- Fever, jaundice and RUQ pain. It may also present with altered mental status and hypotension in more severe cases.

192
Q

This patient is having an operation about 1 week after an episode of acute Pancreatitis.

Q1. What does the patches of white on the fat represent? (examiner needs to point to the area on the right of the picture).
Q2. What is the mechanism of this?
Q3. What effect does this have on the serum Calcium during the actue phase?

A

Q1: What does the patches of white on the fat represent?
Fat necrosis/ saponification

Q2: What is the mechanism of this?
Enzyme damage to the membranes of the adipocytes near the affected pancreatic tissue.
Acute pancreatitis —> activation of lipase —> hydrolysis of peripancreatic fat —> release of free fatty acids —> FFA precipitate with circulating calcium (saponification)

Q3: Leads to hypocalcaemia due to the sequestration of Calcium.

193
Q

This patient presents with painless jaundice, dark urine and itch.

Q1: What organ is the surgeon palpating for in this photo?

Q2: What is the significance of being able to palpate this organ which is not normally palpable?

Q3: Explain why this is so

Q4: What is the name of this sign.

A

Q1: Gallbladder

Q2: This means that the gallbladder is abnormally large. This is Courvoisier’s sign (in the presence of an enlarged gallbladder which is non-tender and accompanied with mild painless jaundice, the cause is unlikely to be gallstones). The enlarged gallbladder is likely to be from neoplasm of the head of the pancreas or cholangiocarcinoma until proven otherwise.

Q3: Pressure in the gallbladder increases from obstruction of the biliary tree which causes a build up of bile and inflammation of the gallbladder.

Q4: Courvoisier’s sign

194
Q
  1. This woman presented with some upper abdominal fullness and early satiety.
    Q1. What is the lesion and what modalities have been used for its investigation?

Q2. What kind of Hepatic cysts do you know? This lesion is much bigger than most hepatic cysts. What complications do you know of large hepatic cysts?

A

Cystic lesion, CT and ultrasound

Hepatic cysts can be broken into simple and complex cysts

  • Simple- congenital cysts, biliary hamartomas, Caroli disease and polycystic liver disease
  • Complex cysts- mucinous cystic neoplasms, hydatid cysts, haemorrhagic cysts, endometrial cysts and cystic hepatocellular carcinoma

Complications of large hepatic cysts include:
Haemorrhage, rupture, infection, biliary obstruction, secondary cholangitis if cysts rupture into the biliary tree, anaphylaxis if a hydatid cyst ruptures, malignant transformation of benign cysts and compression of the venous system in the liver.

195
Q
  1. This Xray shows the right upper Quadrant of a patient who has had cancer of the pancreas.

Q1. What does it show? This patient initially presented with obstructive jaundice and had an endoscopic procedure to relieve that.

Q2. What was the procedure and which stent relates to that? A second procedure was needed 3 months later because of repeated vomiting.

Q3. What was that and outline the stent?

A

Q1
Stent in the biliary tree

Q2
ERCP with stenting
Metal or plastic:
The placement of a plastic stent is inexpensive and effective and the stent can easily be removed or exchanged. Plastic stents however eventually develop occlusion by sludge and/or bacterial biofilm and maintaining biliary drainage with plastic stents usually requires repeated ERCP.
Metal stents extend the duration of stent patency. However, metal stents have significantly higher costs and may not be removable

Q3
Enteral stenting and surgical bypass to solve a gastric outlet obstruction which obstructs gastric emptying at the pyloric sphincter. The procedure involves passing a guide wire through the obstruction and deploying a stent to keep the obstruction open. The stents are self expanding metal stents and can be covered or uncovered depending on whether they are coated. The advantage of covered stents is that they prevent tumour ingrowth.

196
Q
  1. This 40 year old female underwent laparoscopic cholecystectomy. During the operation this Xray was taken.
    Q1. What is this Xray?

Q2. What does it show?

A

Q1
Intra-operative cholangiogram

Q2
A cholangiogram shows the gallbladder, cystic duct, the left and right hepatic ducts, the common bile duct and the ampulla of vater.
Stones of the common bile duct

197
Q

This is an endoscopic photograph taken of the duodenum (second part) from and endoscope.
Q1.What is the structure you can see?

Q2. These 2 pictures show a procedure that is performed on the papilla usually as a prelude to removing stones, what is it called?

A

Q1
Sphincterectomy

Q2
Short term:
-	Bleeding
-	Infection
-	Pain
-	Pancreatitis
-	Perforation of duodenal wall
-	Abscess or fistula formation

Long term:

  • Stone recurrence
  • papillary stenosis
  • Cholangitis
198
Q

This picture shows a metallic biliary stent which is blocked by accumulated biofilm and debris (duodenal view).
Q1. What are the consequences for this patient of this situation?
Q2. How might they present?
Q3. What is the appropriate treatment?

A

Q1
This is the main cause for primary biliary stent failure and the occlusion reduces the quality of life of these patients and increases the risk of complications as there is a new blockage.

Q2
Jaundice, RUQ pain and nausea
Elevated liver enzymes in a cholestatic pattern (raised GGT and ALP)

Q3
Mechanical Cleaning With a Baloon
Plastic stent insertion
Percutaneous transhepatic biliary drainage if endoscopic attempts fail

NB- biliary stenting is recommended in patients with a potentially resectable malignant biliary obstruction, in those who require palliative care drainage of the common bile duct or hilar strictures, in treatment of strictures related to chronic pancreatitis, liver transplantation or cholecystectomy and in leaks and failed biliary stone extraction.

199
Q

This series of photographs shows a duodenal papilla before during and after an ERCP and endoscopic sphincterotomy.

Q1. What are the usual indications for this procedure?

Q2. What are the common complications of this procedure?

Q3. What preventative therapy do you know of that can reduce the risk of some of these complications?

A

Q1
Symptomatic common bile duct stones
Treatment of papillary stenosis
Facilitation of endotherapy such as stenting, stricture dilatation, tissue sampling

Q2
Short term
-	Haemorrhage
-	Acue pancreatitis
-	Perforation of duodenal wall
Long term:
-	Papillary stenosis
-	Ascending cholangitis
-	Stone recurrence

Q3
Lap cholecystectomy to prevent stone recurrence
Rectal NSAIDs to prevent acute pancreatitis (rarely used)
Coags preoperatively and cease anticoagulation therapy
Prophylactic antibiotics to prevent ascending cholangitis

200
Q

What are the differences in pathology between UC and Crohns?

A

In ulcerative colitis there is mucosal inflammation with continuous involvement of the affected part of the colon, there is always rectal involvement, there may be backwash ileitis, there is widespread irregular ulceration and there is no granuloma formation and minimal fibrosis of the bowel wall.

In Crohns disease there is transmural inflammation of the bowel with skip lesions and fissured ulcerations causing cobblestoning. Any part of the GIT may be affected with the most common areas being the ileum (affected in 80% of cases and exclusively affected in 50% of cases) and proximal colon. There is also non- caseating granuloma formation and marked fibrosis of the bowel wall.

201
Q

You are the general surgery resident on call, you are asked to review a 35 year old man who is 36 hours after an open right hemicolectomy for a Crohns stricture.

The nursing staff have measured his oxygen saturations as 87% on 2L oxygen delivered by nasal prongs, and his respiratory rate is 30. What is the significance of this reading?

A

This indicates that the patients oxygen saturation is significantly low despite nasal prong oxygen. Additionally there increased respiratory rate indicates further increase in oxygen requirements. This patient is hypoxemic, which could lead to inadequate perfusion.

This is likely due to type 1 respiratory failure (if the CO2 is low or normal) which can be caused by acute asthma, pulmonary oedema, ARDS, pneumonia, pneumothorax, fibrosing alveolitis or PE.

202
Q

You are the general surgery resident on call, you are asked to review a 35 year old man who is 36 hours after an open right hemicolectomy for a Crohns stricture.

The nursing staff have measured his oxygen saturations as 87% on 2L oxygen delivered by nasal prongs, and his respiratory rate is 30. What are the common causes of this situation?

A
This is likely due to type 1 respiratory failure (if the CO2 is low or normal) which can be caused by:
Pneumonia
PE
Atelectasis
Pleural effusion
Acute upper airway obstruction
Pulmonary oedema
ARDS
Pneumothorax
Fibrosing alveolitis
Acute asthma
203
Q

What are the main types of bariatric operations that are performed for obesity? How do they work?

A

Criteria for bariatric surgery
BMI over 35 and at least one major comorbidity such as type II diabetes, hypertension or sleep apnoea
BMI over 40

There are 3 main types:

  1. Gastric Banding:
    Adjustable silicon band placed just below the gastrooesophageal junction applying gentle pressure that suppresses hunger. Level of restriction can be adjusted by varying the amount of fluid in the band.

Expected Weight Loss: (17-20%)

Advantages:

  • Effective with good long term weight maintenance.
  • Degree of restriction adjustable.
  • Reversible.
  • Lowest morbidity and mortality rate

Disadvantages:

  • Highest long term re-operation rate
  • Gastric pouch dilatation
  • Erosion of the band into the stomach
  • Weight regain
  • Leaks to the adjustable gastric band
  1. Sleeve Gastrectomy
    Greater portion of the fundus and body of the stomach is removed. Gastric volume is reduced by 80%.

Expected Weight Loss: 20-30%

Advantages:
- Very effective with good mid-term weight maintenance

Disadvantages:

  • Staple line leak
  • GORD
  • Dilatation of the gastric remnant
  • Weight regain
  • Limited long term data
  • Moderate nutritional deficiencies due to malabsorption
  1. Roux-en Y gastric bypass
    - Small stomach pouch created, thereby reducing gastric volume.
    - The pouch is joined to the jejunum, hence, diverting nutrients from the lower stomach, duodenum and proximal jejunum.

Advantages:

  • Largest amount of weight loss with good long term weight maintenance
  • Highest rates of diabetes remission (for patients with pre-existing type II diabetes mellitus)

Disadvantages:

  • Staple line leak
  • Dumping
  • Stomach ulcer
  • Intestinal obstruction
  • Gallstones
  • Nutritional deficiencies due to malabsorption
  • Altered alcohol metabolism
  • Right regain
204
Q

You are asked to organise the staging investigations to help decide the treatment plan for a 65 yoa man who has been found to have an oesophageal cancer (adenocarcinoma) when he had an UGI endoscopy performed to investigate dysphagia.
Biopsies taken at that investigation have confirmed that what looked like a cancer, is a cancer.
What tests are needed?
What are they aimed at deciding?

A

Overall the aim is to accurately characterise:
Extent of the primary tumour
Regional lymph node involvement
Presence of distant metastasis
This TNM staging of the disease is important for deciding if the tumour is resectable, if neoadjuvant therapy is required or if palliation is indicated

Note that there are often metastases to the lungs.

Investigations:

  1. Endoscopic Ultrasound Scan (USS)
    Evaluation of choice for the primary tumour and its relationship to the oesophageal wall layers, having an 80-90% accuracy for T and N staging
  2. CT Chest/abdomen
    Evaluate local lymph node and distant metastatic disease and also helps to evaluate the primary tumour, although there is limited value in this
  3. PET Scan
    Detects distant metastatic disease
  4. Diagnostic Laparoscopy
    Controversial however it does detect intraperitoneal metastases that are heard to diagnose non-invasively
  5. Pre-op bronchoscopy with biopsy and brush cytology
    Indicated for patients with locally advanced tumours that are non-metastatic and above the level of the carina
205
Q

You are asked to organise the pre operative assessment to help decide whether a 65 yoa man is fit for surgery for a locally confined oesophageal cancer (adenocarcinoma). He presented with dysphagia.
His cancer has been proven, and staging complete. The best chance of cure is surgery. What issues are most important in this patient’s pre op work up?

A

Oesophagectomy is the only curative treatment for oesophageal cancer, but it has a high associated morbidity and mortality. Factors to consider are:
His age (65)
Pulmonary function and smoking status- spirometry and blood gases
Comorbid illnesses- especially lung disease or cirrhosis (LFTs)
Cardiac fitness- ECG, echo, stress test
Nutritional status
Previous surgeries and the response to anaesthetic
Consider a trimodal approach with chemoradiotherapy and cancer
Whether or not the patient consents
Alcohol and weight
His kidney function- UECs, coagulations

206
Q
  1. You are a GP. A previously fit 65 yoa woman consults you and gives a history of 3 months of progressive dysphagia, initially she struggled to swallow large food boluses, eg bread or meat, now she can only swallow fluids. She is losing weight. What would you like to do to manage this lady?
A

The main concern in this lady is oesophageal carcinoma (age > 50, progressive dysphagia and weight loss).

History:
Problems initiating swallowing or is food getting stuck at a certain point (oropharyngeal dysphagia is difficulty initiating swallowing, oesophageal is food getting stuck)
Coughing, choking or nasal regurgitation (oropharyngeal cancer)
Do you have problems swallowing solids, liquid or both (liquids and not solids suggests a motility disorder)
Have the symptoms progressed or remained the same? and if they have progressed then over what time period?
Associated symptoms- appetite changes, nausea, vomiting, heartburn, vomiting blood, pain on swallowing
Other medical conditions
Previous surgery on the larynx
Radiation therapy in the past?
Medications?

Examination:
Wasting
Hepatomegaly- tumour unresectable
Virchow’s node (supraclavicular node) lymphadenopathy- tumour unresectable
Hoarseness

Investigations:
Gastroscopy and biopsy
Staging- oesophageal ultrasound, CT and PET scan

Management:
Neoadjuvant chemoradiotherapy prior to surgery if there is a chance of cure
Oesophagectomy after chemoradiotherapy for definitive treatment. Type of oesophagectomy is dependent on whether the cancer is below or above the level of the carina.
Palliative care- laser treatment, stent insertion or argon tissue coagulation for oesophageal patency
squamous cell carcinomas- high likelihood of responding to chemoradiotherapy- more proximal
Adenocarcinomas- oesophagectomy

Differentials:
Food impaction- most common cause of acute onset dysphagia
Oesophageal strictures
Oesophagitis
Radiation therapy
Achalasia
Spastic motility dosorders
Scleroderma
Sjogren’s syndrome
Functional motility disorder
207
Q

You are a GP. An otherwise fit 30 year old man consults you. He has persistent gastrooesophageal reflux disease, he is well maintained on 40mg omeprazole per day, but gets bad symptoms very quickly if he stops taking the treatment. He is not keen on a lifetime of medication. He wants to know if there are any other treatments, and what the risks are of them

A

There are three main categories of management for GORD; conservative, medical and surgical

Conservative:
Weight loss
Moderate exercise
If GORD is nocturnal elevation of the bed head and avoidance of food 2-3 hours before bed
Smaller meals
Smoking and alcohol cessation
Avoid anticholinergics

Medical:
PPI single dose once daily for 4-8 weeks and then reduction of dosage to the lowest possible dosage that adequately controls symptoms. If standard therapy is not adequate to control symptoms high dose PPI therapy may be used, which involves taking double the dosage of standard therapy, preferably over 2 doses 12 hours apart.

Surgical:
Laprascopic Nissen fundoplication
Constructs a 360˚ cuff of gastric tissue around the lower oesophageal sphincter. Stomach contraction will therefore contract the lower oesophageal sphincter preventing reflux of gastric contents.
Indications for surgery include:
-	GORD with refractory symptoms despite maximal medical management
-	Intolerance of treatment
-	Symptomatic complications unresponsive to medical therapy such as hoarseness, laryngitis, wheezing, nocturnal asthma, aspiration pneumonia or dental erosion
-	Benign stricture
-	Noncompliance with medical therapy
-	High volume reflux
Complications include:
-	Post op IBS for approximately 2 weeks
-	Inadequate control of GORD
-	Dysphagia
-	Increased flatus
-	Delayed gastric emptying
-	Dumping syndrome- when food, especially sugar, moves into the small intestine too quickly
-	Gas bloat syndrome
-	Achalasia
-	Vagus nerve injury
-	Mortality in less than 1% of patients
208
Q

This patient with dyspepsia is having a Gastroscopy. What does this show in the duodenum?
How might this be complicated?
What is the common cause of this?
How is this condition treated?

A

Duodenal ulcer
Complications: perforation for anterior ulcers, erosion into gastroduodenal artery in posterior ulcers causing bleeding
Common causes- H.Pylori, NSAIDs, smoking
Management:
Mucosal patch for perforation (laprascopic intervention)
Cytoprotective agents- misoprostol
Eradication therapy for H.pylori- triple therapy (PPI + clarithromycin + amoxicillin)

209
Q
  1. Achalasia.
    i. Q1: What is the name of this investigation?
    ii. Q2: What does this test show?
    iii. Q3: What investigation should come next?
A

Achalasia results from progressive degeneration of ganglion cells in the myenteric plexus in the oesophageal wall, leading to failure of relaxation of the lower oesophageal sphincter, accompanied by a loss of peristalsis in the distal oesophagus.
It commonly affects people aged 30-50 and is associated with stress, drastic weight loss, slowly progressive (months to years) dysphagia of both solids and liquids and regurgitation (recognisable food).
Investigations include barium swallow which may show:
- Bird beak appearance
- Oesophageal dilatation
- Tram track sign- central longitudinal lucency bounded by barium on both sides
- Incomplete lower oesophageal sphincter relaxation that is not coordinated with oesophageal contraction
- Failure of normal peristalsis to to clear the oesophagus of barium
and then manometry to confirm the high pressure and the non relaxing lower oesophageal sphincter. An endoscopy may also be performed to ensure the symptoms are not caused by a neoplasm.
Management is with GTN, calcium channel blockers, botox, or endoscopic balloon dilatation. Surgical management is with oesophagomyotomy (cut outer longitudinal layer of oesophagus from above the sphincter to the cardia of the stomach) or oesophagectomy (removal of oesophagus).

210
Q

This is a laparoscopic view of the upper abdomen.

i. Q1. What organs can you see?
ii. Q2. What is the normal laparoscopic insufflation pressure and why?

A

Normal laprascopic insufflation pressure is less than 15mmHg. This is to allow for adequate surgical exposure without causing pneumoperitoneum and vasoconstriction leading to reduced organ perfusion. The normal intrabdominal pressure is 0-5mmHg and increases above 10mmHg are clinically significant, with > 15mmHg causing abdominal compartment syndrome.

211
Q

This is the endoscopic photo of a patient with a small sliding Hiatus Hernia. The photo is taken from just above the Gastro-oesophageal junction.
Q1: What does this show.
Q2: What is the severity of the reflux in this case?
Q3: How should this be treated?
Q4: If this is a patient who has failed to respond to medical therapy, what is the next option?

A

Q1
The lower oesophageal sphincter is higher than usual, and some of the stomach has been pulled up into the oesophagus. There may be some oesophagitis or Barrett’s oesophagus.

Q2
Barrett’s oesophagus = severe reflux

Q3
Most sliding hernias can be managed conservatively, with surgery reserved for intractable cases:

Reducing reflux- weight reduction, alcohol reduction, smoking cessation, avoiding trigger foods, not exercising after eating, not eating 3 hours before bed.
Reducing acid production- PPI therapy for 4 weeks. If this works the dose should be gradually reduced until the patient is on the lowest maintenance dose possible. If single dosing is insufficient double dosing by adding a second dose 12 hours apart is another option.

Q4
Surgery:
Laprascopic reduction of hiatus hernia
360˚ fundoplication

212
Q
  1. This is the endoscopic picture of a 65 year old male with a history of chronic dyspepsia. The bottom 2 photos show an area of slough in the first part of the duodenum posteriorly.
    Q1. What are the likely causes of this?
    Q2. What complications can occur with this pathology?
    Q3. What is the treatment?
A

Q1.
The most likely cause of this is duodenal ulcer from H.pylori
Q2.
The two main complications are bleeding and perforation. If the ulcer is posterior it will more likely cause haemorrhage as the gastro-duodenal artery will be effected. If the ulcer is anterior it is more likely to perforate.
Q3.
The treatment is triple therapy- PPI + amoxicillin + clarithromycin

213
Q

A 52 year old man presents to his GP with 2 months of difficulty swallowing.

Q1. What questions are important to ask?

Q2. What is the most important diagnosis to exclude?

Q3. How should he be investigaed?

A

Q1
Progression of dysphagia
Is it associated with liquids and solids- association with liquids is more likely to be achalasia
Is it associated with coughing or nasal regurgitation (more likely oropharyngeal cancer)
Known neuromuscular diseases such as myasthenia gravis?
Family history of oesophageal cancer
Previous radiotherapy or surgery on the anterior neck
Red flag symptoms- weight loss, night sweats, painful swallowing, otalgia, haematemesis etc

Q2
The most important differential to exclude is oesophageal cancer

Q3
Investigations depend on the history and may include:
Endoscopic ultrasound with biopsy, barium swallow (achalasia), oesophageal manometry, PET scan

214
Q

This series of photographs shows a Gastric Tumour that is very close to the gastro-oesophageal junction at endoscopy.

Q1. What types of gastric tumours do you know?

Q2. How would you investigate this patient if the biopsy of this lesion proved to be an adenocarcinoma?

Q3. What treatment would be required for a lesion in this location?

A

Q1
Adenocarcinomas are by far the most common type of gastric tumour (90-95%)
Lymphoma- cancers of the gut associated lymphoid tissue (GALT)
Gastrointestinal stromal tumours
Neuroendocrine tumours
Squamous cell cancer
Leiomyosarcoma- sarcoma developed from smooth muscle cells

Q2
Staging of the tumour:
- CT scan of chest and abdomen- determine tumour invasion, lymph node involvement and hepatic and more distant metastasis.
- Endoscopic ultrasonography via a flexible endoscope for determining the depth of penetration and therefore the T stage of the cancer as well as local spread int lymph nodes, the liver and the pancreas
- PET scan- for metastatic disease

Q3
Preoperative chemotherapy
Total gastrectomy with a Roux-en-Y reconstruction or a proximal subtotal gastrectomy
Post operative radiotherapy for a proximal subtotal gastrectomy

215
Q

This photo taken at Gastroscopy shows the area of the lower oesophagus just above the gastro-oesophageal junction.

Q1. What can you see?
Q2. What is the most likely cause of this process?
Q3. How would this be treated?

A

Q1
Reflux oesophagitis?- damage to the cells that line the lower portion of the oesophagus (squamous epithelium)

Q2
GORD or hiatus hernia

Q3
PPI and conservative management

216
Q
  1. This CT scan shows an abnormally positioned Kidney.
    Q1. Where is the Kidney located?
    Q2. What are the consequences of this abnormality?
    Q3. How might the significance of this vary between men and women?
A

Q1
This is likely an ectopic kidney and the kidney is likely located in the pelvis, however ectopic kidneys may be located in the abdomen, lumbar region or pelvis.

Q2
Ectopic kidneys are generally not harmful, however can lead to:
- increased risk of calculus formation with consequent hydronephrosis.
- Recurrent UTI as normal drainage can be affected
- Abdominal pain
- Palpable abdominal masses
- Hydronephrosis from obstruction
- Trauma
- Vesico-ureteric reflux

Q3
If the kidney is located in the pelvis there could be complications during pregnancy with the expanding uterus and thus obstruction of the outflow of the kidney.

217
Q

What are Varicose veins?

Q1. What two significant venous abnormalities are visible on this picture? []

Q2. How should this patient be investigated?

Q3. What are the complications if the venous hypertension goes untreated?

Q4. What is the mainstay of treatment for this condition?

A

Varicose veins are dilated superficial veins, usually in the legs, measuring greater than 3mm. It is caused by valvular insufficiency resulting in venous hypertension, reflux and dilatation of veins. There is a strong genetic component to varicose veins.

Q1 []

Q2
Venous duplex ultrasound, documenting:
Patency and competency of the deep system
Patency and competency of the superficial system, including the incompetent segments, site of junctions and perforators
Location and relationship of the sapheno-popliteal junction
Diameter of the affected veins

Q3
Pain
Oedema
Skin pigmentation- commonly seen in Gaiter area of the leg and associated with increased melanin production and deposition of haemosiderin.
Champagne bottle appearance of the leg
Eczema
Atrophie blanche- white scarring
Venous ulceration- usually located on the medial aspect of the leg between the ankle and calf (Gaiter area)
Superficial thrombophlebitis- blood clotting of the vein that causes swelling and pain
Haemorrhage

Q4
While surgery, with great saphenous vein stripping, remains the gold standard most veins can be treated with a range of other endovenous therapy options including:
- Sclerotherapy- using injected liquid or foam to cause venospasm and sclerosis of veins
- Endogenous laser therapy- now covered by medicare
- Radio frequency ablation- now covered by medicare
- Mechanical therapy- using steam or rotating catheter

218
Q
  1. You are the ED intern on duty. You are asked to see a 20 year old who was struck on the left side of his head by a cricket ball. Initially he was drowsy and a bit confused, then he appeared to recover completely. He is now mildly confused, he has a severe headache and has vomited 4 times. His pulse is 60 bpm and regular, and his BP is 170/110mmHg. During the examination his GCS drops rapidly to 8/15.

What is your differential diagnosis and what investigations would you like?

A

The most likely diagnosis is an epidural/ extradural haematoma which is a collection of blood that forms between the inner surface of the skull and the dura mater. They are usually associated with a history of head trauma and the source of bleeding is usually arterial, most often a torn middle meningeal artery. The classical clinical presentation is of a patient involved in a head strike who may or may not lose consciousness transiently. Following the injury they regain a normal level of functioning but usually have an ongoing but severe headache. Over the next few hours the gradually lose consciousness.

Investigations to order include a:
CT scan- extradural haematomas are almost always seen on CT and are typically biconvex (lentiform) in shape, hyperdense and most frequently beneath the temporal bone. There may also be secondary features of mass effect including midline shift, subfalcine herniation and uncle herniation.
MRI- may be ordered if a CT is not conclusive

Other useful investigations include:
Fundoscopy- papilledema indicative of raised ICP
IV access and bloods- FBC, UEC, ABG, BSL, LFT, coagulation study
ECG
Urinalysis

219
Q

You are the ED intern on duty. You are asked to see a 20 year old who was struck on the left side of his head by a cricket ball. Initially he was drowsy and a bit confused, then he appeared to recover completely. He is now mildly confused, he has a severe headache and has vomited 4 times. His pulse is 60 bpm and regular, and his BP is 170/110mmHg. During the examination his GCS drops rapidly to 8/15.

What is your differential diagnosis and what has happened to cause this problem?

A

The most likely diagnosis is an extradural/ epidural haemorrhage.
An extradural haematoma occurs when blood accumulates in the space between the skull and dura mater. It is almost always associated with a skull fracture, typically in the temporal bone, and rupture of the middle meningeal artery. The expansion of the haematoma strips the dura mater from the cranium causing the headache. Further haematoma expansion causes a raised intracranial pressure which decreases cerebral perfusion and (as cerebral perfusion pressure is increased above blood pressure) leading to a decline in consciousness and eventually death.

220
Q

A 56 year old woman is brought to the ED by her partner. She had complained of a sudden onset, severe headache immediately before collapsing, unconscious on the floor. She has regained consciousness after a few minutes, and now complains of neck stiffness. PMH is completely unremarkable, other than being a long term smoker.

What is the likely diagnosis and what are the causes?

A

The most likely diagnosis is a subarachnoid haemorrhage.
The major cause of this (in 80% of cases) is a rupture of an intracranial saccular aneurysm, most commonly occurring in the communicating arteries. Factors contributing to the formation of a saccular aneurysm are:
Smoking
Alcohol abuse
Hypertension
Connective tissue disorders (Marfans, Ehlers Danos)
Sickle cell disease
Congenital arteriovenous malformations
Anticoagulants
Sympathomimetic drugs (cocaine, caffeine, amphetamines)
Family history
Diabetes mellitus

221
Q

A 56 year old woman is brought to the ED by her partner. She had complained of a sudden onset, severe headache immediately before collapsing, unconscious on the floor. She has regained consciousness after a few minutes, and now complains of neck stiffness. PMH is completely unremarkable, other than being a long term smoker.

What is the likely diagnosis, and how would the patient’s neurological condition best be monitored clinically during initial assessment?

A

The most likely diagnosis is a subarachnoid haemorrhage.
During initial assessment this patient should receive clinical monitoring:
Regular monitoring of their vital signs
Airway evaluation
GCS
Full neurological examination
Early referral to ICU

222
Q

A 56 year old woman is brought to the ED by her partner. She had complained of a sudden onset, severe headache immediately before collapsing, unconscious on the floor. She has regained consciousness after a few minutes, and now complains of neck stiffness. PMH is completely unremarkable, other than being a long term smoker.

What is the likely diagnosis and how would the diagnosis be confirmed?

A

The most likely diagnosis is a subarachnoid haemorrhage.
The main investigation used to diagnose SAH is CT brain.

History:

  • Presence of risk factors- smoking, alcohol, hypertension, pregnancy, anticoagulation, known aneurysm
  • Thunderclap headache
  • Nausea and vomiting
  • Photophobia
  • Loss of consciousness

Examination:

  • Neck stiffness
  • Altered mental status
  • CN abnormalities in cranial nerves 3 and 6

Investigations:

  • Bloods- FBC, UEC, troponin, coagulation studies
  • ECG- non specific ECG patterns including bradycardia are common with SAH
  • Lumbar puncture- elevated opening pressure, elevated RBC count and xanthochromia due to lysed RBCs
  • CT- blood in the subarachnoid space, most commonly around the circle of Willis
  • MRI- more sensitive to SAH
  • Digital subtraction catheter- gold standard for the diagnosis of an aneurysm post diagnosis
223
Q

A 56 year old woman is brought to the ED by her partner. She had complained of a sudden onset, severe headache immediately before collapsing, unconscious on the floor. She has regained consciousness after a few minutes, and now complains of neck stiffness. PMH is completely unremarkable, other than being a long term smoker.

What is the likely diagnosis, and what is the usual treatment if this diagnosis is confirmed?

A

The likely diagnosis is a subarachnoid haemorrhage.
Primary survey
Correct electrolyte and coagulation (FFP) imbalances
Manage hypertension- nimodipine (dihydropyridine CCB that quickly reduces blood pressure)
Discontinuation of antithrombotics
Prophylactic anti-emetics and stool softeners
Analgesia
Normothermia

Specific:
Surgery and coil embolisation (most common management)
Prevent delayed cerebral ischaemia- nimodipine to prevent vasospasm
Seizure prophylaxis
Manage raised ICP- osmotic therapy and diuresis (mannitol)

224
Q

You are the ED intern on duty. You are asked to see a 75 yoa man who has been brought into the hospital from home where he has been becoming increasingly confused and clumsy. He was previously well and independent. He had a fall about 3 weeks ago, when he cut his forehead. He has little medical history of note, he used to smoke and still drinks about 30 standard units of alcohol per week. On direct questioning he is confused in time and place. Physical and neurological examination are normal.
What is your differential diagnosis? What investigations would you like?

A

The most likely diagnosis is a chronic subdural haematoma.
Differentials include delirium, alcohol withdrawal, stroke, epidural haematoma, seizure, substance abuse

Investigations:
CT brain is most important. A chronic subdural haematoma will appear as an isodense or hypodense crescent shaped lesion.

Vitals
Full neurological exam and MMSE
Bloods- FBC, UEC, BGL, LFTs, coagulation studies
MRI
Angiography
225
Q

You are the ED intern on duty. You are asked to see a 75 yoa man who has been brought into the hospital from home where he has been becoming increasingly confused and clumsy. He was previously well and independent. He had a fall about 3 weeks ago, when he cut his forehead. He has little medical history of note, he used to smoke and still drinks about 30 standard units of alcohol per week. On direct questioning he is confused in time and place. Physical and neurological examination are normal. A CT head shows a subdural haematoma.

What has happened inside this man’s head? What treatment is required?

A

Subdural haemorrhages are believed to be due to stretching and tearing of bridging cortical veins as they cross the subdural space to drain into adjacent dural sinus. These veins rupture due to shearing forces when there is a sudden change in the velocity of the head. 10-30% of chronic subdural haematomas show evidence of repeated haemorrhage. Rebleeding usually occurs from the rupture of stretched cortical veins as they cross the enlarged fluid filled subdural space. Subdural haematomas are interspersed between the dura and arachnoid mater, typically crescent shaped and are usually more extensive than extradural haematomas as they are limited only by the dural reflections. Initially the haematoma is encapsulated by a thick collagen and fibroblast membrane. When the haematoma becomes large enough and blood is drawn into the surrounding area by osmosis symptoms will begin to occur. At this point the haematoma can be evacuated via burr holes and the subdural space irrigated with warm saline or a hemicraniectomy and duraplasty can be performed. In the interim mannitol (an osmotic diuretic) can be used.

For patients with chronic SDH and the potential for recovery, surgical haematoma evacuation is recommended if there is evidence of moderate to severe cognition impairment or progressive deterioration. Surgical intervention is also recommended if the clot thickness is >10mm or midline shift > 5mm. Conservative management with monitoring and serial CT scans can be used if the patient does not meet these criteria.

In the patient alcohol + age = cortical atrophy stretching the cortical veins
Fall = shearing forces
Craniotomy and evacuation of haematoma as visible on CT and therefore likely greater than 1cm

226
Q

What is the importance of the primary and secondary brain injuries when managing an acute head injury?

A

Primary injury occurs at the time of the initial traumatic event, and may be local or diffuse. Focal injuries include haematomas, contusions and lacerations resulting from blunt or penetrating trauma. Diffuse injuries typically result from acceleration/ deceleration forces and affect the whole brain resulting in axonal shearing or concussion.
Secondary injury is potentially preventable and reversible and occurs after the time of the injury. Mechanisms of injury include oedema (raised intracranial pressure), ischaemia/hypoxia, hypotension, and metabolic disturbance.
Once the primary brain injury has been recognised the main objective of the management of acute traumatic brain injury is the prevention of secondary brain injury.

227
Q

What are the methods most commonly used for clinical assessment of a head injury?

A

Primary survey
Determine consciousness level on AVPU
GCS- eye opening response (4), best verbal response (5), best motor response (6)
3-8 = severe head injury = 1 in 7 probability of haematoma
9-12 = moderate head injury = 1 in 50 probability of head injury
13-14 = mild head injury = 1 in 3500 probability of haematoma

Systemic analgesia should be avoided until the full neurological assessment is performed
Secondary survey
Full assessment of head and neck
Signs of basal skull fracture (panda eyes, Battles sign [bruising over the mastoid process], CSF otorrhoea or rhinorrhea)
Repeated monitoring of vitals and GCS/AVPU
Pupil size and reactivity
Limb movements and responses

Indications for CT in head injury:
GCS < 13 at any point
Suspected open or depressed skull fracture
Any sign of basal skull fracture
Post-Traumatic seizure
Focal neurological deficit
More than one episode of vomiting
Amnesia for greater than 30 minutes
Age > 65 years, coagulopathy, dangerous mechanism of injury provided there is amnesia or loss of consciousness experienced.
228
Q

This is an abdominal x-ray taken of a 40 year old lady who has been admitted through ED with a 2 week history of diarrhoea, weight loss, lethargy and abdominal cramps.
Please look at this image.
Please tell me what the significant features are, whether they are normal or abnormal.

A

The history sounds like colitis, which can be caused by:
Inflammation- UC or Crohn’s disease
Infective- C.difficile, campylobacter
Ischaemic- likely inferior mesenteric artery if distal bowel

The main sign of colitis is thumb printing of the bowel
There may also be toxic megacolon

229
Q

This is an X ray of a 71 year old man, previous laparotomy 10 years ago. Now has 24 hours of abdominal cramps, distension and vomiting. Please look at this image. Please tell me what the significant features are, whether they are normal or abnormal (ignore the zipper).

A

Given the history of previous laparotomy there is a high suspicion of adhesion small bowel obstruction, also the classical presentation is cramping abdominal pain, abdominal distension with nausea and vomiting.
Look for:
Dilated loops of bowel proximal to the obstruction with gas fluid levels and free gas in the peritoneum. The dilated loops will be predominantly centralised in a small bowel obstruction.
Transition point of obstruction
Decompressed small bowel distal to the obstruction
Signs of herniation

NB- other causes of small bowel obstruction include neoplasm, herniation and gallstone ileus
Mucosal folds will be spread all the way across the diameter of the small bowel loops whereas the mucosal folds (haustra) are only spread across part of the diameter of the bowel loops.

230
Q

What are the advantages of CT as an investigation?

A

Excellent spatial resolution (very good for assessment of bony structures)
Good contrast resolution (the ability to distinguish between differences in intensity in an image)- 10x better than plain X-ray
Fast and therefore less susceptible to motion artefact
Widely available
Can be used to image bones, organs, tissues and tumours
Multiple planes can be seen without repositioning the patient
Reproducible findings
Gold standard for many pathologies- e.g intracranial haemorrhage

231
Q

What are the disadvantages of CT as an investigation?

A

Radiation dose (100x higher than plain x-ray) and therefore shouldn’t be used for children or in pregnancy
Use of contrast has potential adverse effects including constipation and renal dysfunction
Not portable to patient bedside
Cost is more expensive than x-ray and ultrasound

232
Q

This is a CT of a young person’s head, taken after an assault. Please look at this image. Please tell me what the significant features are and whether they are normal or abnormal.

A

Extradural/ epidural = biconvex/ lentiform shape
Subdural = crescent shaped
Unilateral 85% of the time

233
Q

This is a CT abdomen of a 19 year old woman who has presented with a history of “full abdomen, and swelling up” which has happened over several weeks. Please look at this image. Please tell me what the significant features are, whether they are normal or abnormal.

A

Crohn’s disease? []

234
Q

This is a CT of the legs of the font seat passenger of a car involved in a high speed accident. Please look at this image. Please tell me what the significant features are, whether they are normal or abnormal.

A
Things to look for include:
Hip dislocation
Hip fracture
Patella fracture/ dislocation
Femoral fracture
Tibial plateau fracture
235
Q

This is a CT of the abdomen of the driver of a car involved in a high speed accident. Please look at this image. Please tell me what the significant features are, whether they are normal or abnormal.

A
Most common is splenic injury in blunt trauma. Findings on CT include lacerations, sub scapular or parenchymal haematomas, active haemorrhage and vascular injuries.
Haemoperitoneum
Lacerations
Haematomas
Contusions
Pneumoperitoneum
Devascularisation of organs or parts of organs
Sub scapular haematomas
236
Q

This is the chest of a motorbike rider involved in a high speed accident. Please look at this image. Please tell me what the significant features are, whether they are normal or abnormal.

A

Rib fractures
Pneumothorax/ haemothorax
Pulmonary contusions
Haematoma, aortic dissection

237
Q

What are the advantages of MRI as an investigation?

A

Excellent contrast resolution (gold standard for differentiating different types of tissue)
Very good spatial resolution (not quite as good as CT)
No ionising radiation
Non-contrast vascular imaging
Lots of detail available without use of a contrast
Large field of view
Image acquired in multiple plains without the patient having to move

238
Q

What are the disadvantages of MRI as an investigation?

A
Long scan times
Large chance of motion artefact
Suboptimal bony detail
Contraindicated in patients with metal in situ
Large cost
Not mobile
Not widely available, especially in rural settings
Claustrophobia
239
Q

What are the advantages of ultrasound as an investigation?

A
No Ionising radiation
Mobile
Dynamic imaging, viewing structures and flow of blood in real time
Relatively cheap
Fantastic in pregnancy
Readily available
Easily transportable
Minimal contraindications
No need for contrast
240
Q

What are the disadvantages of ultrasound as an investigation?

A
Unable to visualise gas filled or bony structures well
Limited field of view
Heavily operator dependent
Very difficult in obese patients
Low penetration
Requires pressure- discomfort if in pain
Lots of artefacts
241
Q

What is the role of a chest drain in treating a haemothorax?

A

A chest drain allows the removal of blood from the pleural space. Removing blood allows the lung to expand against the chest wall and helps to arrest continuing haemorrhage from intercostal vessels.
The goals of chest drain in acute haemothorax are drainage of fresh blood, measurement of the rate of bleeding, improvement of ventilation, decreasing risk of clotted haemothorax and evacuation of any coexisting pneumothorax.

The aim of treatment of a haemothorax is fluid resuscitation as needed, usually tube thoracotomy and sometimes a thoracotomy.
Patients with sign of hypovolaemia (e.g tachycardia and hypotension) are given IV crystalloid and sometimes blood transfusions.
If blood volume is sufficient to be visible on chest x-ray (usually requiring 500ml) or if pneumothorax is present a large calibre chest tube is inserted in the 5th or 6th intercostal space mid-axillary line. Blood collected via tube thoracosotmy can be autotransfused.
Urgent thoracotomy is indicated when there is more than 1500ml of blood loss or bleeding rate is greater than 200ml/hour for more than 2 hours and causes haemodynamic compromise.

242
Q

How can rib fractures cause the death of a patient?

A

Rib fractures are often simple fractures that heal while managed conservatively with analgesia and physiotherapy. However they may also result in other significant injuries that can become life threatening.
Immediate:
Cardiac damage- bleeding into the pericardial cavity causing severe haemodynamic shock and may be the cause of a cardiac arrest
Tension pneumothorax
Haemothorax
Pulmonary contusion
Aortic rupture
Splenic laceration
Hepatic injury
Flail chest- which can lead to respiratory compromise failure. On inspiration the detached segment gets sucked in leading to paradoxical movement and the inhaled air shunts back and forth between the lungs

Late:
Pneumonia- this is the most common complication as the pain prevents the patient from breathing deeply and coughing
Empyema- result from residual haematoma

243
Q

DVT and PE are common in trauma patients. How can they be prevented?

A

Mechanical prophylaxis:
Treatment of avoidable risk factors
Active mobilisation
Thromboembolic deterrent stockings (TEDs)- graded compression stockings increase venous return by continuous direct compression of the infrapopliteal veins. This should be accompanied by elevation of the legs.
Intermittent pneumatic compression devices- cuffs around the legs that fill with air squeezing veins and increasing venous blood flow.
Drugs that act on the clotting cascade:
Heparin
LMWH- e.g enoxaparin (activates antithrombin III)

NB:
Risk factors for VTE in trauma patient include spinal cord injury, head trauma, lower extremity fracture, pelvic fracture, need for surgical intervention, increasing age, femoral vein line insertion, surgical repair of venous injuries, prolonged immobilisation, prolonged hospital stay, high injury severity score.
Prophylaxis should be used until patients regain full mobility.

244
Q

Why do we use a simple mnenomic (ABCDE) when assessing trauma patients? What does it stand for?

A

WHY:
The ABCDE mnemonic for the primary survey of patients allows for the ability to identify and treat life threatening conditions according to priority, with a clear, simple, organised and easily remembered approach to managing a severely injured patient. This also allows for consistency between doctors and centres and allows for life threatening injuries to be addressed during the ‘golden hour’ (the period of time following traumatic injury in which there is the highest likelihood that prompt medical and surgical treatment will prevent death).

WHAT DOES IT STAND FOR:
Airway protection, including stabilisation of the cervical spine
Breathing (maintain adequate oxygenation)
Circulation (control haemorrhage and maintain adequate organ perfusion)
Disability (perform basic neurologic evaluation)
Exposure (undress patient and search everywhere for possible injury while preventing hypothermia)

Airway
Head tilt + chin lift or jaw thrust + look inside oropharynx for foreign bodies
Endotracheal intubation (if GCS < 8 or other indication) + CXR to check intubation
Crichothyroidotomy (if intubation not possible)

Breathing
Assess ventilation with inspection, palpation, percussion/ auscultation
Oxygen saturation, RR
CXR
Manage specific injuries such as open chest wound, flail segment, pneumothorax, haemothorax, cardiac tamponade, diaphragmatic rupture

Circulation
Assess pallor, HR, BP, temp, cap refill, ECG and urine output
Control haemorrhage
Insert two large bore IV cannulas into large veins
Urgent cross matching and standard biochemical and haematological analysis
If shocked rapid infusion of 2L warmed crystalloid solution in both cannulas
If failure to respond consider transfusion of packed red cells + FFP
FAST scan and reduction of fractures

Disability
Basic neuro assessment: AVPU (alert, respond to verbal stimuli, responds to painful stimuli, unresponsive) + pupil size, reactivity, inequality
BSLs
Estimate GCS

Exposure:
Rapid top to toe assessment for external injury and for signs of injury while ensuring the patient does not become hypothermic and patient dignity.
Imaging that has not already been performed (FAST, CXR, CT etc)

245
Q

In a laparotomy for trauma, what are the surgical priorities?

A

There are two main indications for a laparotomy in trauma; peritonitis and haemorrhage
The main priorities are:
1. To control haemorrhage- immediate blunt pressure control of haemorrhage sites
2. Control contamination- rapid closure of hollow viscus injuries.
3. Provide temporary abdominal closure to prevent abdominal compartment syndrome
4. Facilitate subsequent procedures once the patient has been resuscitated. The strategy aims to bring the lethal triad of acute coagulopathy (hypothermia, coagulopathy, acidosis) under control so that the patient will be able to tolerate further surgery once he or she improves.

This is achieved by:
Short time in OT
Limited focused surgery to control haemorrhage and decontaminate
Packing
Partially resect organs
Staple off and remove injured bowel
Fibrin sealants
Leave abdomen open
246
Q

What is a pneumothorax? How can it be treated?

A

Pneumothorax is air in the pleural space causing partial or complete lung collapse. Pneumothorax can occur spontaneously or result from trauma or medical procedures.
Management of a pneumothorax includes:
- Observation and follow up x-ray for small asymptomatic, primary spontaneous (no underlying lung pathology) pneumothraces
- Catheter aspiration for large or symptomatic primary spontaneous pneumothraces- small bore catheter into 2nd intercostal space midclavicular line and attached to 3-way stopcock and syringe, followed by air withdrawing from the pleural space until the lung re-expands or until 4L of air is removed.
- Tube thoracostomy for secondary (underlying pulmonary disease) and traumatic pneumothraces

247
Q

What is a tension pneumothorax?

A

Tension pneumothorax is accumulation of air in the pleural space under pressure, compressing the lungs and decreasing venous return to the heart.
Tension pneumothorax develops when a lung or chest wall injury is such that is allows air into the pleural space but not out of it. As a result air accumulates and compresses the lung, eventually shifting the mediastinum, compressing the contralateral lung and increasing intrathoracic pressure enough to decrease venous return to the heart, causing shock.
Causes include mechanical ventilation and simple pneumothorax with lung injury that fails to seal following penetrating or blunt chest trauma or failed central venous cannulation.

248
Q

What are the symptoms and signs of a simple pneumothorax?

A

Symptoms of pneumothorax include dyspnoea and pleuritic chest pain. Dyspnoea may be sudden or gradual in onset depending on the rate of development and size of the pneumothorax. Pain may also be referred to the shoulder or abdomen.
Physical findings typically consist of absent tactile fremitus, hyperresonance to percussion, and decreased breath sound on the affected side.

249
Q

What are the symptoms and signs of a tension pneumothorax?

A

Symptoms and signs initially are those of simple pneumothorax. As intrathoracic pressure increases, patients develop hypotension, tracheal deviation and neck vein distension. The effected hemithorax (half of the thorax) is hyper resonant to percussion and often feels somewhat distended, tense and poorly compressible to palpation.

250
Q

What is the treatment for a suspected tension pneumothorax?

A

Tension pneumothorax is a medical emergency and should be diagnosed clinically; time should not be wasted confirming the diagnosis with a chest x-ray. It should be treated immediately by inserting a 14 or 16 gauge needle with a catheter through the chest wall in the 2nd intercostal space at the midclavicular line, just above the third rib to avoid the neuromuscular bundle just below the second rib. The sound of high pressure air escaping confirms the diagnosis. The catheter can be left open to air or attached to a Heimlich valve (one way valve that prevents air going back into the chest). Emergency decompression must be followed immediately by tube thoracostomy (insertion of a chest tube into the pleural space), after which the catheter is removed.

251
Q

What is shock, and what is the most common cause of shock in trauma patients?

A

Shock is inadequate perfusion of vital organs due to circulatory failure, especially the heart and the brain. If untreated shock rapidly proceeds to irreversible organ damage and death of the patient.
The most common cause of shock in trauma patients is hypovolaemic shock, usually as a result of haemorrhage.
A reduction in circulating volume results in a reduction in stroke volume and cardiac output. Initially the body compensates by increasing sympathetic activity, raising the systemic vascular resistance and therefore the blood pressure. As volume loss increases blood pressure falls leading to the signs and symptoms of hypotension, hyperventilation, rapid weak pulse, cold, clammy cyanotic hands and oliguria.

Other types of shock include:
Cardiogenic- occurs when heart function is impaired
Septic/ distributive- systemic inflammation causes cardiac depression and microvascular changes
Anaphylactic- acute hypersensitivity to an allergen

252
Q

What is permissive hypotension and what is haemostatic resuscitation?

A

Permissive hypotension and haemostatic resuscitation are terms relating to hypovolaemia and are components of damage control resuscitation.

Permissive hypotension is a strategy to suggest that limited volume replacement the maintains minimally adequate organ perfusion may improve outcomes. Permissive hypotension targets early fluid resuscitation only to a systolic BP of 70 mmHG.
Controlled hypotension may be beneficial in patients with haemorrhagic shock due to torso injuries, from gun shots or stab wounds, or patients with ruptured AAA. However it may be detrimental to patients with brain injury as hypotension reduces cerebral perfusion and increases mortality. The rationale behind permissive hypotension is that aggressive fluid resuscitation might lead to dilution of clotting factors, production of hypothermia, oedema leading to compartment syndrome and acute lung injury and disrupt thrombus formation and enhance bleeding.

Haemostatic resuscitation aids to avoid or ameliorate acute coagulopathy of trauma and the complications of aggressive crystalloid fluid resuscitation while maintaining circulating volume. It involves the use of blood products in ratios resembling that of whole blood, often 1:1 FFP to packed red blood cells. This is also to prevent the lethal triad of hypothermia, metabolic acidosis and acute coagulopathy of trauma.

253
Q

What is the difference between spinal shock and neurogenic shock?

A

Neurogenic shock is classically characterised by hypotension, bradycardia and peripheral vasodilation. Neurogenic shock is due to loss of sympathetic vascular tone and happens only a significant portion of the sympathetic nervous system has been damaged- as occurs with lesions at the T6 level or higher. This affects vasomotor tone and cardiac function due to failure of descending sympathetic pathways of the upper thoracic cord.
Spinal shock is not a true form of shock. It refers to flaccid areflexia with possible priapism (prolonged erection of the penis) that may occur after spinal cord injury, and may last hours to weeks. It resolves as soft tissue swelling improves.

254
Q

What are the 5 sources of hypovolaemic life threatening bleeding, and how do you identify/diagnose them?

A

Pelvis- pelvic X-ray
Long bones- exposure/ inspection or X-ray
Abdomen- FAST scan or diagnostic peritoneal lavage or CT (more accurately diagnosis solid organ injuries but is not as specific for blunt bowel and mesenteric injuries)
Chest- CXR
External- exposure/ inspection

255
Q

What is the treatment for a trauma patient who has bleeding from an open wound identified as their immediate life threatening injury?

A

Primary survey
Organise trauma team, call surgeon, notify blood bank.
Perfusion indices including ABGs, lactate, Hb and HCT

Definitive management:
Apply direct pressure to the open haemorrhaging wound
If bleeding persists consider sutures, cautery, interventional radiology or damage control surgery

256
Q

What is the treatment for a trauma patient who has a haemothorax identified as their immediate life threatening injury?

A

The aim of treatment of a haemothorax is fluid resuscitation as needed, usually tube thoracotomy and sometimes a thoracotomy.

Patients with sign of hypovolaemia (e.g tachycardia and hypotension) are given IV crystalloid and sometimes blood transfusions.

If blood volume is sufficient to be visible on chest x-ray (usually requiring 500ml) or if pneumothorax is present a large calibre chest tube is inserted in the 5th or 6th intercostal space mid-axillary line. Blood collected via tube thoracosotmy can be autotransfused.
Urgent thoracotomy is indicated when there is more than 1500ml of blood loss or bleeding rate is greater than 200ml/hour for more than 2 hours and causes haemodynamic compromise.

257
Q

What is the treatment for a trauma patient who has intrabdominal bleeding identified as their immediate life threatening injury?

A

Primary survey
Organise trauma team, notify blood bank, call surgeon

Interventional radiology and demobilisation
Damage control laparotomy

258
Q

What is the treatment for a trauma patient who has intracranial bleeding identified as their immediate life threatening injury?

A

Primary survey
Organise trauma team, notify blood bank, call surgeon

Non-surgical management is only indicated in patients with minimal neurological symptoms or haemorrhage volume <10ml.

Surgical options are generally for decompression of the brain to release pooled blood and relieve pressure. This may be done through:
Burr hole procedure
Craniectomy incision (partial removal of the skull to allow the brain swelling to expand)
Craniotomy (opening of the skull cavity) after which the skull fragment is immediately put back into place.

Other treatments include:
Anti-anxiety medications to control blood pressure
Anti-epileptic drugs for seizure control
Analgesia for pain
Stool softeners to prevent straining during constipation
Fluids and nutrients as necessary

259
Q

What is the treatment for a trauma patient who has bleeding from a long bone (femur) fracture identified as their immediate life threatening injury?

A

Primary survey and secondary survey
Organise trauma team, notify blood bank, call surgeon

The leg should be realigned and splinted with skin traction and a Thomas splint. This will help to control pain and haemorrhage. An x-ray should only be attained after splinting has occurred.
If the fracture is an open fracture the wound should be photographed, cleaned, dressed with a butadiene dressing, IV antibiotics should be commenced and tetanus given if necessary and the fracture should be reduced.

NB: the average blood loss from a femoral fracture is 1-2.5L.

260
Q

What is the treatment for a trauma patient who has bleeding from a pelvic fracture identified as their immediate life threatening injury?

A

Primary survey
Organise trauma team, notify blood bank, call surgeon

Definitive management:
Reducing the pelvic volume by use of a pelvic binder or binding a bedsheets tightly around the pelvis and internally rotating the hips (emergency procedure) or by application of an external pelvic fixator in theatre as a more definitive solution
Emergency laparotomy, during which pelvic stabilisation and/or pelvic packing is performed pending definitive management of the injury.
Once the parent is haemodynamically stable the patient may have an abdominopelvic CT with IV contrast and surgical fixation as required.

261
Q

Why should a trauma patient who has a need for a large volume transfusion due to their bleeding be given a combination of Packed red cells, Fresh frozen plasma, platelets and cryo-precipitates, rather than just packed red cells?

A

The combination of packed red blood cells, fresh frozen plasma (FFP), platelets and cryoprecipitates is given in order to achieve haemostasis and correct coagulopathy caused by massive haemorrhage. It has been shown that FFP to packed RBC ratios of 1:1 drastically improve survival rate. Cryoprecipitate provides an additional option for factor replacement for a lower volume of fluid, notably replacing factor VIII (and therefore Von Willebrand’s factor) and fibrinogen.
Blood products also provide additional advantages over isotonic crystalloid solutions in trauma patients as large volumes of crystalloid solution can lead to dilution coagulopathy and exacerbate bleeding as well as decrease organ function due to volume overload and oedema. Furthermore, crystalloid solutions have no O2 carrying capacity and do little to correct the anaerobic metabolism associated with shock.

Cryoprecipitates- contains factor VIII of the intrinsic pathway (so it decreases your aPTT), Von Willebrand’s factor (as it is bound to factor VIII in circulation) and fibrinogen.
Fresh frozen plasma- contains all of the clotting factors, fibrinogen, plasma proteins electrolytes, physiological anticoagulants (protein C, protein S, antithrombin etc) and added anticoagulants.

NB to reverse warfarin prothrombinex is the preferred agent as it contains factors II, IX and X and a small amount of factor VII. When prothrombinex is not readily available FFP may also be used.

262
Q

How do you assess a middle aged patient who presents with an episode of microscopic haematuria?

A

History:
- Timing of haematuria- blood from the kidneys, ureters and bladder will completely mix in the urine whereas urethral bleeding may occur independently of micturition
- Acute trauma
- Currently in menses (false positive for haematuria)
Associated symptoms- colic pain, retention, symptoms of UTI (dysuria, frequency, urgency, pneumaturia)
- Difficulty initiating urine stream, slow stream, hesitancy or post void dribbling
- Systemic symptoms- fever, weight loss
- PMHx- bleeding diathesis, sickle cell, stones, cancer, structural kidney diseases, pelvic surgery/ radiation, catheterisation, endometriosis
- Medications- anticoagulants
- Past surgical history- renal or urological procedures
- Smoking history
Examination:
Loin inspection and palpation/ ballot kidneys
DRE for prostate assessment
Blood pressure

Investigations:
Urine dipstick
MSU and MCS
Urine cytology
Bloods- FBC, ESR/CRP, serum albumin, UEC, PSA, CMP, uric acid and PTH levels (stone disease), coags
Imaging- renal ultrasound, cystoscopy, X-ray KUB (stones), CT KUB (renal/prostate/ stones/ bladder mass)

Differential diagnosis:
Urinary tract infection- most common cause
Renal stones
Malignancy (typically painless haematuria)
Renal cell carcinoma
Transitional cell carcinoma
Post interventional (e.g post TURP or post cystoscopy)
Glomerulonephritis
Vasculitis
Obstruction such as BPH
Trauma of kidney- polycystic kidney disease especially susceptible
Emboli or renal vein thrombosis

263
Q

A 60 year old woman attends ED with a 3 week history of blood in the urine, including some clots. She has had an intermittent urinary stream in the past 24 hours, and is complaining of suprapubic pain when voiding. She has had frequency and urgency of voiding for 1 month. There is nothing remarkable in the rest of her history. On exam, she is pale, pulse of 100bpm, BP 105/70mmHg, her Hb is 8.2g/dl with a pattern consistent with iron deficiency, rest of her blood work is normal.
What is the important diagnosis to exclude and what investigations do you want to do?

A

The important diagnosis to exclude is a urinary malignancy, likely a transitional cell/ urothelial carcinoma.

Investigations include:
FBC- may show mild anaemia
Urinalysis- haematuria and pyuria (pus in the urine)
Renal and bladder ultrasound or CT urogram to image the urinary tract
Urine cytology- identifies malignant cells that have been exfoliated from the urothelium or squamous cells into the urine. The specificity is greater than 90% while the sensitivity for high grade disease and carcinoma in situ is 80-90% however there is low sensitivity for detecting low grade neoplasms (20-50%).
Cystoscopy- Low grade tumours are papillary and readily visible, while high grade tumours may be flat and more difficult to see. Carcinoma in situ is often not visible.

To evaluate metastatic disease I would also like to do
LFTs- increased alkaline phosphatase indicates bony metastasis
Bone scan- if alkaline phosphatase is raised

NB:
Overall 80-90% of cancers of the bladder are urothelial carcinomas (previously termed transitional cell carcinomas). Squamous cell carcinomas are the next most common form of bladder cancer and adenocarcinomas are the rarest form.
The most common presenting symptoms include:
- Presence of risk factors- risk factors include tobacco exposure, male gender, age > 55, exposure to chemical carcinogens, pelvic radiation, systemic chemotherapy and family history of bladder cancer.
- Haematuria- present in over 80% of cases. Gross, painless haematuria which is present throughout the entire urinary stream is the most common.
- Dysuria- typical of carcinoma but also seen in high grade urothelial carcinoma
- Urinary frequency

Urine cytology
Cystoscopy

264
Q

You are asked to see a 75 year of age man in ED. He has not been able to urinate for 12 hours and is in acute pain. The ED doctor referring him has excluded all other conditions, but has had to attend a resus room case before considering his urinary tract, no imaging modality is available.

What will you do to assess this man and how will you treat him?

A

The patient has acute urinary retention which is a urologic emergency and should be promptly managed.
History:
- Signs and symptoms- inability to pass urine, abdominal pain, haematuria, fever, dysuria, neurologic symptoms, low back pain
- Previous history of retention or lower urinary tract symptoms
- History of prostate disease
- Pelvic or prostate surgery, pelvic trauma or radiation
- Recent medication changes

Examination:
-	General inspection
-	Vitals
-	Percuss bladder
-	Rectal examination- masses, faecal impaction, perineal signs, prostate
-	Neurologic exam- strength, sensation, reflexes, muscle tone
Investigations:
-	FBC, UEC
-	Urinalysis
-	Bladder ultrasound, cystoscopy

Management:

  • Analgesia- relaxation and may aid spontaneous micturition
  • Cease any causative medication
  • Catheterise if retention persists
  • Alpha blockers in the case of BPH and consider TURP

Differential diagnosis of acute urinary retention:

  • BPH
  • Carcinoma
  • Post urologic surgery
  • Bladder or urethral stone impaction
  • Pressure on bladder (pregnancy or faecal impaction)
  • Infection- acute prostatic
  • Trauma
  • Pharmacological- anticholinergics, anaesthetics, alpha agonists
  • Loss of neurological control- spinal injury, epidural/spinal anaesthesia
265
Q

An 81 yoa man presents to the ED complaining of difficulty passing urine. This has been a problem for 6 months, and is getting worse. He has a poor flow when he passes water, is getting up at night more frequently, and has wet the bed on several occasions in the last month. His bowels open regularly, every second day, and have not changed. On examination, there is a palpable, non-tender, suprapubic mass which is dull to percussion. The rest of the abdominal and systemic exams are normal. DRE reveals a large, smooth, soft prostate gland and nothing else.
What is the diagnosis?
Why is he complaining of bedwetting?

A

The diagnosis is benign prostatic hyperplasia which is causing a bladder outlet obstruction and chronic urinary retention leading to overflow incontinence.

He is complaining of bedwetting because BPH leads to compression of the urethra leading to a bladder outflow obstruction. This leads to chronic urinary retention, bladder distension, an atonic bladder (loss of muscle tone) and overflow incontinence. This primarily affects him at night because the combination of the full bladder and nocturnal relaxation of the pelvic floor, tends to occur.

NB: The symptoms of bladder obstruction include:

  • Incomplete bladder emptying after urination
  • Frequency (less than 2 hours)
  • Intermittent flow (stopping and starting during urination)
  • Urgency (difficult to postpone urination)
  • Week stream
  • Straining to begin urination
  • Nocturia
  • Hesitancy
  • Post micturition dribbling
  • Symptoms precipitated by excessive fluid intake
266
Q

An 81 yoa man presents to the ED complaining of difficulty passing urine. This has been a problem for 6 months, and is getting worse. He has a poor flow when he passes water, is getting up at night more frequently, and has wet the bed on several occasions in the last month. His bowels open regularly, every second day, and have not changed. On examination, there is a palpable, non-tender, suprapubic mass which is dull to percussion. The rest of the abdominal and systemic exams are normal. DRE reveals a large, smooth, soft prostate gland and nothing else.

What is the diagnosis?

What is the management?

A

The diagnosis is most likely benign hypertrophic hyperplasia
Management options include:

Drug treatment:
- 5 alpha reductase inhibitors (finasteride)- block the conversion of testosterone to dihydrotestosterone, preventing hyperplasia
- Alpha adrenergic receptor blocker (tamsulosin)- present in the bladder neck and prostate- relax smooth muscle of prostatic urethra to decrease outlet resistance
Surgical therapy:
- Transurethral resection of prostate (TURP)- This is the standard surgical therapy and involves removing the bulk of the prostate and leaving a compressed venous plexus to prevent bleeding. It has a 5-10% risk of erectile impotence.
- Retropubic prostatectomy- rarely performed
- Laser vaporisation
Long term catheterisation- only recommended for patients with significant perioperative morbidity.

Management is determined by the IPSS score, with alpha adrenergic blockers given in mild disease and 5 alpha reductase inhibitors given in more severe disease.

267
Q

An 81 yoa man presents to the ED complaining of difficulty passing urine. This has been a problem for 6 months, and is getting worse. He has a poor flow when he passes water, is getting up at night more frequently, and has wet the bed on several occasions in the last month. His bowels open regularly, every second day, and have not changed. On examination, there is a palpable, non-tender, suprapubic mass which is dull to percussion. The rest of the abdominal and systemic exams are normal. DRE reveals a large, smooth, soft prostate gland and nothing else.
What is the diagnosis? What features of the prostate, if found on examination, would make you concerned about prostate cancer?

A

The diagnosis is most likely benign prostatic hyperplasia.
It is hard to clinically differentiate between benign prostatic hyperplasia and prostate cancer however concern for prostate cancer from examination of the prostate would be due to:
Smooth enlargement
Nodular asymmetrical surface
Large, hard irregular gland with evidence of extension beyond the capsule or into the seminal vesicles.
General features to note are tenderness, nodules, asymmetry and induration (hardened mass)
Other features on history that would make cancer more likely include; fatigue, weight loss, night sweats, bone pain, age > 50, FHx, ethnicity, high dietary fibre

268
Q

What are the advantages of screening 55 year old man with a PSA test for prostate cancer?

A
  • Low risk test
  • Easily reproducible and not operator dependent
  • Early PSA tests can detect cancer before symptoms occur
  • Most prostate cancers detected in patients with a > 15 year life expectancy are significant tumours that are likely to cause significant morbidity and mortality. Therefore testing is likely to be more beneficial in men between the ages of 50 and 70, especially if there is a family history of prostate cancer
269
Q

What are the disadvantages of screening 55 year old man with a PSA test for prostate cancer?

A
  • PSA is not specific- PSA is also raised in other conditions such as benign prostatic hyperplasia and prostatitis.
  • Only 40% of patients with PSA between 4 and 10 will have cancer
  • 20% of patients with PSA less than 4 will have cancer
  • Most prostate cancers are slow growing and are not a threat to life. These cancers may be treated unnecessarily leading to the risk of side effects such as impotence, urinary incontinence and proctitis.
  • There is no good clinical trial evidence that PSA screening saves lives
  • Prostate biopsies can cause rectal bleeding, dysuria, haematuria, haematospermia and infection and sepsis
270
Q

What are the classical symptoms and signs of renal cell carcinoma?

A

A renal cell carcinoma is a malignant tumour of the renal parenchyma originating in the renal tubules.
The classic presentation is with the triad of haematuria, a renal mass and flank pain; although all three features only occur in about 15% of cases, one is present in 40% of patients. Uncommon presentations include iron deficiency anaemia, polycythaemia due to erythropoietin production, hypertension due to renin production, hypercalcemia due to parathormone-like protein production, pyrexia of unknown origin, elevated ESR or as secondary lesions.

NB- renal cell carcinoma is a renal cell malignancy arising from the real parenchyma/ cortex and accounts for about 85% of renal cancers.

271
Q

A 60 year old woman attends ED with a 3 week history of blood in the urine, including some clots. She has had an intermittent urinary stream in the past 24 hours, and is complaining of suprapubic pain when voiding. She has had frequency and urgency of voiding for 1 month. The is nothing remarkable in the rest of her history. On exam, she is pale, pulse of 100bpm, BP 105/70mmHg, her Hb is 8.2g/dl with a pattern consistent with iron deficiency, rest of her blood work is normal.

What is the important diagnosis to exclude, what factors are relevant in taking a history in this case?

A

The important diagnosis to exclude = carcinoma in situ of bladder
History:
- How long? Has it happened before? Recent trauma?
- Painful or painless? (painless suggests malignancy)
- Timing of blood in urine stream
• Initial haematuria = urethral cause
• Terminal haematuria = bladder stones
• Throughout urine stream = bleeding of bladder or upper tract (kidney/ ureteral)
- Lower urinary tract symptoms (e.g. dysuria, frequency, urgency, urethral discharge)
• Presence suggests infectious or inflammatory process
- Risk factors – smoking, pelvic radiation, occupational exposure to benzenes and amines (dry cleaners)
- Symptoms of glomerular cause (e.g. oedema, weight gain, oliguria, dark urine)
- Family history (inc. kidney stones, cancer, prostatic enlargement, sickle cell anaemia, collagen vascular disease, renal disease)
- Menstruation
- Trauma
- Sexual activity
- Medications (aspirin, NSAIDs, warfarin, NOACs, cytotoxic agents, rifampicin)
- Recent sore throat - ? glomerulonephritis
- Other respiratory tract infection - ?IgA nephropathy
- Systemic illness
- Travel abroad - ?malaria, ?schistosomiasis

Investigations:
- Urinalysis for haematuria (and exclude infection)
- Urine cytology – positive in 90% of carcinoma in situ
- US renal and bladder – for bladder tumour or upper tract obstruction
- Cystoscopy enables pathological diagnosis
• Typically, via a flexible cystoscopy under local anaesthetic
• Rigid cystoscopy will then be required for more definitive assessment
• Biopsy of any tumours identified and potential resection via transurethral resection of bladder tumour (TURBT)
- CT urogram with contrast in excretory phase
- FBC – will show reduced haemoglobin levels
- Urinary markers of bladder cancer

272
Q

You are an ED intern. A 40 yoa woman presents to the emergency department. She has a fever of 39.4C, Blood pressure of 90/40mmHg, pulse 120 regular, respiratory rate of 24, she is drowsy and confused when answering questions. She initially complained of left sided abdominal pain, and the pain came in waves 3 - 10 minutes apart, with an urge to pass urine. She has a history of left sided renal calculi, and 48 hours before presentation she had lithotripsy (ESWL) to her left sided kidney stone.

What is the likely diagnosis? What will you do?

A

The likely diagnosis is a ureteric stone that is acting as a nidus for infection leading to urosepsis.
The patient is clearly septic and likely as a consequence of the obstruction has developed pyenephrosis (collection of pus in the renal pelvis[ broaden top part of the ureter]) of the kidney.
This has likely occurred as a result of the failed passage of calculus fragments from lithotripsy, leading to steinstrasse (acute stone fragment impaction).

Considering this, she needs initial resuscitation, followed by relief of the obstruction as this is a urological emergency.
Resuscitation:
Primary survey
O2 with non rebreather
2x large bore IV cannulas- fluid resuscitation
Bloods- FBC, UEC, lactate, cultures, serum calcium, phosphate and uric acid, CRP (with WCC raises suspicion for infection)
Analgesia
Antiemetics
Indwelling catheter and monitor hourly urine output
IV antibiotics- gentamicin and ampicillin
Urological consult for decompression with a retrograde ureteric stent insertion.

Investigations:
Urine dipstick, midstream urine MCS, 24 hour urine for calcium, phosphate, oxalate, urate, cystine and xanthine
KUB x-ray (90% of ureteric stones are radio-opaque as they contain calcium
Renal ultrasound
CT abdo/pelvis- gold standard for locating stones and evidence of complications

Management:

  1. Conservative management for stones less than 1cm with no complications- hydration, pain control, anti-emetics)
  2. Stones between 1cm and 2cm are in general treated with extracorporeal shock wave lithotripsy.
  3. Percutaneous nephrostolithotomy is a minimally invasive form of treatment that is usually reserved for renal and proximal ureteric stones and more distal stones > 2cm or that don’t respond to lithotripsy.
273
Q

A 30 year old man presents with a lump in the right testicle. It is painless, he noticed it a few weeks ago, due to discomfort running. He is a smoker and has a persistent cough. On examination, there is a 3cm distinct lump within the right testis, it is separate from the rest of the testis and epididymis, is not tender and does not transilluminate. Abdominal and chest exam are normal.

What is the likely diagnosis, what are the necessary investigations?

A

The most likely diagnosis is a testicular cancer.
Signs and symptoms include a testicular mass, swelling of the testis, heaviness in the scrotum and discomfort or pain.
Cryptochordism, family history and infertility are risk factors for testicular cancer.

Germ cell tumours account for the majority of testicular cancers. Germ cells give rise to two main kinds of cancer, seminomas and nonseminomatous tumours.
• Seminomas- Are the most common germ cell type of testis cancer. They occur most often in men between the ages of 20 and 45 years old. They are usually treated surgically sometimes with adjuvant radiotherapy and may also be treated with chemotherapy in the case of metastasis.
• Nonseminomatous germ cell tumours- may be more aggressive and are treated somewhat differently from seminomas. These tumours also occur in men in their 20s and 30s. The histologic types are:
- Embryonal carcinoma
- Yolk sac carcinoma
- Choriocarcinoma
- Teratoma
• Carcinoma in situ- a non-invasive form of germ cell cancer that hasn’t spread beyond the tubes in the testes. Sometimes the best option for treatment is watchful waiting.
• Stromal cell tumour- Cancers that develop in the stromal cells, the supportive tissue of the testes that produce testosterone. However the tumours are usually benign. They account for fewer than 1 in 20 adult testicular tumours but up to 1 in 5 testicular tumours in boys. There are two types, Leydig cell tumours and sertoli cell tumours. They are usually cured by surgery.
• A tumour in the testicles can also be secondary from lymphoma, leukaemia, prostate, lung, skin and kidneys.

• Scrotal doppler ultrasound
- First line investigation
- Can confirm a solid intratesticular mass, as opposed to a fluid filled cyst, that is indicative of testicular cancer.
• Tumour markers:
- Beta HCG- secreted by synctiorophoblastic cells
- Alpha fetoprotein (AFP)- produced by yolk sac elements
- LDH- elevated in more than half of metastatic seminomas.
- Seminomas- occasionally raised beta HCG but never raised AFP
- Nonseminomatous- often raise AFP and/or beta HCG

•	Urine:
-	MSU and MCS
•	Blood tests:
-	Raised inflammatory markers (WCC, CRP)
-	Low haemoglobin and albumin
-	Tumour markers as above
•	Surgical orchidectomy and histology of the mass to confirm diagnosis
274
Q

A 30 year old man presents with a lump in the right testicle. It is painless, he noticed it a few weeks ago, due to discomfort running. He is a smoker and has a persistent cough. On examination, there is a 3cm distinct lump within the right testis, it is seperate from the rest of the testis and epididymis, is not tender and does not transilluminate. Abdominal and chest exam are normal.
What is the likely diagnosis, how do you differentiate between different scrotal lumps on clinical examination?

A

The likely diagnosis is testicular cancer.
Inspection:
Rash, ulcers, erythema (cellulitis/ fungal infections)
Enlargement of the testis and is it bilateral or unilateral

Examination of the testes:

  • Use both thumbs and index fingers to gently palpate the whole testicle
  • Determine the size/ shape of the mass
  • Is the mass regular or irregular
  • Consistency- hard (solid), soft (cystic), bag of worms (varicocele)
  • Are you able to get above the mass:
  • No = inguinal hernia
  • Yes = hydrocele
  • Is the mass fixed to the testicle or separate:
  • Separate and hard = epididymitis/ orchitis
  • Separate and soft = epididymal cyst / spermatocele
  • Is there a cough impulse- suggests hernia/ varicocele
  • Does the mass transilluminate- transillumination suggests the mass is filled with fluid (e.g hydrocele)

Epididymal cysts- separate from the body of the testicle. Found posterior to the testicle. Does transilluminate. Smooth surface with a fluctuant consistency.
Hydrocele- Soft non tender swelling usually below and anterior to the testicle. It is confined to the scrotum and you can get above it on examination. It does transilluminate. The testes may be difficult to palpate if the hydrocele is large.
Varicocele- Classically described as a bag of worms. Cough impulse present. Hard to get above. Varicoceles do not transilluminate.
Inguinal hernia- Hard to get above, does not transilluminate, found superior to the testes.

Differential Diagnosis:
Epididymal cysts
•	Epididymal cysts are the most common cause of scrotal swellings seen in primary care. 
-	Features
-	Separate from the body of the testicle
-	Found posterior to the testicle
•	Associated conditions
-	Polycystic kidney disease
-	Cystic fibrosis
-	Von Hippel-Lindau syndrome
•	Diagnosis may be confirmed by ultrasound.
•	Management is usually supportive but surgical removal or sclerotherapy may be attempted for larger or symptomatic cysts.

Hydrocele
• A hydrocele describes the accumulation of fluid within the tunica vaginalis. They can be divided into communicating and non-communicating:
- Communicating: caused by patency of the processus vaginalis allowing peritoneal fluid to drain down into the scrotum. Communicating hydroceles are common in newborn males (clinically apparent in 5-10%) and usually resolve within the first few months of life
- Non-communicating: caused by excessive fluid production within the tunica vaginalis
• Hydroceles may develop secondary to:
- Epididymo-orchitis
- testicular torsion
- testicular tumours
• Features
- Soft, non-tender swelling of the hemi-scrotum. Usually anterior to and below the testicle
- The swelling is confined to the scrotum, you can get ‘above’ the mass on examination
- Transilluminates with a pen torch
- The testis may be difficult to palpate if the hydrocele is large
• Diagnosis may be clinical but ultrasound is required if there is any doubt about the diagnosis or if the underlying testis cannot be palpated.
• Management
- Infantile hydroceles are generally repaired if they do not resolve spontaneously by the age of 1-2 years
- In adults a conservative approach may be taken depending on the severity of the presentation. Further investigation (e.g. ultrasound) is usually warranted however to exclude any underlying cause such as a tumour

Varicocele
• A varicocele is an abnormal enlargement of the testicular veins. They are usually asymptomatic but may be important as they are associated with infertility.
• Varicoceles are much more common on the left side (> 80%).
• Features:
- classically described as a ‘bag of worms’
- Subfertility
• Diagnosis
- ultrasound with Doppler studies
• Management
- Usually conservative
- Occasionally surgery is required if the patient is troubled by pain. There is ongoing debate regarding the effectiveness of surgery to treat infertility

275
Q

What are the indications for a person to be put on the kidney transplant waiting list?

A

Medical indications:
All patients with chronic kidney disease at stage 4 or 5 (GFR < 15 and GFR between 15 and 30 respectively) with progressive disease likely to require renal replacement therapy within the next 6 months
Anticipated low perioperative mortality
80% likelihood of surviving for at least 5 years after transplantation

Psychosocial indications:
Ability to understand risks and benefits and cooperate with medical regimen
Adequate psychosocial support
Able to afford follow up care, post transplant medications etc

No absolute contraindications to transplantation such as:
Untreated malignancy
Active infection
Untreated HIV or AIDS
Any condition where life expectancy is under 2 years

276
Q

What tissue matching tests are required when the transplant service is deciding which patient on the waiting list gets a donor organ when one becomes available?

A
HLA typing:
-	HLA antigen type
-	HLA antibody screening
-	Lymphocyte cross matching- if recipient cells attack and kill donor cells this is indicative of rejection and transplantation should not go ahead
ABO blood group compatibility

HLA = chromosome 6

ABO []

277
Q
  1. This is a CT scan of a patient with a congenital renal abnormality.

Q1. Can you identify the problem?
Q2. What does this condition predispose to?
Q3. How might the patient present?

A

Q1
Likely to be polycystic kidney disease

Q2
Hypertension
Chronic renal failure
Recurrent UTIs
Renal stones
Preeclampsia during pregnancy
Liver cysts (most common extra-renal manifestation)
Berry aneurysm- leading to subarachnoid haemorrhage
Heart valve abnormalities (mitral valve prolapse)
Diverticulosis in patients who have had a renal transplant

Q3
Often asymptomatic
Patients often present in adulthood as the cysts slowly expand, compressing the parenchyma disrupting local control of blood pressure and impairing renal function
May present with chronic renal failure, hypertension, haematuria, loin pain or recurrent urinary tract infections

The other main possibility is a horseshoe kidney. Complications include pleviureteric junction (PVJ) obstruction, renal stones, infection, tumours and trauma. Patients will often present with hydronephrosis, obstructive symptoms or urinary retention.

NB- echogenic spaces within the kidneys on ultrasound is a characteristic finding of polycystic kidney disease

278
Q

What are the usual indications, in an otherwise fit person, for considering them for repair of an abdominal aortic aneurysm?

A

Abdominal aortic aneurysm (AAA) is a permanent pathological dilation of the aorta with a diameter > 1.5 times the expected AP diameter of that segment, given the patients sex and body size. The most commonly adopted threshold is a diameter of 3cm or more. More than 90% of aneurysms originate below the renal arteries.
The usual indications for considering repair of an abdominal aortic aneurysm are:
Ruptured or leaking aneurysm
Symptomatic aneurysm (usually abdominal, back and groin pain)
Aneurysm that is larger than 5.5 cm in males or 5cm in females
Rapidly expanding AAA (>0.5cm/year)
AAA associated with peripheral arterial aneurysm or peripheral artery disease
Saccular aneurysm (aneurysm that only bulges on one side of the artery wall)

279
Q

What are the features of a patient who presents with a rupturing Abdominal Aortic Aneurysm?

A

A patient with a ruptured AAA classically presents with a triad of acute abdominal pain (severe epigastric + or - back pain), abdominal distension and haemodynamic instability (hypotension, diaphoresis, tachycardia, tachypnoea) and possible cardiovascular collapse.
Other possible symptoms include flank or groin pain or an expansile, pulsatile abdominal mass.

Risk factors include; increasing age, male, smoker, FHx, atherosclerosis, hypertension, hypercholesterolaemia, other vascular aneurysms.

280
Q

A 68 year old man presents to ED with a 1 hour history of pain in the left side of his abdomen and mid back. It started suddenly, has not moved, and is getting worse. It is the worst pain he has experienced. He has a history of stable angina, managed with beta blockers and nitrates. On examination, he is pale, sweaty, has a pulse of 110bpm, blood pressure of 100/65mmHg, normal chest and cardiac exam. Abdominal exam reveals a tender mass in the epigastrium, which is pulsatile and expansile, peripheral pulses and neuro exam are normal.
What is the diagnosis? What investigations and treatments are needed?

A

The diagnosis is most likely to be a ruptured abdominal aortic aneurysm.

Urgent or emergent AAA repair is generally indicated for patients with ruptured AAA and diagnosis is therefore made from clinical picture in order to save time. No investigations are warranted except for those that facilitate the emergency surgery. These include UECs for renal function, coagulation screen, cross match, and ECG. If there is uncertainty in the diagnosis a FAST scan may be performed as long as this does not delay the time to theatre.

Initial management is to notify ED and surgical staff and the insertion of two large bore IV cannulas for fluid resuscitation while maintaining a permissive hypotension due to risk of worsening AAA with aggressive fluid resuscitation. The patient should receive oxygen and analgesia (morphine). Theatre should also be booked, consent for surgery attained, catheterisation and calling of the blood bank.

Two methods of aneurysm repair are currently available:
Open AAA repair- open aneurysm repair involves replacement of the diseased aortic segment with a tube or bifurcated prosthetic graft through a midline abdominal or retroperitoneal incision
Endovascular aneurysm repair- guide wire inserted in femoral artery or iliac artery up to the abdominal aorta followed by a sheathed stent graft, which is positioned with fluoroscopy. The stents are then deployed and the guide wires and removed.

281
Q

What are the treatment options for an otherwise fit 75 year old man who has been found to have a 6cm AAA (abdominal aortic aneurysm) on routine screening?

A

The size of the AAA (>5.5cm) indicates the need for surgery due to his 10-22% risk per year of AAA rupture.
Two methods of aneurysm repair are currently available:
Open AAA repair- open aneurysm repair involves replacement of the diseased aortic segment with a tube or bifurcated prosthetic graft through a midline abdominal or retroperitoneal incision
Endovascular aneurysm repair- guide wire inserted in femoral artery or iliac artery up to the abdominal aorta followed by a sheathed stent graft, which is positioned with fluoroscopy. The stents are then deployed and the guide wires are removed. This surgery requires fulfilment of anatomic criteria including a sealing zone above and below and adequate access vessels (commonly femoral and iliac) to accomodate stent graft.

If the patient declines surgery or is not fit for surgery watchful waiting and cardiovascular risk reduction (smoking cessation, BP control, lipid control and weight loss) is also an option.

NB EVAR has a lower mortality rate (0.5-2%) compared to open AAA repair

282
Q

What are the common complications of endovascular (stent) repairs of abdominal aortic aneurysms?

A

Complications associated with endovascular abdominal aortic repair are most commonly related to some technical aspect of the endograft placement and include:
Access site complications such as infection, haematoma, thrombosis of accessed vessels, pseudoaneurysm, dissection, distal embolism and AV fistula
Endograft complications include device migration, separation of components and endloeak, which is a term that describes the presence of persistent flow of blood into the aneurysm sac after endograft placement. This causes a continued risk of expansion and rupture. There can also be endograft infection with staphylococcus epidermidis or staphylococcus aureus and this can cause enterovascular fistula formation between the aorta and duodenum
Systemic complications such as cardiopulmonary, IV contrast nephropathy and allergy and ischaemia
Other complications such as abdominal compartment syndrome, post-implantation syndrome and conversion to open repair

Endoleak type I- incompetent seal at the proximal or distal
Endoleak type II- sac filling via a branch vessel
III- mechanical failure of the graft
IV- intentional leaking
V- continuos expansion of the aneurysm sac with evidence of a leak

283
Q

What are the main indications for Carotid endarterectomy (CEA, Carotid surgery)?

A

Carotid endarterectomy (CEA) is a surgical procedure used to treat carotid artery stenosis in which an incision is made in the anterior side of the neck, a filter and balloon catheter is used to removed the atherosclerotic plaque from the artery and the artery is closed with stitching or a graft. CEA is recommended for:

  • Symptomatic individuals with moderate (50%-70%) stenosis
  • Symptomatic individuals with high grade (70%-99%) stenosis.
  • Asymptomatic individuals with stenosis greater than 60% stenosis

Symptomatic is defined as causing focal neurological symptoms, sudden in onset and referable to the carotid artery distribution (ipsilateral to the significant carotid atherosclerotic pathology) with symptoms in the last 6 months. Symptoms include transient visual obscuration from retinal ischaemia, contralateral weakness or numbness of an arm, leg or face, visual field defect, dysarthria and aphasia.

The only absolute contraindication to CEA is asymptomatic individuals with complete carotid occlusion as the chance of stroke would be higher with CEA than without. Other relative contraindications include history of neck irradiation, concurrent tracheostomy, prior radical neck dissection, contralateral vocal chord paralysis, atypical surgically inaccessible lesion, unacceptably high medical risk.

[] double check
[] vascular runoff

284
Q

A 71 year old man presented to the ED with weakness and numbness in his left arm. These symptoms started suddenly 2 hours ago. His vision was not affected, and the numbness and weakness is getting better. He has no other symptoms, he is a lifelong smoker. Physical exam is normal, other than the numbness.

What is the likely diagnosis? What investigations are needed?

A

The diagnosis is likely a TIA.
Differentials include stroke, hypoglycaemia, seizure, complex migraine, subdural haematoma and brachial plexus injury.

Initial investigations include FBC, UEC, cholesterol studies, lipids, glucose and PT, INR and APTT and a CT brain.

A 12 lead ECG and carotid imaging with a carotid doppler ultrasound should also be performed.
As an outpatient Holter monitor testing should also be performed to check for AF and an echocardiogram with bubble study may be performed in patients with a high clinical suspicion of a cardiac shunt, valvular disease or intraventricular thrombus.

285
Q
  1. A 71 year old man presents to the ED with weakness and numbness in his left arm. These symptoms started suddenly 2 hours ago. His vision was not affected, and the numbness and weakness is getting better. He has no other symptoms, he is a lifelong smoker. Physical exam is normal, other than the numbness.

What is the likely diagnosis?

What are the risk factors for this disease?

A

The likely diagnosis is a TIA.
Differentials include stroke, hypoglycaemia, seizure, complex migraine, subdural haematoma, brachial plexus injury

Risk factors for TIA include:
Increasing age
Alcohol abuse
Smoking
Diabetes mellitus
Hypertension
Congestive heart failure
Carotid stenosis
Valvular disease- especially aortic disease
Atrial fibrillation
Patent foramen ovale
Hyperlipidaemia
Inactivity
Obesity
Hypercoagulability
286
Q

A 65 year old man presents to the emergency department with an 8 hour history of severe, generalised abdominal pain. Earlier in the day he passed fresh blood in his stool, there is no previous episodes of blood PR. His medical history included AF, Diabetes mellitus and hypertension. He is a long term smoker. He is restless with pain. His pulse is 110 irregularly irregular. His BP is 90/50mmHg. His temperature is 37.5C. Abdominal examination shows acute pain but a soft, non-rigid abdomen. Rectal exam shows fresh blood mixed into normal stools. His venous blood tests show a normal Hb, slightly raised white cell count (neutrophilia), a moderately raised CRP at 40, a near normal amylase and the rest are normal. His arterial blood gases show a pH of 7.29, lactate of 9.4 and base deficit of - 6.5, and a PaCO2 of 3.5.
What does the arterial blood gas show? What is the most likely diagnosis? What are reasonable differential diagnoses?

A
The arterial blood gas shows metabolic acidosis (as shown by low pH and base deficit), with lactic acidosis (elevated lactate), with partial respiratory compensation (low PaCO2). The most likely diagnosis is intestinal ischaemia, given the clinical history it is most likely to be secondary to emboli from atrial fibrillation.
The reasonable differential diagnoses include:
-	Infectious colitis
-	Ulcerative colitis or Crohn’s disease
-	Diverticular disease
-	Peptic ulcer disease
-	Acute pancreatitis
-	SBO or LBO
-	Gastroenteritis
-	Bowel perforation
-	Acute myocardial infarction
287
Q

A 65 year old man presents to the emergency department with an 8 hour history of abdominal severe, generalised abdominal pain. Earlier in the day he passed fresh blood in his stool, there is no previous episodes of blood PR. His medical history included AF, Diabetes mellitus and hypertension. He is a long term smoker. He is restless with pain. His pulse is 110 irregularly irregular. His BP is 90/50mmHg. His temperature is 37.5C. Abdominal examination shows acute pain but a soft, non-rigid abdomen. Rectal exam shows fresh blood mixed into normal stools. His venous blood tests show a normal Hb, slightly raised white cell count (neutrophilia), a moderately raised CRP at 40, a near normal amylase and the rest are normal. His arterial blood gases show a pH of 7.29, lactate of 9.4 and base deficit of - 6.5, and a PaCO2 of 3.5.
What does the arterial blood gas show? What is the most likely diagnosis? How should he be managed?

A

The ABG shows metabolic acidosis with lactic acidosis (elevated lactate) with partial respiratory compensation (low PaCO2). The most likely diagnosis is intestinal ischaemia. Given the clinical history it is most likely to be secondary to emboli from atrial fibrillation.
Management:
Primary survey and resuscitation and supportive measures:
supplemental oxygen
Fluids
Gastrointestinal decompression- NBM and NG tube
Haemodynamic monitoring and support
Correction of electrolyte abnormalities
Pain control- parenteral opioids
Anticoagulation- parenteral heparin
Broad spectrum antibiotics- acute mesenteric or colonic coverage (triple therapy)
Abdominal exploration, especially if peritoneal signs are present

NB- Intestinal ischaemia develops as a consequence of severe hypoperfusion leading to transmural necrosis of the bowel wall, which can progress to sepsis, peritonitis, free intrabdominal air or extensive gangrene. Surgery should not be delayed patients suspected of having intestinal infarction or perforation based upon clinical findings, regardless of aetiology. For patients with non-occlusive mesenteric ischaemia, surgical exploration should be limited to patients with peritoneal signs.

288
Q

You are the ED intern on duty. You are asked to see and treat a 75 yoa man who has presented with sudden onset of pain in his right leg. What are you going to do?

A

Primary survey assessing vitals and haemodynamic stability
Assess levels of pain and provide appropriate analgesia
History:
SOCRATES of pain and assess for the presence of rest pain
Past medical history- ulcers, gangrene, MI, carotid artery disease, AF, smoking history, diabetes, hypertension
Examination:
Cardiovascular examination
Neuromuscular examination- pain, paraesthesia, pulses, pallor, perishingly cold, paralysis
Investigations:
Bloods- FBC, UEC, LFTs, group and hold, coags, lipids, BGL
ECG- AF or AMI
Ankle brachial index
Call vascular surgeon as soon as possible, potentially start heparin, possible CT angiography

289
Q

You are the ED intern on duty. You have seen 75yoa man with a painful left leg of sudden onset. He has a cardiac history (AF and hypertension) and he smoked for 60 years. On examination he has a left cold leg from mid thigh downwards with no palpable pulses below the femoral pulse. His right leg is normal.

What is the likely diagnosis, what may have caused it and what are you going to do?

A

The likely diagnosis is an acute ischaemic limb, most likely due to an emboli and this case likely secondary to atrial fibrillation or could be from thrombosis due to atherosclerotic occlusion.
This is a surgical emergency and I would therefore call a vascular surgeon, call the operating theatre and call the anaesthetist. I would insert a cannula and correct any dehydration with fluids as hypotension is an exacerbating factor for an acute ischaemic limb. I would also give IV heparin and consider a CT angiography depending on the surgeons advice. I would also keep the patient NBM and give appropriate analgesia. I would take bloods including an FBC, UEC, CK, coagulation profile, lactate (which may be helpful in assessing and tracking the degree of tissue hypoperfusion) and group and save. I would also perform an ECG to check cardiovascular function and determine if AF is present at the moment, making an embolic cause of ischaemia more likely.

Causes of ischaemic limb:
Embolisation- AF, valvular vegetation, tumours such as an atrial myxoma (benign primary heart tumour most commonly found in the atrium), atherosclerotic, aneurysm
Thrombosis- atherosclerosis or occlusion of a bypass graft
Trauma- Blunt, penetrating or intravenous drug use
Rarer- hypoperfusion, vasculitis, drugs, aortic dissection, venous gangrene

Presentation of ischaemic limb:
Pain, pallor, pulselessness, perishingly cold, paraesthesia and paralysis. Paraesthesia and paralysis indicate a threatened limb that requires emergency surgical review, regardless of the cause.

290
Q
  1. You are the ED intern on duty. You have seen 75yoa man with a painful left leg of sudden onset. He has a cardiac history (AF and hypertension) and he smoked for 60 years. On examination he has a left cold leg from mid thigh downwards with no palpable pulses below the femoral pulse. His right leg is normal.

What are the clinical signs that you will assess to determine whether the leg is salvageable, or that it is beyond help and needs amputation?

A
Determination of whether a leg is salvageable depends on the viability of the tissue. This is determined based on clinical findings and investigations including:
Pain
Capillary refill time
Motor deficit
Sensory deficit
Arterial doppler findings
Venous doppler findings

If tissue is not viable it needs amputation. However, tissue commonly presents as threatened and revascularisation with heparin infusion is continued post-operatively in continued revascularisation attempts.
The ischaemic area is initially white but later becomes mottled/ livedo reticularis. If this blanches on pressure the limb is still viable. If the mottling does not blanch then the limb is not viable.

291
Q
  1. A 54 year old insulin diabetic presents to the ED complaining of increasing pain in her right foot that has been developing over 1 week. It is worse at night, partially relieved by hanging her foot over the side of the bed. Her right great toe has become swollen, red in parts and discoloured in others. She is febrile (37.7C), pulse 86 regular, BP 130/80mmHg, blood glucose 13.2mmol/L. Femoral pulses are present bilaterally, but no distal pulses can be felt below this, the right great toe is erythematous with a large fluctuant swelling at the base.

What does this clinical appearance suggest?
What investigations are needed?
What treatment is needed?

A

The clinical appearance in this patient suggests peripheral vascular disease with critical limb ischaemia (rest pain relieved with dependency), an abscess at the base of the right toe with possible cellulitis surrounding the abscess and poorly controlled diabetes.
Investigations include:
FBC, UEC, blood cultures, coags, LFT, HBA1c
Foot x-ray looking for osteomyelitis
ECG
ABI (the ratio of the blood pressure at the ankle to the blood pressure in the arm, with lower blood pressures at the ankle suggesting peripheral artery disease), duplex doppler ultrasound, ?CT angiogram

Treatment needed is three pronged:
For the abscess this patient needs incision and drainage, wound debridement and antibiotics, likely iV tazocin in diabetic foot infection patients but depends on the swab results
Diabetes control with lifestyle modification, inpatient endocrine consult and management of insulin dosing
Control of ischaemic limb with vascular surgeon for possible intervention such as angioplasty or bypass (if arterial thrombosis is present).

292
Q

What factors will a vascular surgeon consider when planning treatment for peripheral vascular disease in a patient’s foot?

A

Management considerations for peripheral vascular disease:
Is the limb viable or not? Is revascularisation possible or is there a need for amputation
Chronic presentation:
Disease activity- claudication (does it limit lifestyle) and is there critical limb ischaemia
Age and expected lifespan
Patient compliance to medical therapy and lifestyle modifications
Comorbidities such as diabetes

Treatment
Mild-moderate claudication- stop smoking, exercise, statin, aspirin, weight loss, BP control. consider balloon angioplasty
Disabling claudication- balloon angioplasty or reconstructive arterial surgery
Critical ischaemia- IV drug therapies such as prostacyclin (vasodilation), balloon angioplasty, reconstructive arterial surgery, amputation, appropriate analgesia

293
Q

How does a vascular surgeon decide the level of amputation if one becomes necessary for peripheral vascular disease?

A

While determining the level of amputation several factors are taken into account including:
Physical factors- tissue necrosis or infection, transcutaneous oxygen level and circulation of the extremity
Co-existing diseases- diabetes, peripheral vascular disease and other systemic infections
Experience and skills of the surgeon
Nutritional status of the patient

Some of the important principles include:
The amputation is made through a level of healthy tissue to ensure all of the necrosis is removed
Fitting of a prosthetic limb. Therefore common levels of amputation include transferral (just above the knee), transtibial (just below the knee), Syme’s malleolar amputation and mid tarsal amputation.

294
Q

What anatomical levels are commonly used for amputations in the lower limb for peripheral vascular disease?

A
Above knee femoral amputation
Below knee tibial amputation
Knee disarticulation/ through knee
Syme’s malleolar amputation
Mid tarsal amputation
295
Q

A 59 yoa woman presents to the emergency department with pain and tingling in her right hand and forearm. It started suddenly 4 hours ago. It has improved, and the pain is gone, but the numbness remains. There is no history of trauma and no previous episodes. She has no significant medical history, but is waiting to see a cardiologist for “palpitations”. On examination the right hand and forearm are cool to touch. The radial and arterial pulses are absent on that side, pulses elsewhere are normal. Sensation is mildly reduced, but she has full ranges of movement in the hand.
What is the likely diagnosis? What investigations are needed?

A

The most likely diagnosis is acute upper limb ischaemia (possibly secondary to AF)

Investigations include a cardiovascular examination, difference in blood pressure between the two arms or an ankle brachial index (ABI), bloods (FBC, UEC, coags, thrombophilia screen, CRP), ECG, duplex ultrasound of upper limb and CT angiogram.

296
Q

A 59 yoa woman presents to the emergency department with pain and tingling in her right hand and forearm. It started suddenly 4 hours ago. It has improved, and the pain is gone, but the numbness remains. There is no history of trauma and no previous episodes. She has no significant medical history, but is waiting to see a cardiologist for “palpitations”. On examination the right hand and forearm are cool to touch. The radial and arterial pulses are absent on that side, pulses elsewhere are normal. Sensation is mildly reduced, but she has full ranges of movement in the hand.

What is the likely diagnosis?
What are the common causes of this problem?
How are they diagnosed?

A

The most likely diagnosis is acute upper limb ischaemia.
Causes of ischaemic limb:
Embolisation- AF, valvular vegetation, tumours such as an atrial myxoma (benign primary heart tumour most commonly found in the atrium), atherosclerotic, aneurysm
Thrombosis- atherosclerosis or occlusion of a bypass graft
Trauma- Blunt, penetrating or intravenous drug use
Rarer- hypoperfusion, vasculitis, drugs, aortic dissection, venous gangrene

Diagnosis includes:
History:
Palpitations, family Hx of AF, known AF or valvular disease
Diabetes, hypertension, smoking, alcohol, sedentary lifestyle
History of trauma
Intravenous drug use

Examinations
Cardiovascular examination

Investigations
ECG
Echocardiogram
Doppler ultrasound

297
Q

A 59 yoa woman presents to the emergency department with pain and tingling in her right hand and forearm. It started suddenly 4 hours ago. It has improved, and the pain is gone, but the numbness remains. There is no history of trauma and no previous episodes. She has no significant medical history, but is waiting to see a cardiologist for “palpitations”. On examination the right hand and forearm are cool to touch. The radial and arterial pulses are absent on that side, pulses elsewhere are normal. Sensation is mildly reduced, but she has full ranges of movement in the hand.

What is the likely diagnosis?
What are the treatment options?

A

The most likely diagnosis is acute upper limb ischaemia possibly secondary to atrial fibrillation.
Treatment options angioplasty, stenting, carotid-subclavian bypass or embolectomy.
Treatment should also include secondary prevention measures such as:
Control of hypertension
Control of AF with anticoagulation therapy (warfarin or apixaban)
Lipid modification
Smoking cessation
Antithrombotic therapy (anticoagulants or anti platelets)